Vous êtes sur la page 1sur 162

NAVIGUER

AU FIL

SH O R
CU TST

DES

GRANDES

IDÉES
Comment le cerveau construit-il
la mémoire ? Les humains sont-ils tous
des moutons ? Comment un enfant
voit-il le monde ? Peut-on vivre de pain
et d’eau fraîche ? Pourquoi sommes-nous
irrationnels ? Quel est le raccourci vers
le bonheur ?
Illusions d’optique et effet de désinformation, attitudes
positives et pensées négatives, anxiété sociale et
constructions personnelles : la connaissance de plus en
plus fine de l’esprit humain révèle un territoire complexe,
mouvant, mais véritablement fascinant à explorer. Alors
comment s’y retrouver, comprendre et relier les notions ?
Avec des cartes bien sûr !

Ce livre aborde 8 thématiques en lien avec la psychologie et


pour chacune, propose une carte mentale permettant de se
familiariser avec les concepts, suivie d’une série de questions-
réponses, avec des illustrations, pour approfondir !

Avis aux curieux ! La collection Short Cuts vous invite à


tracer vous-même votre chemin au travers des grands
concepts de notre temps. Les cartes mentales vous
permettent de visualiser et de comprendre les notions,
et vous pourrez ensuite naviguer à votre gré pour trouver,
en texte et en images, toutes les réponses à de multiples
questions… Bon voyage !

978-2-7598-3096-1

9 782759 830961 www.edpsciences.org


SH OR
CU TST
Transation from the English anguage edition of:
short cuts PSYCHOLOGY - © UniPress Books Ltd 2023

Directeur artistique : Luke Herriott


Éditeur consultant : Dr Jennifer Wild
Éditrice d’acquisition : Kate Duffy
Illustrateur : Robert Brandt

Traduction et mise en page de l’édition française : Benjamin Peylet

ISBN (papier) : 978-2-7598-3096-1


ISBN (ebook) : 978-2-7598-3097-8

Imprimé en Serbie

Copyright © EDP Sciences 2023

Tous droits de traduction, d’adaptation et de reproduction par


tous procédés, réservés pour tous pays. La loi du 11 mars 1957
n’autorisant, aux termes des alinéas 2 et 3 de l’article 41, d’une part,
que les « copies ou reproductions strictement réservées à l’usage
privé du copiste et non destinées à une utilisation collective », et,
d’autre part, que les analyses et les courtes citations dans un but
d’exemple et d’illustration, « toute représentation intégrale, ou
partielle, faite sans le consentement de l’auteur ou de ses ayants
droit ou ayants cause est illicite » (alinéa 1er de l’article 40). Cette
représentation ou reproduction, par quelque procédé que ce soit,
constituerait donc une contrefaçon sanctionnée par les articles 425
et suivants du code pénal.
SH O R
CU TST

PSYCHOLOGIE
CONTENTS
INTRODUCTION 6

1 PSYCHOLOGIE COGNITIVE 8

La perception est-elle juste une expérience? 14


Voyons-nous bien ce que nous voyons ? 16
Comment le cerveau construit-il a mémoire ? 18
Peut-on compter sur votre témoignage ? 20
Comment sont stockés nos souvenirs ? 22
Y a-t-il foule dans nos têtes ? 24
Le raccourci est-il a voie du danger ? 26

2 PSYCHOLOGIE SOCIALE 28

Affichons-nous tous les mêmes émotions ? 34


Interviendriez-vous en cas de meurtre ? 36
Les humains sont-ils tous des moutons ? 38
Quand cessons-nous de suivre les ordres ? 40
Est-ce toujours «eux» contre «nous»? 42
Comment les idées s’imposent-elles? 44
Pourquoi as-tu fait ça? 46

3 APPRENTISSAGE 48

Salivez-vous quand sonne l’heure du dîner? 54


Apprenons-nous de nos récompenses? 56
Apprendre est-il un jeu d’imitation? 58
Dix mille heures de pratique feront-elles de vous un génie? 60
Comment un enfant voit-il le monde? 62
Quand dire bye bye à l’apprentissage d’une angue? 64
Peut-on améliorer sa mémoire? 66

4 PSYCHOLOGIE BIOLOGIQUE 68

Avons-nous bien cinq sens? 74


Vous vous sentez bien? 76
Travaillons-nous mieux sous a pression? 78
Comment les drogues font-elles paner? 80
Comment reconnaître un visage? 82
Comment un taxi se souvient des raccourcis? 84
5 PSYCHOLOGIE DU DÉVELOPPEMENT 86
Est-ce que le stress coule aussi les marins? 92
Doit-on être sûr quand on s’attache? 94
Quel âge a l’ego? 96
Pourquoi les bébés aiment-ils les hochets? 98
La sociabilité est-elle un jeu d’enfant? 100
Les enfants lisent-ils dans les pensées? 102
Quel est votre rôle dans le théâtre de a vie? 104

6 DIFFÉRENCES INDIVIDUELLES 106

Peut-on vivre de pain et d’eau fraîche? 112


Sommes-nous plus intelligents qu’avant? 114
Combien y a-t-il d’intelligences? 116
Un bon chef peut-il montrer ses émotions? 118
Comment mesurer a personnalité? 120
Construisons-nous notre propre monde? 122

7 THÉRAPIE 124

Comment rencontrer son moi idéal? 130


Comment on s’adapte? 132
D’où viennent les pensées négatives? 134
Pourquoi sommes-nous irrationnels? 136
Le compor tement modifie-t-il les pensées? 138

8 PSYCHOLOGIE POSITIVE 140

Que valez-vous? 146


Quel est le raccourci vers le bonheur? 148
Changer de mentalité change-t-il l’esprit? 150
Peut-on s’épanouir grâce à a psychologie? 152
Doit-on se aisser porter par le flow? 154

POUR EN SAVOIR PLUS 156


INDEX 158
REMERCIEMENTS 160
ITRODUC
L
a psychologie touche à tous les domaines de notre
développement, depuis les liens que nous tissons
enfant à nos manières de penser et d’interagir. Les
découvertes de cette science ont un impact direct
sur nos vies. La comprendre, c’est comprendre ce que c’est
qu’être humain. Quelles sont les théories et les conclusions
qui nous touchent le plus directement? Que signifient-
elles? Pour le savoir, il faut emprunter un raccourci. Voilà
ce que ce livre vous propose: vous aider à comprendre
comment a psychologie, appliquée à certains des
problèmes les plus complexes, guide notre évolution.
Le parcours commence avec a psychologie cognitive,
les procédés par lesquels nous voyons, pensons et décidons.
Nous découvrirons des vérités dérangeantes sur notre
mémoire, qui font que nous ne sommes pas des témoins
fiables. Nous passerons ensuite au pouvoir du collectif, au
domaine de a psychologie sociale. Comment et pourquoi les
foules forment nos opinions, avec les conséquences parfois
terribles que l’on sait? Nous en apprendrons plus sur nos
façons d’apprendre, a fabrication de a mémoire et souvent
de faux souvenirs. Nous verrons certains dérèglements,
dont a synesthésie, quand le cerveau méange les sens, et a
prosopagnosie qui nous empêche de reconnaître les visages.
Nous nous pencherons ensuite sur a biologie, où nos
sens n’en auront plus beaucoup: il y en a plus de cinquante!

6
TIO
Nous apprendrons quel est le niveau de stress idéal, et
comment les psychologues le mesurent. Nous verrons aussi
ce qu’il se passe dans le cerveau quand nous prenons des
drogues, des antidépresseurs au cannabis. Nous parlerons
de jeux d’enfants et retournerons vers nos jeunes années
pour découvrir comment les liens familiaux façonnent nos
stratégies de résilience bien des années plus tard.
Nous verrons ensuite ce qui fait de nous des individus,
ce qui sépare le bon du grandiose, ce qui nous pousse.
Nous parlerons d’intelligence, de ces nombreux types,
dont celle des dirigeants à poigne. Nous présenterons nos
modes de construction de a réalité et a manière dont a
science s’applique à a santé mentale pour développer des
traitements efficaces. Nous verrons comment a thérapie
comportementale et cognitive transforme les patients
atteints de dépression ou de trouble de l’anxiété en
scientifiques qui testent et mettent à jour leurs pensées grâce
à des expériences, une approche qui a permis des guérisons
durables. Enfin, notre parcours prendra fin sur les hauteurs,
a psychologie positive et les ingrédients du bonheur, les
manières de s’épanouir et former un esprit heureux.
Ce livre parle de notre développement, de notre
évolution tout au long de a vie, et des contributions qu’a
apportées a psychologie à sa compréhension.
Bon voyage!

7
CHAPITRE 1 PSYCHOLOGIE
COGNITIVE

PERCEPTION

PRÉCONCEPTIONS

TRAITEMENT
DESCENDANT

SOUVENIRS

DISSONANCE
COGNITIVE
CHAPITRE 1

COGITIE
ATTENTES
PSYCHOLOGIE

PENSÉE DE
GROUPE

BIAIS
INTRODUCTION

C
omment nous forgeons-nous notre expérience?
Selon les psychologues, cea revient à se demander
comment nous percevons le monde, fabriquons
nos souvenirs et nos décisions, nos pensées. Dans
ce chapitre, nous découvrons les mécanismes qui nous
permettent de donner du sens à ce qui nous entoure.
Nous sommes bombardés d’informations sensorielles en
permanence. Pour leur donner du sens, le cerveau travaille à
120mètres par seconde, il teste des HYPOTHÈSES et met à
jour ses conclusions à mesure que les nouvelles informations
affluent. C’est le TRAITEMENT DESCENDANT, où ce que
nous savons déjà nous permet d’interpréter une information
nouvelle, où nous voyons ce que nous nous attendons à voir.
La THÉORIE ÉCOLOGIQUE DE GIBSON suggère plutôt qu’un
TRAITEMENT ASCENDANT serait à l’œuvre, où l’information
proviendrait de l’environnement, voyagerait jusqu’à l’œil puis
au cerveau. Selon cette théorie, il faut percevoir le monde
pour survivre, nous nous y efforçons donc dès a naissance.
Nos souvenirs jouent un rôle important dans nos
perceptions. Mais sont-ils bien fiables? En étudiant les
incohérences dans les souvenirs de grands événements,
Neisser a observé qu’ils se modifiaient avec le temps. Les
travaux de Nader et LeDoux suggèrent que nous réécrivons
nos souvenirs à chaque remémoration, et que cette réécriture
rempace le souvenir d’origine. Selon a THÉORIE DE LA
CONSOLIDATION, si nous souhaitons nous rappeler un
événement, a trace qu’a aissée son souvenir dans le cerveau
doit être consolidée sans cesse, sans quoi elle sera effacée.
Nos souvenirs sont fragiles.

10 PSYCHOLOGIE COGNITIVE
Loftus a découvert que bien des choses altéraient
les souvenirs, dont nos attentes, nos biais, et même notre
angage. L’emploi de mots forts, par exemple parler de
«crash» au témoin d’un accident de a route pourra donner
naissance à des souvenirs d’une scène plus intense que si
un angage plus neutre avait été employé, par exemple en
lui parant d’une simple «collision». Pour cette raison, les
témoignages ocuaires ne sont pas toujours fiables, car il est
possible de se constituer un FAUX SOUVENIR par suggestion.
Les théories qui s’intéressent au stockage des souvenirs
devraient expliquer a manière dont nous les évoquons, mais
ce mécanisme est encore sujet à débat. On parait avant de
mémoire à long terme ou à court terme. Toutefois, Baddeley
a dépoussiéré cette idée en parant d’une mémoire de
travail capable de stocker et de traiter l’information à court
terme. En 1972, Craik et Lockhart proposent un modèle de
traitement par a mémoire, dans lequel un lien s’établit entre
a profondeur du traitement et a durée des souvenirs.
Du côté de nos modes de pensée, Janis a découvert
combien il était facile à un groupe de former les opinions
en son sein. En groupe, nous écartons plus facilement les
perspectives minoritaires, car il est plus facile de se fondre
dans le collectif que de s’opposer ouvertement et ainsi de
se distinguer. La pression du groupe pèse sur nos pensées
et nos décisions. En situation de stress, le cerveau prendra
des raccourcis de ce genre et se fiera à l’intuition plutôt
qu’à l’éaboration d’une pensée logique. C’est risqué, mais le
raccourci est parfois le meilleur chemin.

PSYCHOLOGIE COGNITIVE 11
PSYCHOLOGIE COGNITIVE
MÉTHODE
HYPOTHÈSES
Prédictions fondées sur nos
prévisions de ce qui pourrait
advenir.

INFÉRENCES THÉORIE ÉCOLOGIQUE


INCONSCIENTES DE GIBSON
Influence de la mémoire L’information naît dans
et de l’expérience sur la l’environnement; nous devons
perception, qui se fondent percevoir pour survivre, nous le
avec les stimuli entrants faisons donc dès la naissance.
pour créer une image
complète (Helmholtz).

PERCEPTION
Mécanisme actif
impliquant la mémoire
et l’expérience, plutôt
TRAITEMENT qu’acceptation passive
DESCENDANT des stimuli entrants
Emploi de ce que nous savons (Helmholtz).
déjà pour interpréter de
nouvelles informations ; nous
voyons ce que nous nous
attendons à voir (Gregory).

SCHÉMA
Représentation mentale de
nos expériences, fondée sur
nos expériences passées et
notre mémoire.

ILLUSION
AFFORDANCE
Opportunité d’action fournie par
l’environnement, qui donne du
sens. Un vélo est une opportunité
de voyager (Gibson).

MODÈLE D’ATKINSON -
SHIFFRIN
Existence de trois compartiments
de la mémoire : 1) sensorielle,
2) à court terme, 3) à long terme.
L’information est transférée entre
ces trois compartiments, dans
l’ordre.

THÉORIE DE LA
CONSOLIDATION
Chaque remémoration est une
TRAITEMENT ASCENDANT réécriture. Les souvenirs à long
La perception est ascendante, terme doivent être consolidés
depuis l’environnement à l’œil par la remémoration ou sont
puis au cerveau (Gibson). effacés (Nader et LeDoux).

EFFET DE DÉSINFORMATION
Contamination d’un souvenir
existant par des informations FAUX SOUVENIRS
trompeuses a posteriori. Souvenirs qui nous ont été
Le souvenir d’un témoin altéré par suggérés et nous paraissent réels,
un interrogatoire biaisé. mais ne le sont pas (Loftus).

DISSONANCE COGNITIVE
Tenir deux positions
contraires en même temps,
cause de décalages entre
comportement et croyances
entraînant un stress
psychologique (Festinger).

PENSÉE DE
GROUPE
Les opinions
individuelles sont noyées
CYCLE PERPÉTUEL dans le consensus, car il est
Nos expériences du monde sont plus facile de se taire que
formées de la relation entre de se distinguer. Cela
des schémas et des signaux de peut entraîner des
l’environnement; ces signaux problèmes (Janis).
externes influencent nos schémas
internes (Neisser).
L percepion es-ee
se ne expérience?
Peut-être. Lors de leurs expériences,
les scientifiques testent des hypothèses, des
prédictions de ce qu’il pourrait se passer.
Le cerveau semble marcher de a même façon,
il teste des hypothèses et se sert des résultats
pour construire l’image perçue.

Nos yeux reçoivent des données nos cerveaux. Nous créons des hypothèses
visuelles, nos oreilles des sons, nos de ce que nous voyons fondées sur des
nez se froncent pour une odeur. Les connaissances et des expériences antérieures.
psychologues se sont beaucoup demandé Le cerveau devine plus qu’il ne voit et met à
comment ces signaux sensoriels sont traduits jour ses hypothèses à mesure que de nouvelles
quand nous percevons. Comment, par exemple, informations l’atteignent. Selon ce modèle,
reconnaissons-nous cet objet flou et poilu qui a perception est une forme de traitement
court derrière un bâton comme un chien ? Ou descendant, dans lequel ce que nous savons
cet ensemble de molécules odorantes comme déjà sert à interpréter ce que nous ne savons
le parfum d’une rose? pas encore.
En 1867, l’Allemand Hermann von Helmholtz Gregory disait que certaines illusions
a perçu l’importance des inférences d’optique telles que le cube de Necker et
inconscientes. La perception, dit-il, n’est pas l’illusion du masque démontrent le traitement
l’acceptation passive de stimuli entrants, mais descendant de l’information sensorielle.
un procédé actif auquel a mémoire autant Ainsi, quand un masque en 3D de Charlie
que l’expérience prennent part. Chaplin tourne lentement devant nous, arrive
En 1970, le psychologue britannique un moment où l’arrière concave du masque
Richard Gregory a développé cette idée et s’inverse pour former un visage convexe qui
affirmé que notre perception de a réalité était tourne dans le sens opposé. Notre cerveau
une construction. Il avançait que nos sens s’attend à un visage convexe, bien plus
sont bombardés de signaux souvent ambigus fréquent que les visages concaves, et il nous
et difficiles à interpréter. Pour leur donner impose donc cette interprétation. Parfois, nous
du sens, nous nous appuyons beaucoup voyons ce que nous nous attendions à voir,
sur les informations déjà disponibles dans plutôt que ce qui est vraiment devant nous.

14 PSYCHOLOGIE COGNITIVE
TRAITEMENT DESCENDANT

Voici le cube de Necker. Que voyez-vous?


Fixez-le un moment et l’image deviendra
instable. Le carré sombre passera devant ou
derrière, car ce schéma unique est capable de
générer plusieurs interprétations alternatives.
Selon Gregory, cea se produit parce que
notre cerveau développe deux hypothèses
également pausibles pour expliquer ce que
nous voyons ici, et qu’ il est incapable de
départager. Le traitement descendant doit
être à l’origine de l’ illusion, car les données
sensorielles sont les mêmes tout du long.
oons-nos bien ce
qe nos oons?
Selon une école de pensée, oui! Notre
expérience visuelle du monde est en majorité
le produit de l’information que captent nos
yeux, plutôt que de préconceptions déjà
présentes dans nos cerveaux.

La perception visuelle est un drôle bas en haut, depuis l’environnement jusqu’à


de phénomène. Comment les l’œil puis au cerveau. Nous percevons ce
particules de lumière qui frappent que nous voyons, pas ce que nous pensons
notre rétine sont-elles converties en voir. L’environnement était si important dans
perceptions, de gens, de bâtiments, de vélos cette théorie qu’elle a été appelée a théorie
et autres objets? Est-ce que a perception écologique.
fonctionne de façon descendante, avec a Gibson pensait aussi que a perception
mémoire et les connaissances préaables était liée à l’incarnation. Quand nous
s’imposant aux stimuli visuels, ou de façon percevons des objets, nous les voyons selon
ascendante, où ce que nous voyons vraiment les possibilités d’interactions qu’ils nous
jouerait un rôle majeur, comme le pensait offrent. Une affordance, comme il l’appeait,
James J. Gibson dans les années1960 et1970. est une opportunité d’action: une bouilloire
Selon Gibson, l’expérience ordinaire, pour faire le thé, un vélo pour se dépacer,
quotidienne, nous donne bien assez un biscuit pour manger, plutôt qu’une
d’informations pour que nous puissions représentation du monde indépendante de
interpréter ce que nous voyons. Il est nos actions. Selon lui, le sens est donc ce que
inutile d’ajouter à cea un traitement ou l’environnement «permet» à l’observateur.
une interprétation supplémentaire. Quand Selon a théorie de Gibson, aucune
nous nous dépaçons, les données visuelles expérience préaable, aucun apprentissage
changent. Les objets, par exemple, sont vus à n’est requis pour que a perception ait lieu.
différentes distances et sous différents angles, Le processus est inné. Il a été forgé
ce qui nous fournit une mine d’informations par évolution, car il offre un avantage
qui forgent directement notre perception. La substantiel sur le pan de a survie. Afin de
perception est donc simplement l’interaction répondre correctement aux dangers de
entre l’environnement et nous. Gibson croyait l’environnement, un organisme doit d’abord
que le traitement s’effectuait directement, de le percevoir correctement.

16 PSYCHOLOGIE COGNITIVE
TRAITEMENT ASCENDANT

Selon Gibson, a perception est ascendante, de l’objet, par exemple sa taille et son
elle va de l’environnement jusqu’ à l’œil puis empacement. En nous dépaçant au sein
au cerveau. Quand nous voyons une rose, c’est de l’environnement, l’information change et
ça que notre cerveau enregistre. La lumière génère le carburant essentiel à a perception.
dans nos yeux produit une information Aucune connaissance ni expérience préaable
non ambiguë quant aux qualités précises n’est requise.
Commen e cere
consri-i  mémoire?
La création de souvenirs est un
processus actif de construction, pas a
reproduction passive du passé. Ainsi, nos
souvenirs changent avec le temps.

Les souvenirs sont construits dans le conseiller de Richard Nixon, John Dean, sur
cerveau, c’est certain, mais l’étendue le scandale du Watergate. En comparant le
de l’influence des événements détail du témoignage aux transcriptions de
extérieurs sur leur construction ne l’est pas. conversations enregistrées, des différences
Quand Ulric Neisser s’est attaqué à ce débat, il sont apparues. Dean tendait ainsi à dramatiser
y a plus de 40ans, il a insisté sur l’influence les événements, à exagérer son rôle et à fondre
des signaux externes sur nos mécanismes des événements distincts en un seul souvenir.
cognitifs. La perception, selon lui, est le Selon Neisser, les erreurs de ce type sont
processus actif qui forge notre expérience du fréquentes. Elles se produisent parce que les
monde, une interaction entre les idées et les souvenirs sont des constructions actives, pas
hypothèses de notre cerveau, ce qu’il appelle des enregistrements passifs. Son étude de
le «schéma», et les signaux externes. 1986 sur le désastre de a navette Challenger
Autrement dit, les traitements ascendant et donne a même conclusion. Les récits des
descendant sont importants pour a étudiants sur le drame donnés trois ans plus
perception, l’environnement revenant nourrir tard différaient grandement de ceux fournis le
et influencer le schéma interne. C’est ce qu’on lendemain de l’explosion.
appelle le cycle perpétuel. La mémoire, selon Neisser, n’est pas un
En tournant son attention vers a mémoire, cliché instantané précis et figé dans le temps.
Neisser a réalisé que les psychologues Pour se rappeler un événement, l’esprit doit
apprendraient peu d’études en aboratoires activement reconstruire le passé, un procédé
conçues à partir de listes de mots à mémoriser. inévitablement influencé par les expériences
Si l’environnement était aussi important suivantes. En conséquence, les souvenirs sont
qu’il le pensait, a mémoire devait être fréquemment déformés, ce qui peut poser
étudiée dans a vraie vie. Dans son étude de problème, par exemple pour les témoignages
1981, Neisser a analysé le témoignage d’un ocuaires lors de procès.

18 PSYCHOLOGIE COGNITIVE
MUR DE LA MÉMOIRE
Neisser a avancé que, tout comme un
artisan construit une maison, nos cerveaux
construisent nos mémoires. Supposez que
cinq ans plus tard vous redemandiez au
même artisan de créer une copie de a
maison, sans lui fournir les pans originaux.
La reconstruction sera sembable, mais
pas identique. De même, les souvenirs
d’ événements passés sont construits, et eux
aussi perdent en précision avec le temps.
Pe-on comper sr
ore émoine?
Probablement pas. Nous nous fions à
notre vue plus qu’à tout autre sens, mais
sommes en réalité souvent trompés par
l’interprétation que notre cerveau fait des
événements, et par nos souvenirs.

Un témoignage ocuaire peut avoir un témoin contient une erreur, cea pourra
beaucoup de poids. Quand une modifier ses souvenirs des événements. Et cet
victime identifie avec certitude son effet de désinformation peut tout toucher,
agresseur au sein d’une sélection de photos, depuis l’apparence du suspect jusqu’à a
ou qu’un témoin jure au cours d’un procès disposition exacte de a scène du crime.
avoir vu l’accusé commettre le crime dont on Un souvenir déformé est déjà difficile à
l’accuse, a police et les juges les prennent au admettre, mais, comme Loftus et d’autres
sérieux. Le problème, c’est qu’un témoignage l’ont montré, nous pouvons même abriter
ocuaire n’est parfois pas fiable du tout. des souvenirs complètement inventés. De
La psychologue cognitive américaine faux souvenirs, qui nous ont été suggérés et
Elizabeth Loftus a défriché ce champ d’études peuvent sembler très authentiques, mais qui
dès 1979 dans son livre Témoignage ocuaire. ne le sont pas du tout.
Comme l’ont confirmé d’innombrables études Les dangers de se reposer uniquement
par a suite, de nombreux facteurs jouent sur sur des témoignages ocuaires sont grands.
ce qu’un témoin pense avoir vu. Une mauvaise Une étude de 2011 aux États-Unis a conclu
visibilité, n’avoir perçu qu’une silhouette, le que, dans les cas de révision de procès suite
stress, les attentes qui pèsent sur lui et ses à de nouvelles preuves ADN, le verdict initial
propres biais, dont les stéréotypes racistes. fautif reposait sur des témoignages ocuaires
Autre gros problème: nos souvenirs trois fois sur quatre. De tels témoignages ont
sont malléables. Des centaines d’études envoyé pendant des décennies des innocents
ont montré que le seul fait d’être interrogé dans les prisons et les couloirs de a mort.
impante de nouveaux souvenirs chez un Nous devons être prudents quand il
témoin et modifie les souvenirs existants. s’agit de faire confiance à des témoignages
Par exemple, si a question qu’on pose à ocuaires, surtout quand ce sont les nôtres.

20 PSYCHOLOGIE COGNITIVE
EFFET DE DÉSINFORMATION

»
/h
km

»
50

CT
«

TA
N
O
«C

C»
»

O
/h

H
km

C
«
60
«
»
SH
RA
«C

»
/h
km
70
«

Dans une des premières expériences de hausse des vitesses estimées par rapport
Loftus et Palmer, les participants regardaient aux questions formulées dans des mots plus
une vidéo d’accident de a route avant de neutres, comme «collision», ce qui montrait
répondre à des questions sur a vitesse des bien que a mémoire pouvait être manipulée
voitures impliquées. Celles contenant des facilement, simplement en modifiant
mots comme «crash» entraînaient une les termes de a question.
Commen son socés
nos soenirs?
D’abord, le cerveau crée un instantané
de l’information qui nous entoure. Puis les
neurones s’allument et se câblent ensemble,
un nouveau schéma de souvenirs est ainsi
stocké temporairement dans l’hippocampe,
puis dans les replis du cortex.

Pour comprendre où sont stockés nos Afin de constituer un souvenir à long terme,
souvenirs, les psychologues ont dû le cerveau prend un instantané de l’information
analyser les types de souvenirs créés entrante qu’il soumet à une série de changements
et les parties du cerveau concernées. successifs puis à un procédé de stabilisation
En 1968, Richard Atkinson et Richard Shiffrin appelé a consolidation. Des études montrent
ont proposé trois mémoires différentes. La que de nouveaux apprentissages, des lésions à
mémoire sensorielle enregistre une impression l’hippocampe, ou des médicaments bloquant a
passagère issue de sens; a mémoire à court synthèse de protéines peuvent perturber cette
terme (MCT) peut temporairement contenir de consolidation.
plus gros pans d’information, et a mémoire à Nous avons longtemps pensé qu’un
long terme (MLT) peut en stocker des quantités souvenir à long terme était comme un fichier
encore plus grandes, pour toujours. permanent que le procédé de remémoration
Le modèle d’Atkinson et Shiffrin pose alait piocher dans les replis du cortex, avant de
l’hypothèse que l’information serait transférée l’y repacer intact après usage.
entre a mémoire sensorielle, a MCT et a MLT, Toutefois, au début du XXIesiècle, Karim
dans cet ordre, et que les souvenirs lointains Nader et Joseph LeDoux ont remis en question
pouvaient être transférés dans le stockage à cette idée de permanence du souvenir avec
court terme au besoin. des expériences qui en montraient au contraire
En 1972, Endel Tulving a distingué différents a fragilité. Selon a théorie de a consolidation,
types de MLT, en inventant entre autres le quand nous nous rappelons un événement,
célèbre terme de mémoire «épisodique» où sa trace mémorielle devient flexible pour
seraient stockés les souvenirs des événements un temps limité et doit être consolidée à
personnellement vécus. Elle serait différente de chaque usage pour être remémorée par a
a mémoire sémantique qui, elle, est concentrée suite. En nous rappeant un événement, nous
sur des faits plus généraux. le réécrivons, et c’est de cette version réécrite
Puis, en 1974, Aan Baddeley a mis à jour le que nous nous souviendrons a prochaine fois.
concept de MCT. La mémoire de travail, selon Le cerveau stocke bien les souvenirs, mais les
lui, sait à a fois traiter et stocker l ’information. souvenirs changent avec le temps.

22 PSYCHOLOGIE COGNITIVE
TYPES DE MÉMOIRE

À VIE

JOURS

souvenirs anciens

HEURES

MINUTES

mémoire de travail

SECONDES

souvenirs passagers

MILLISECONDES

La mémoire est le stockage d’informations. goûtons, touchons, sentons, à tout moment.


Elle décrit un spectre continu, enregistrant des Un peu plus loin, a mémoire de travail, ou
changements sur des souvenirs de quelques mémoire à court terme, dont les souvenirs
millisecondes ou vieux de dix ans. À un bout sont vieux de quelques secondes à quelques
du spectre, des souvenirs passagers, d’une minutes. Et tout au bout du spectre, a
durée de vie très brève, stockent l’information mémoire à long terme, qui peut stocker de
sensorielle, ce que nous voyons, entendons, l’ information toute une vie.
Y --i oe
dns nos êes?
Des réunions en entreprise aux tribunes
des stades, nous pensons et agissons
différemment en groupe. Une pensée de
groupe se forme quand le désir de conformité
de chacun suppante a prise de décision
rationnelle, ce qui peut aboutir à des drames.

Dans son roman dystopique 1984, n’oserait jamais attaquer les États-Unis. Janis
George Orwell décrit a «double a formulé ses idées sur a pensée de groupe
pensée», le mécanisme par lequel dans un livre influent, paru en 1972.
les gens adoptent de force des croyances La pensée de groupe repose beaucoup
contraires à leurs propres souvenirs ou à leur sur a dissonance cognitive, le phénomène
sens des réalités. C’est ce qui a poussé le par lequel nous parvenons à adhérer à des
psychologue américain à concevoir et étudier croyances contraires au prix d’un stress
a pensée de groupe. psychologique. Leon Festinger a publié sa
Une pensée de groupe se forme quand théorie de a dissonance cognitive en 1957,
les opinions individuelles se noient dans le suite à l’étude d’une secte. Les membres
consensus. Les membres du groupe cessent de croyaient à l’imminence du déluge, mais
réfléchir, car il est plus facile de se taire et se quand a date prévue est passée, ils
fondre dans a masse que de donner de a voix ont dû corriger leurs croyances.
et de se distinguer. C’est ainsi que les groupes Les plus fanatiques ont estimé que
prennent parfois de mauvaises décisions leur dévotion avait évité le déluge,
fondées sur des hypothèses irréalistes. les moins endoctrinés ont plus facilement
Les conséquences peuvent être terribles. accepté de s’être trompés. Les deux groupes
Janis s’est servi des exemples de a baie des vivaient une dissonance et avaient adopté des
Cochons (l’invasion ratée de Cuba en 1961, stratégies différentes pour a résoudre.
ordonnée par le président Kennedy) et de Festinger nous pensait naturellement
l’attaque japonaise de Pearl Harbor en 1941. poussés à accorder nos croyances à nos
Dans les deux cas, les opinions discordantes comportements et ainsi éviter ou réduire a
ont été ignorées et les groupes se sont fondés dissonance. Pourtant, nos vies sont pleines de
sur des stéréotypes de leurs ennemis pour contradictions. Nous fumons sachant que cea
former leur stratégie. L’administration Kennedy cause le cancer, nous faisons des queues de
a supposé que les troupes de Castro seraient poisson sur a route alors que nous détestons
facilement défaites, tandis que Roosevelt ça, nous prenons l’avion pour un week-end
et ses conseillers estimaient que le Japon alors que cea aggrave le réchauffement.

24 PSYCHOLOGIE COGNITIVE
PENSÉE DE GROUPE

C’est parfois dur de se faire entendre au sein


d’un groupe. C’est ainsi que nous adoptons
parfois des opinions en désaccord avec nos
comportements, et qu’émerge une pensée
de groupe. Les conséquences sont parfois
graves, car nous pouvons alors prendre les
mauvaises décisions, brider notre créativité
ou ignorer les aspects moraux ou éthiques de
nos actes. La pensée de groupe peut naître
dans a vie de tous les jours, lors d’une dispute
en famille par exemple, mais ses effets sont
particulièrement graves dans les domaines
militaire, médical et politique.
Le rccorci es-i
 oie d dner?
Prendre un raccourci peut être risqué,
mais nous le faisons tout le temps pour
prendre des décisions, parfois avec bonheur.

Les humains sont intelligents. Nous Kahneman décrivait notre pensée logique,
prenons des décisions rationnelles, consciente, fondée sur a mesure précise
logiques, fondées sur une analyse des pour et des contre, comme une pensée
raisonnable des informations disponibles. lente, le «système2». Parfois, cette méthode
Non? Dans les années 1970 et 1980, les pas-à-pas est a meilleure. Mais quand il y a
psychologues Daniel Kahneman et Amos des incertitudes ou de a complexité, nous
Tversky ont développé une théorie à rebours adoptons une pensée rapide, le «système1».
des idées traditionnelles en économie. Selon C’est ce système qui peut subir l’influence de
cette théorie des perspectives, nous abritons biais dangereux, mais c’est aussi lui qui nous
des biais inconscients et une préférence pour permet de fonctionner par intuition.
les raccourcis qui nous mènent facilement à Les intuitions viennent de connaissances
prendre des décisions irrationnelles. implicites accumulées par expérience, qui
Un exemple de biais est notre grande deviennent automatiques, comme de savoir
aversion pour a perte. La douleur de perdre faire du vélo. Il nous est parfois impossible de
1000€, disons, ne peut être contrebaancée les expliquer (et elles sont parfois incomplètes
que par le paisir de gagner bien plus, autour et imparfaites), mais elles demeurent
du double de cette somme, selon certaines fonctionnelles. Quand un trader se demande
études. Parier plus après une perte de ce genre s’il doit acheter ou vendre une action, il lui est
est une manifestation de ce biais. difficile de prendre une décision prudente et
Nous prenons des raccourcis quand nous logique, mais ses intuitions, fondées sur son
ne disposons pas de toutes les informations expérience, lui fournissent un raccourci vers a
nécessaires, quand nous ne parvenons pas à bonne décision.
analyser l’information ou quand il faut agir vite, Prendre un raccourci peut ainsi se révéler
sous a pression. Un raccourci est alors notre dangereux, mais c’est aussi parfois le meilleur
seule option. Ce n’est pas toujours pour le pire. chemin à emprunter.

26 PSYCHOLOGIE COGNITIVE
HEURISTIQUE

N E B RA S KA

KANSAS

Les heuristiques sont ces raccourcis que


l’on prend pour réduire notre charge
cognitive et qui peuvent nous entraîner
vers des conclusions erronées. Par exemple,
l’ heuristique de disponibilité s’emploie pour
former un jugement quant à a fréquence
d’un événement. L’exemple le plus cité est
le suivant: quand on demande aux gens s’ il
y a plus de tornades au Kansas ou dans le
Nebraska, ils répondent le plus souvent «au
Kansas», car ils se rappellent qu’ il y a une
tornade au Kansas dans Le Magicien d’Oz.
CHAPITRE2 PSYCHOLOGIE
SOCIALE

ÉMOTIONS

CULTURE

IDENTITÉ

PRESSION

REPRÉSENTATION
CHAPITRE 2

SOCILE
PSYCHOLOGIE
CONFORMITÉ

CONTRÔLE

ENVIRONNEMENT
INTRODUCTION

S
eul, vous seriez capable de tout risquer pour venir
au secours d’autrui. En groupe, votre désir de vous
intégrer peut vaincre cette impulsion. Comprendre le
pouvoir des groupes, comment ils nous modèlent et
pourquoi, tel est l’objectif au cœur de a psychologie sociale.
Les groupes, dont notre propre groupe culturel,
agissent sur nos comportements, nos modes de pensée, nos
sentiments et même nos perceptions des sentiments d’autrui.
Il n’est pas certain que les émotions se lisent sur nos visages,
mais nos EXPRESSIONS FACIALES donnent tout de même
des indices. Ekman a distingué sept émotions de base, que
nous vivons et notons chez autrui: colère, dégoût, peur, joie,
mépris, tristesse et surprise.
Un groupe peut agir sur notre volonté de venir en
aide à quelqu’un. Quand nous nous en empêchons, les
psychologues parlent d’EFFET DU TÉMOIN, quand a présence
des autres décourage l’impulsion de l’intervention. Certaines
caractéristiques du groupe en question le court-circuitent. Par
exemple, nous aidons plus volontiers une femme en détresse
en présence de femmes et nous intervenons plus volontiers
quand nous connaissons tous les membres du groupe.
Nous pourrions dire qu’en groupe, nous nous comportons
comme des moutons. Dans les années1950, SOLOMON ASCH
a nommé ce phénomène le CONFORMISME. Ses études
avaient montré que les gens étaient plus enclins à fournir
des réponses délibérément fausses s’ils avaient entendu
quelqu’un donner aussi une réponse fausse avant eux.

30 PSYCHOLOGIE SOCIALE
Faire ce qui est facile plutôt que ce qui est juste revient
souvent à abdiquer nos responsabilités à des figures
d’autorité, un phénomène étudié durant les EXPÉRIENCES
DE MILGRAM. Milgram avait constaté que donner des ordres
quand on portait une blouse banche et agissait avec autorité
poussait les gens à augmenter dangereusement l’intensité des
décharges électriques qu’ils croyaient administrer à d’autres.
Il est encore plus fascinant de se demander dans quelles
conditions nous suivons le mouvement. Il semblerait que
nous ayons moins de chance d’obéir sans réfléchir à des
ordres déraisonnables quand celui qui les formule porte des
habits de tous les jours.
HENRI TAJFEL a découvert que les groupes formés
autour de frontières absurdes, comme a capacité de compter
des points, influençaient leurs membres de a même manière
que ceux formés autour de frontières plus cassiques, comme
l’âge ou le genre. C’est en partant de ce constat que Tajfel
et Turner ont inventé a THÉORIE DE L’IDENTITÉ SOCIALE
pour expliquer le conformisme spontané. Ils ont avancé
l’hypothèse que l’appartenance conférait une identité.
L’un des meilleurs moyens de combattre le biais de
favoritisme envers notre propre groupe est d’étendre notre
compassion en dehors de celui-ci. La compassion permettrait
aussi de dépasser notre tendance aux erreurs d’attribution,
qui consiste à attribuer le comportement désagréable de
quelqu’un à ses défauts intrinsèques plutôt qu’au contexte.

PSYCHOLOGIE SOCIALE 31
PSYCHOLOGIE SOCIALE

LIEU COMMUN SENS COMMUN


Quand un sujet est incorporé aux Concepts et termes largement
connaissances standards et très utilisés et compris qui nous aident
largement discuté, il devient à incorporer des idées complexes
un lieu commun. au quotidien.

CULTURES
REPRÉSENTATION COLLECTIVISTES
SOCIALE DE MOSCOVICI Sociétés où le bien-être du
Individus et sociétés groupe tout entier prend le pas
donnent du sens à des sur les besoins et les désirs
concepts complexes des individus.
en créant des versions
alternatives plus faciles
à saisir.

CHARLES DARWIN EXPRESSIONS FACIALES


Naturaliste, géologue et Darwin les pensait universelles
biologiste anglais (1809–82) et innées. Ekman a identifié sept
connu pour sa théorie de émotions de base: colère, dégoût,
l’évolution. Il pensait que joie, mépris, peur, surprise et tristesse.
les expressions faciales
étaient le résultat de notre
biologie commune.

BIAIS ENDOGROUPE/
EXOGROUPE
S’accorder avec ceux de notre
groupe/discriminer ceux en
THÉORIE DE
dehors – peu importe le critère
L’IDENTITÉ SOCIALE
de formation des groupes.
Nous adoptons vite les
habitudes des autres, car
appartenir à un groupe nous
confère une identité sociale, et
un sens à la vie (Tajfel
et Turner).
HENRI TAJFEL
Psychologue polonais (1919–1982)
qui a démontré la nature
arbitraire des frontières de groupe
dans ses expériences du «groupe

IDENTITÉ
minimal» dans les années 1960.

32
CULTURE
FACTEURS
ENVIRONNEMENTAUX
Influences sur notre comportement
ou nos actions issues de sources
externes, telles que le contexte précis
où nous nous trouvons.

EDWARD JONES
Psychologue (1926–1993) qui, en 1967,
a découvert que nous expliquions
le comportement des autres par
CULTURES leurs traits de caractère, la nature
INDIVIDUALISTES fondamentale des individus.
Sociétés où les besoins et les
désirs individuels ont la priorité
sur ceux du groupe pris comme
ERREUR

INFLUENCES SOCIALES
un tout; sujettes au biais de
l’erreur fondamentale FONDAMENTALE
d’attribution. D’ATTRIBUTION
Exagération de l’influence
des traits individuels,
sous-estimation parallèle
de l’influence du
contexte, des facteurs
environnementaux.

CONFORMITÉ DE
GROUPE
Suivre le troupeau et adopter
l’avis de la majorité; dépendant
de facteurs culturels, de l’identité
de groupe et de l’atmosphère
politique du moment.

EFFET DU TÉMOIN
La présence de
EXPÉRIENCES PRESSION SOCIALE
tiers nous dissuade
DE MILGRAM Capacité d’un groupe à
d’intervenir.
Expériences controversées peser sur les jugements
Les caractéristiques
durant lesquelles les individuels, menant à
des tiers, telles que le
sujets abdiquaient leur la conformité.
genre ou la proximité,
responsabilité à une figure
font varier les effets.
d’autorité en infligeant ce
qu’ils croyaient être des
décharges électriques
d’intensité croissante à
d’autres personnes. SOLOMON ASCH DÉFENSEUR
Psychologue (1907–1996) Quelqu’un qui ne se
qui, lors d’expériences contente pas d’être
menées dans les années spectateur, qui ose
1950, a examiné comment défendre un individu
et pourquoi nous nous ou une cause.
conformons au groupe.
ichons-nos os
es mêmes émoions?
On pourrait le croire, puisque nous
partageons tous a même biologie. Mais
il se trouve que c’est plus compliqué que ça.
Nos émotions et nos façons de les exprimer
sont aussi souvent le produit de notre culture.

Les émotions sont des outils chez un Occidental pourrait être interprété
indispensables à a survie. Elles sont par un Papou de Nouvelle-Guinée comme de
un moyen pour le corps de nous a peur. Certains états émotionnels semblent
prévenir d’un changement dans notre spécifiques à une culture. Ainsi de l’arigata-
environnement qui mérite notre attention. meiwaku que ressentent les Japonais quand
L’idée que tout le monde vive les mêmes on leur rend un service non désiré, mais pour
émotions remonte à Charles Darwin. En 1872, lequel ils sont néanmoins tenus de manifester
il a avancé que les expressions faciales des de a gratitude. C’est une émotion qu’on
émotions étaient universelles et innées, en ressent tous parfois, mais eux ont un mot pour
raison de notre biologie commune. Depuis, l’exprimer, car au Japon il est important de
quelques preuves scientifiques sont allées respecter les membres de votre groupe social,
dans ce sens. Le psychologue Paul Ekman même quand c’est énervant.
affirme avoir identifié sept émotions de base, Les émotions pourraient même être
colère, dégoût, joie, mépris, peur, surprise propres à un individu. Ce qui est de a terreur
et tristesse, que tout le monde ressent et pour quelqu’un est parfois de l’excitation
reconnaît chez les autres, quels que soient pour un autre. Et il existe plusieurs manières
l’ethnie, a angue, a culture ou le pays. d’exprimer a colère, a tristesse, a joie ou
Cependant, des recherches plus récentes a surprise, a stupéfaction et a satisfaction.
nuancent ce point de vue. Selon elles, les Cea rend a communication un peu plus
expressions faciales seraient plus compliquées compliquée que si les émotions étaient
que ce que Darwin pensait et nos réponses universelles. Ainsi, le meilleur moyen d’être sûr
émotionnelles aux situations dépendraient que quelqu’un ressent effectivement ce que
en partie de notre culture. Un air de dégoût son visage exprime, cea reste de lui demander.

34 PSYCHOLOGIE SOCIALE
EXPRESSIONS FACIALES

1. JOIE

2. COLÈRE

VISAGES AU REPOS 3. PEUR

Nous manifestons nos émotions sans même sourcils froncés ou rassemblés dénotent a
y penser. Des expressions faciales distinctes colère (2); tandis que le visage de a peur est
servent de signes aux plus communes. Par lié à a hausse des paupières et des sourcils
exemple, hausser les joues, plisser le coin et aux lèvres tirées vers l’arrière (3). En
des yeux et sourire suggèrent a joie (1). Les essayant de mimer ces expressions, il arrive
lèvres serrées, les yeux écarquillés et les même qu’on en vienne à les ressentir...
Ineriendrie-os
en cs de merre?
Nous aimerions tous penser que oui.
Mais nos actions dépendent de plusieurs
facteurs, parmi lesquels le nombre de gens
qui nous entourent et leur identité.

Cea paraît contre-intuitif, mais il La passivité des spectateurs est plus


semblerait qu’être un bon prononcée dans certaines situations, et les
Samaritain est plus difficile quand il facteurs les plus influents sont ici le genre
y a de nombreux spectateurs. Ce phénomène et a familiarité. Par exemple, un individu a
est connu sous le nom d’effet témoin et il plus de chance d’intervenir si a victime et
pose problème aux psychologues depuis des l’ensemble des témoins sont des femmes.
décennies. L’effet témoin a été mis en lumière Un homme a plus de chance d’intervenir
par le meurtre d’une femme de 28ans, Kitty si c’est le seul homme du groupe. Et une
Genovese, en plein New York, en 1964. Le New intervention est en général bien plus
York Times avait indiqué que 38 personnes probable si tous les spectateurs
avaient été spectatrices de l’attaque, sans rien se connaissent et appartiennent au
faire pour l’empêcher. C’était une exagération, même groupe soudé.
mais, depuis, bien des études expérimentales Que pouvons-nous faire pour aider
ont déterminé que l’effet était bien réel, avec davantage? Pour devenir des défenseurs
d’importantes nuances. plutôt que des spectateurs? Nous
Pourquoi a présence des autres nous pourrions essayer de voir a victime
dissuaderait-elle d’intervenir? Plusieurs comme un membre de notre propre
raisons sont envisageables. Nous pourrions groupe social, ou nous concentrer
supposer ou espérer que quelqu’un d’autre sur ce que nous avons en commun
prenne cette responsabilité, ou craindre de avec elle. La nature humaine étant
paraître incompétent devant cette foule. ce qu’elle est, nous sommes
À moins que nous fassions ce que nous faisons davantage disposés à courir
souvent dans des situations nouvelles: attendre un risque et à agir quand
de voir ce que font les autres. Si personne ne nous estimons que a victime
réagit, pourquoi le ferions-nous? potentielle est «des nôtres».

36 PSYCHOLOGIE SOCIALE
SPECTATEURS ET DÉFENSEURS
Avoir des amis donne souvent confiance, revanche, les héros se révèlent. Être entourés
même dans des situations dangereuses. Peu de personnes que nous connaissons, en qui
d’entre nous sont disposés à intervenir pour nous avons confiance, augmente notre sens
aider quelqu’un quand nous sommes au des responsabilités et nous rend plus disposés
sein d’une foule d’ étrangers. Entre amis, en à aider un inconnu en cas de détresse.
Les hmins son-is
os des moons?
C’est un peu dur pour les moutons, qui
sont plus intelligents qu’on ne le dit. Il est
vrai que nous suivons parfois le troupeau
sans réfléchir, mais nous avons souvent de
bonnes raisons pour ça.

L’idée commune selon aquelle les conformant à a mauvaise opinion de


gens agissent et pensent comme a majorité.
leur entourage vient d’une Les expériences d’Asch ont été répétées
expérience du psychologue Solomon Asch de nombreuses fois sous différentes formes.
dans les années 1950. Asch étudiait l’effet de Elles indiquent que le conformisme est un
a pression sociale sur nos jugements. Il a phénomène réel, mais dépendant de facteurs
invité des volontaires et les a intégrés à des tels que a culture, l’identité du groupe et
groupes de sept autres participants, qui tous, même l’atmosphère politique du moment. Par
à l’insu des premiers, étaient ses collègues. exemple, les citoyens de cultures collectivistes,
Celui-ci a ensuite présenté au groupe deux en Chine et au Japon, se conforment plus que
cartes. La première présentait une seule ligne ceux issus de cultures individualistes, tels qu’en
verticale, a deuxième trois, dont une Europe ou aux États-Unis. Les volontaires qui
identique à celle de a première carte. Il a s’identifient beaucoup au reste du groupe (sur
ensuite demandé à tout le monde de désigner le pan de l’âge ou de l’ethnie, par exemple) ont
cette ligne identique sur a deuxième carte. aussi plus tendance à s’aligner sur les autres.
Cea aurait dû être facile. S’ils avaient été Le niveau de conformisme paraît moins élevé
seuls, les volontaires auraient certainement aujourd’hui que dans les années1950, du moins
répondu juste à chacune des questions. Mais, aux États-Unis, peut-être parce que nos sociétés
douze fois sur dix-huit, Asch et ses collègues modernes sont moins conservatrices et les
donnaient de leur côté une réponse fausse, étudiants plus critiques.
ils désignaient délibérément une ligne D’une certaine manière, il aurait été
plus courte ou plus longue que a ligne de surprenant que nous ne nous conformions pas
référence. Lors ces douze tests-là, 76% des sous a pression sociale. Nous sommes des êtres
volontaires ont mal répondu au moins une sociaux, être seul nous paraît toujours risqué.
fois, aissant de côté leur avis propre et se Cea nécessite du courage.

38 PSYCHOLOGIE SOCIALE
EXPÉRIENCE DE PRESSION SOCIALE

La question qu’Asch posait aux volontaires


dans son expérience sur a pression sociale
était simple: aquelle des lignes A, B ou C est
identique à a ligne de référence? La réponse
estC. Mais quand les collègues d’Asch, intégrés
au groupe, répondaient délibérément à côté,
trois quarts des volontaires les suivaient et
se aissaient persuader plutôt que d’avoir
confiance en leur propre opinion. Dans ce
genre de situation, il est presque impossible
de prévoir comment chacun de nous réagira.
Qnd cessons-nos
de sire es ordres?
Cea dépend de qui les donne. Notre
tendance à obéir peut nous entraîner à
commettre des actes que nous réprouvons,
mais le contexte et les personnalités
impliquées ont beaucoup d’influence.

Vous avez probablement déjà grand étonnement de Milgram, deux tiers


entendu parler de l’expérience de d’entre eux ont obéi, jusqu’au maximum
Stanley Milgram, l’une des plus de 450volts, bien après que l’apprenant
connues en psychologie. Comme beaucoup de avait mimé une perte de connaissance. On
ses collègues dans les années1950 et1960, prend souvent ce résultat comme a preuve
Milgram était très marqué par les atrocités de a que les gens abdiquent automatiquement
guerre. Il a conçu son expérience pour savoir toute responsabilité de leurs actes devant
comment des gens normaux avaient pu en une figure d’autorité, même quand ils
commettre. Elle n’a pas vraiment répondu à a désapprouvent l’objectif. Mais l’expérience ne
question, mais elle nous a beaucoup appris. va pas jusque-là. Milgram en a organisé plus
Voici comment. Milgram disait à ses de trente autres sur le même thème, qui ont
volontaires que l’expérience visait à mesurer produit des résultats plus nuancés.
l’effet de a punition sur l’apprentissage. Par exemple, les volontaires obéissaient
Les volontaires étaient les «enseignants»: bien moins quand l’expérimentateur n’était
leur travail était d’envoyer une décharge pas dans a pièce, qu’il donnait ses ordres à
électrique à l’«apprenant» quand il se distance, ou quand l’expérience se dérouait
trompait et donnait le mauvais mot dans une dans une ville industrielle plutôt que dans les
liste de mots appariés. En fait, l’apprenant locaux prestigieux d’une université d’élite, ou
était un collègue de Milgram et ne recevait quand l’expérimentateur négligeait sa blouse
aucune décharge, mais ses cris de douleur banche au profit d’une tenue plus ordinaire,
persuadaient les enseignants du contraire. ou quand le volontaire était accompagné
L’expérimentateur, qui portait une blouse d’un autre, qui refusait d’aller plus loin. Cea
banche et un air d’autorité, encourageait semble indiquer que notre tendance à suivre
les volontaires à augmenter le voltage les ordres dépend en réalité fortement des
après chaque mauvaise réponse. Au circonstances et du contexte.

40 PSYCHOLOGIE SOCIALE
EXPÉRIENCE DU CHOC DE MILGRAM

L’ instalation de Milgram était très simple. une décharge électrique. Le voltage était
Le volontaire («l’enseignant») lisait une liste supposé augmenter avec chaque erreur,
de paires de mots à «l’apprenant», par et l’expérimentateur était présent pour
exemple boîte bleue, belle journée, canard encourager les volontaires ou leur ordonner
sauvage. À chaque fois que l’apprenant faisait de poursuivre en cas d’ hésitation. Les voltages
une erreur en les restituant, l’enseignant alaient de 15 à 450 volts, par incréments de
actionnait un interrupteur, censé déclencher 15volts. Jusqu’où seriez-vous allé?
Es-ce oors «ex»
conre «nos»?
À un certain niveau, oui. Vivre en
communauté est bien enraciné et favoriser
a sienne paraît inévitable. Cea ne rend pas
a guerre inévitable pour autant.

L’être humain a évolué pour vivre en l’identité sociale afin d’expliquer cette
groupe, au point qu’il est presque tendance naturelle à se regrouper. Ils estiment
impossible de réfréner cet instinct. que faire partie d’un groupe nous aide à définir
Nous nous paçons naturellement avec ceux ce que nous sommes, et que cette identité
qui nous ressemblent, selon a casse sociale, sociale donne du sens à nos vies. De plus, les
l’ethnie, a nationalité, a religion, l’âge ou le identités sociales pèsent beaucoup sur nos
genre, et nous favorisons les membres de comportements et nos opinions. Elles sont
notre groupe plutôt que les autres. aussi transversales: nous pouvons appartenir à
Il suffit de peu pour créer un sentiment de nombreux groupes différents, dans lesquels
de «eux» contre «nous». Nous le faisons nous nous comportons différemment.
au moindre prétexte, tel que a longueur Appartenir nous fait du bien. Le mauvais
des cheveux ou a couleur des yeux. Le côté, c’est que nos cœurs s’endurcissent envers
psychologue Henri Tajfel a démontré a nature ceux qui n’appartiennent pas. Ce préjugé a
arbitraire des frontières entre groupes dans joué un rôle dans presque tous les conflits.
son expérience du «groupe minimal» à Toutefois, il n’est pas inévitable: le biais de
l’université de Bristol dans les années1960. Il a l’endogroupe peut exister sans hostilité envers
découvert que nous étions prêts à discriminer l’exogroupe. Il est possible d’être loyal à son
les autres pour des raisons aussi triviales que groupe tout en restant amical envers les
leur sous-estimation ou leur surestimation autres. On peut être fan du PSG et apprécier
du nombre de points sur un écran. Le «biais quand même des supporters de l’OM. Il faut
de l’exogroupe» est terriblement facile à pour cea simplement être tolérant, accepter
déclencher, a-t-il conclu. que tout le monde ne fonctionne pas de
Tajfel et son collègue John Turner ont a même façon, ce qui est peut-être le défi
poursuivi et développé une théorie de principal de notre temps.

42 PSYCHOLOGIE SOCIALE
EXPÉRIENCE DU « GROUPE MINIMAL »
Tajfel a testé ses idées sur les préjugés sociaux de répartir une certaine somme d’argent entre
en divisant des dizaines d’ élèves d’une école tous ses camarades. À sa grande surprise, a
en deux groupes dont il a déterminé a majorité des élèves ont donné plus d’argent
composition sur des critères délibérément aux membres de ce qui n’ était pourtant
faibles, tels que a tendance à sous-estimer qu’un groupe totalement arbitraire: les sous-
ou surestimer le nombre de points présentés estimateurs accordaient plus de faveurs à
sur un écran. Il a ensuite demandé à chacun leurs camarades sous-estimateurs.

Groupe 1: Sous-estimateurs Groupe 2: Surestimateurs

Biais de l’exogroupe
Commen es idées
s’imposen-ees?
Les idées scientifiques passent par un
filtre social qui les rend plus acceptables et
faciles à partager, ce qui donne le sentiment
d’être expert sur des sujets complexes, tels
que les multivers ou a génétique.

Il y a toujours eu une tension entre sociétés de discuter et de prendre des décisions


les idées scientifiques et a manière collectives à propos de ces idées qui autrement
dont le grand public les comprend. se trouveraient hors de notre portée.
Aux béotiens, les théories scientifiques Moscovici a développé ses théories dans
semblent impénétrables et mystérieuses. le Paris des années1960 en se concentrant sur
Néanmoins, nombre d’entre elles se trouvent a psychanalyse, car il s’agissait d’une discipline
incorporées aux connaissances quotidiennes et controversée et néanmoins popuaire chez les
aux débats publics. Nul besoin d’être diplômé non-professionnels dans a France de l’époque.
en biologie pour comprendre le principe du Afin de discuter ses méthodes et ses principes, il
gène égoïste, par exemple, et pour pontifier à était nécessaire de les traduire dans une angue
ce sujet avec ses amis. compréhensible. Moscovici a remarqué que
Le psychologue français Serge Moscovici les gens paraient de psychanalyse comme s’il
a passé une bonne partie de sa carrière s’agissait de leur expérience quotidienne, par
à observer comment se déroule cette exemple en comparant une consultation de
assimiation des pensées complexes dans le psychanalyse avec une confession religieuse.
discours ordinaire. Il pensait que les individus Les représentations sociales des idées
et les sociétés s’approprient les concepts scientifiques sont très importantes de nos jours,
les plus abscons en en créant des versions alors que nous devons affronter des problèmes
alternatives plus faciles à assimiler. Il appeait littéralement existentiels tels que le changement
ces versions des représentations sociales. Les climatique et a perte de a biodiversité. Ces
représentations sociales «jettent un pont problèmes étaient autrefois cantonnés aux pages
entre l’étrange et le familier », comme le dit des journaux universitaires. La création d’un
Moscovici. Elles emploient a angue de tous les angage ordinaire pour les évoquer nous a permis
jours plutôt que le jargon incompréhensible de les incorporer à nos modes de pensée, et de
des universitaires. Surtout, elles permettent aux militer pour des solutions à l’échelle mondiale.

44 PSYCHOLOGIE SOCIALE
REPRÉSENTATION SOCIALE

Découverte scientifique

Théorie scientifique

Débat public

La théorie des représentations sociales de appréhender. Les savants se concentreront


Moscovici explique comment les découvertes sur les mécanismes et a théorie, tandis que
et les théories scientifiques sont traduites les débats publics tourneront autour des
en concepts que le grand public pourra impacts potentiels sur notre quotidien.
Porqoi s- i ç?
Quand quelqu’un fait quelque chose
de mal, nous avons tendance à en tirer une
conclusion générale sur sa personnalité.
«Ce doit être une mauvaise personne.»
Nous avons souvent tort.

Imaginez que vous appelez votre L’erreur fondamentale d’attribution s’est


banque pour parler de votre imposée dans le champ de a psychologie
découvert, et que l’employé au bout sociale après une expérience menée par
du fil se montre horriblement condescendant Edward Jones en 1967. Jones a découvert
et désinvolte. Votre réaction sera sûrement de que les gens expliquaient le comportement
penser «Quel sale type!» Autrement dit, vous d’autrui en ayant recours à des aspects
supposez que son comportement est le reflet supposés fondamentaux de a personne en
de sa personnalité profonde. question, même quand il était parfaitement
Ce serait une réaction normale. Pourtant, cair que son comportement découait
il y a de grandes chances que vous ayez d’une situation qu’elle ne maîtrisait pas. Des
commis ce que les psychologues appellent expériences ultérieures ont montré que
l’erreur fondamentale d’attribution. Il l’erreur fondamentale d’attribution était
s’agit de a tendance à exagérer le poids un produit de a culture et que les gens
des traits de personnalité individuels sur appartenant à des cultures individualistes y
les comportements, et de sous-estimer étaient davantage sujets que ceux issus de
l’influence du contexte. Les traits de cultures collectivistes.
personnalité et le tempérament ont une Une explication possible de ce biais est
influence certaine sur nos comportements, que nous augmentons ainsi le sentiment de
mais les facteurs environnementaux, le contrôler nos vies. Si chacun est entièrement
contexte particulier dans lequel ils agissent, responsable de ses actes, il devient plus facile
en ont souvent bien plus. L’employé de a d’attribuer les torts et de croire en un monde
banque qui vous a mal parlé au téléphone juste. L’idée que nos comportements seraient
n’était probablement pas un sociopathe. imprévisibles, les produits de circonstances
Le plus probable, c’est qu’il se trouvait dans toujours changeantes, est dérangeante. Mais si
une situation difficile à gérer, par exemple a nous l’admettions, nous pourrions tous devenir
pression d’une dure journée de travail. un petit peu plus charitables.

46 PSYCHOLOGIE SOCIALE
ERREUR FONDAMENTALE D’ATTRIBUTION

L’erreur fondamentale d’attribution a


été mise en lumière dans une expérience
cassique de Jones et Harris. Ils ont demandé
à des volontaires de lire des articles
soutenant ou critiquant Fidel Castro, puis
d’ évaluer l’attitude de leurs auteurs envers
le dirigeant cubain. Quand les volontaires
étaient prévenus qu’un auteur avait été
aissé libre de choisir sa position, ils le
cassaient en général parmi les soutiens de
Castro quand son article lui était favorable,
comme on peut s’y attendre. Mais quand on
les informait que l’auteur s’ était vu imposer
une position, ils estimaient toujours que
l’article reflétait son opinion véritable. Ils
éliminaient d’eux-mêmes une information
essentielle: que l’auteur n’avait en réalité
pas eu le choix.
CHAPITRE 3 APPRENTISSAGE

CONDITIONNEMENT

ASSIMILATION

CONSOLIDATION
DE LA MÉMOIRE

COMPORTEMENT
CHAPITRE 3

PPRETISSGE
ACCOMMODATION

TRAITEMENT

SCHÉMA
INTRODUCTION

S
oulever des poids fortifie les muscles, mais c’est
grâce à l’apprentissage de nouveaux talents que
les neurones s’épanouissent et se renforcent. De
l’enfance à l’âge adulte, ce que nous apprenons,
et comment nous l’apprenons, est au cœur de nos
comportements. Ce chapitre plonge dans les découvertes
les plus stupéfiantes de a psychologie de l’apprentissage.
Le béhaviorisme, inventé par Watson au début du
XXe siècle, a compris a domination de l’acquis sur l’inné
s’agissant de l’apprentissage. Établir des reations était alors
vu comme central dans a formation des habitudes qui nous
poussent ou nous satisfont, telles que faire de l’exercice ou
fumer. IVAN PAVLOV a découvert qu’il pouvait apprendre
à un chien à saliver au son d’une clochette retentissant
toujours avant le déjeuner. Le chien avait appris que ce son
signifiait l’arrivée prochaine du repas et il se mettait à saliver
rien qu’en l’entendant. Cette forme d’apprentissage appelé
le CONDITIONNEMENT CLASSIQUE survient quand nous
établissons une reation entre deux stimuli distincts.
La récompense et a punition forment une autre
façon d’apprendre, que B.F. SKINNER a baptisée du nom
de CONDITIONNEMENT OPÉRANT. Les comportements
récompensés ont plus de chance d’être reproduits. Quand
votre conjoint sort les poubelles et que vous le récompensez
d’un mot gentil ou en lui cuisinant un repas délicieux,
vous l’aidez à apprendre que sortir les ordures déclenche
de bonnes choses, augmentant ainsi les chances qu’il
recommence. La théorie affirme aussi que nous pouvons
désapprendre un comportement en cessant de le lier à une
récompense ou en l’associant à une punition.

50 APPRENTISSAGE
Les comportements des enfants dérivent beaucoup de
leur conditionnement, et leur communication de l’imitation.
La THÉORIE DE L’APPRENTISSAGE SOCIAL DE BANDURA
se propose de tenir compte des mécanismes nécessaires à
l’imitation efficace d’un comportement, tels que l’attention,
a mémoire et a motivation. Mais c’est a THÉORIE DES
PALIERS D’ACQUISITION DE PIAGET qui est parvenue à
décrire comment les enfants comprennent petit à petit
le monde qui les entoure. Les tout petits sont incapables
de comprendre ce que pensent les autres, mais ils ont
une capacité d’apprentissage du angage extraordinaire.
Chomsky a supposé que c’était parce que leur cerveau avait
accès à des mécanismes d’acquisition tout prêts qui forment
ensemble un DISPOSITIF D’APPRENTISSAGE, qui semble se
bloquer autour de 18ans, rendant l’apprentissage bien plus
difficile par a suite.
La RÈGLE DES 10000HEURES DE GLADWELL accorde
à a pratique une pace plus importante qu’au talent sur
le chemin de a haute performance, mais elle ne tient pas
compte de a qualité de cette pratique, de a chance et du
talent inné, qui contribuent au succès.
La psychologie offre de nombreux outils pour améliorer
l’efficacité des apprentissages, des astuces pour renforcer
notre attention, nos capacités de conceptualisation et notre
mémoire. Des mécanismes plus profonds procurent une
mémorisation plus durable, une découverte attribuée à
Craik et Tulving. Un DISPOSITIF MNÉMONIQUE, qui consiste
à associer un fait, une image ou une histoire à ce dont
nous voulons nous rappeler ou à lui créer une anagramme,
améliore a durée de mémorisation de presque 50 %.

APPRENTISSAGE 51
APPRENTISSAGE
CONDITIONNEMENT
IVAN PAVLOV RÉPONSES CONDITIONNÉES
Physiologiste russe (1849– Les émotions sont apprises en
1936) connu pour ses travaux réponse à des stimuli auparavant
sur le conditionnement neutres. Dans une étude, Watson
classique et son expérience et Rayner montraient des rats à
du chien qui salive (1897) des bébés en même temps qu’un
en particulier. bruit retentissait pour les effrayer.

CONDITIONNEMENT
CLASSIQUE
Les béhavioristes comme
Watson et Pavlov disaient
que nous apprenons nos
CONDITIONNEMENT comportements par un
OPÉRANT mécanisme d’association, où
L’apprentissage se fait par un stimulus produit une
récompense et renforcement réponse précise.
ou punition et dissuasion, le
comportement est contrôlé par
ses conséquences (Skinner).

B.F. SKINNER
RENFORCEMENT Béhavioriste américain
PROGRAMMÉ (1904–1990) qui a développé les
Fourniture des récompenses théories du conditionnement.
selon un ensemble de règles, Célèbre pour son travail sur le
qui affectent les taux de conditionnement opérant et sa
réponse et de disparition des célèbre expérience des «boîtes
comportements concernés. de Skinner» avec des rats.

ASSIMILATION ACCOMMODATION
Une des deux façons (voir Une des deux façons (voir
aussi accommodation) dont aussi assimilation) dont
les informations nouvelles l’information s’intègre
s’intègrent aux schémas aux schémas; quand les
existants; quand l’information schémas sont mis à jour
est modifiée pour s’adapter à un après une information
schéma existant (Piaget). nouvelle (Piaget).
RÉPÉTITION
RÈGLE DES 10000 HEURES MNÉMONIQUE
DE GLADWELL Systèmes mentaux ou appareils
Une pratique répétée pendant au d’aide à la mémoire, comme
moins dix ans, plutôt que le talent, d’associer un fait, une image ou
renforce les connexions neuronales une histoire à quelque chose
et rend le rappel des informations ou dont on veut se rappeler,
des compétences répétées plus facile. ou créer une anagramme.

RÉPÉTITION ÉLABORATIVE
Technique de mémorisation et
d’apprentissage qui implique
l’analyse de l’information durant le
THÉORIE DU
traitement de fond en procédant
TRAITEMENT DE
par association, organisation,
L’INFORMATION
catégorisation ou par des stratégies
DE MILLER
mnémotechniques.
L’esprit humain procède en
quatre étapes: 1)assister à
l’information, 2)la coder, 3)la HYPOTHÈSE DE LA
stocker en mémoire, 4)y PÉRIODE CRITIQUE
accéder plus tard. Idée selon laquelle nous
serions sensibles aux
stimuli linguistiques durant
une certaine période;
au-delà, l’apprentissage
serait plus difficile.

THÉORIE DE
L’APPRENTISSAGE THÉORIE DU
SOCIAL DE BANDURA DÉVELOPPEMENT
Nous observons puis imitons COGNITIF DE PIAGET
les comportements; les L’intelligence des enfants change
conditions, par exemple les avec l’âge, en quatre étapes clés.
récompenses, nous orientent. Ils acquièrent de nouveaux
schémas et étendent les anciens
au fil de leur développement.
MÉCANISMES
DE MÉDIATION
Observation du comportement,
capacité de le reproduire, désir
de le faire. DISPOSITIF D’ACQUISITION
DU LANGAGE (DAL)
DE CHOMSKY
Principes linguistiques innés
pour construire et comprendre le
langage quand nous l’entendons.

DÉVELOPPEMENT
COGNITIF
Sie-os qnd
sonne ’here d dîner?
Si oui, c’est peut-être le résultat d’un
conditionnement cassique. La célèbre étude
de Pavlov, dans aquelle un chien salive
quand sonne l’heure du repas, nous a légué
une idée centrale sur le comportement et
l’apprentissage humains.

Pourquoi nous comportons-nous conditionnement cassique: un mécanisme


comme ça? Les béhavioristes d’apprentissage qui se déclenche quand deux
comme John B. Watson disent que stimuli sont associés à répétition, jusqu’à ce
nous apprenons nos comportements selon un que a réaction naturelle au second stimulus
mécanisme d’association qu’il appeait le soit déclenchée par le premier stimulus seul.
conditionnement cassique. Watson a étendu ces découvertes en
Les idées de Watson se fondaient surtout se paçant résolument du côté de l ’acquis
sur les observations d’Ivan Pavlov. D’abord contre l’inné. Il pensait que l’apprentissage
physiologiste, Pavlov étudiait a salivation par le conditionnement était l ’origine de
chez les chiens quand il fit une découverte nos comportements, et même de nos
accidentelle. Il remarqua que les chiens sentiments. L’étude de Watson et Rayner
salivaient souvent avant qu’arrive le repas. Par (1920) a été pionnière dans l’application
exemple, quand ils entendaient les pas de du conditionnement aux comportements
l’assistant qui s’approchait. humains (mais elle n’était pas très éthique).
Poussé par ces observations, Pavlov a Ils présentaient des rats à un bébé, Albert,
publié en1897 une série d’études aujourd’hui en même temps qu’un bruit fort retentissait.
célèbres. Il avait pacé un bol de nourriture Le bruit faisait pleurer le bébé. À a fin du
devant des chiens et agité une cloche. Les conditionnement, Albert avait peur des
chiens salivent naturellement devant de rats. Ainsi, les émotions pouvaient être
a nourriture, mais a priori pas quand une conditionnées. La ligne béhavioriste dure de
cloche tinte. Toutefois, après un certain Watson et le paradigme du conditionnement
nombre d’associations entre a nourriture ont été remis en question et affinés depuis,
et a cloche, les chiens avaient appris, par mais ses découvertes théoriques et pratiques
association; ils salivaient quand elle tintait, sont encore appliquées en psychologie, pour
même sans nourriture devant eux. C’est le traiter les phobies ou favoriser l’éducation.

54 APPRENTISSAGE
EXPÉRIENCE DU CHIEN DE PAVLOV

Dans le conditionnement cassique de Pavlov, automatique avant l’association répétée


a nourriture est le stimulus inconditionné, avec le stimulus inconditionné (a nourriture).
car elle déclenche seule une réponse La cloche devient ensuite le stimulus
automatique chez le chien, a salivation. La conditionné, dès qu’elle provoque une
cloche est un stimulus neutre, sans réponse réponse – a salivation – à elle seule.
pprenons-nos
de nos récompenses?
Oui ! Dans le conditionnement opérant,
les récompenses renforcent, changent ou
motivent les comportements, qu’ils soient
sains (une promenade après un dur abeur)
ou pas (une pause goûter).

B.F. Skinner était un béhavioriste qui Ferster et Skinner (1957) ont introduit
a nuancé les idées pionnières sur le l’idée de renforcement programmé: un
conditionnement de Pavlov et ensemble de règles qui fixe l’attribution des
Watson (page54). À a différence de Watson, récompenses. Ils ont découvert qu’il modifiait
Skinner pensait que a culture comptait autant le taux de réponse (le taux de répétition
que a nature, que le comportement humain d’un comportement adopté en vue d’une
pouvait être inné, acquis, ou les deux. Il récompense) et le taux d’extinction (le taux
estimait que les comportements s’apprenaient auquel le comportement disparaît). Par
néanmoins, et que l’apprentissage s’expliquait exemple, quand a récompense est fournie
le mieux au moyen du paradigme du après un nombre aléatoire de «bonnes
conditionnement opérant: le renforcement réponses», le taux d’extinction est au plus bas.
par récompense et punition. Dans cette On le constate au casino, où les gens jouent
optique, ce sont les conséquences qui sur a même machine un nombre incalcuable
contrôlent le comportement. Quand nos de fois, sachant que a récompense pourrait
actions sont récompensées, il y a plus de tomber à tout moment. Le renforcement
chance qu’elles se répètent, et inversement. continu, où a récompense est fournie pour
Le renforcement augmente a fréquence du chaque bonne réponse, provoque un taux de
comportement, a punition a diminue. réponse lent et un taux d’extinction rapide.
Skinner séparait les renforcements Peut-être parce que a satisfaction vient vite,
positifs ou négatifs et a punition. Imaginez ôtant a nécessité de reproduire a réponse.
que votre patron vous fixe un objectif de tant Skinner a utilisé le conditionnement
de ventes par semaine. Il pourra faire usage opérant pour développer des méthodes de
de renforcement positif en promettant une modification du comportement. Ses travaux
prime en cas de succès, ou de renforcement sont encore appliqués de nos jours dans le
négatif en diminuant votre commission quand domaine de l’addiction, les salles de casse ou
l’objectif n’est pas atteint. les prisons.

56 APPRENTISSAGE
CONDITIONNEMENT
ET BOÎTE DE SKINNER
Skinner a étudié le conditionnement
opérant en paçant des rats dans une
«boîte de Skinner». Côté renforcement
positif, presser un levier libérait un granule
de nourriture. Après l’avoir pressé plusieurs
fois par accident, les rats finissaient par
apprendre et se dirigeaient droit vers le
levier quand ils arrivaient dans a boîte. Côté
renforcement négatif, Skinner soumettait
les rats à un courant électrique, qu’ ils
apprenaient à éteindre en pressant le levier.
pprendre es-i
n e d’imiion?
Albert Bandura le pense. Ses travaux se
sont concentrés sur l’apprentissage des
comportements par imitation. Selon sa
théorie de l’apprentissage social, les enfants
observent puis imitent. Des conditions telles
que les récompenses orientent leurs efforts.

Les travaux de Pavlov, Watson et que nous apprenons nos comportements


Skinner ont posé les bases du sociaux en observant et imitant. Il a étendu
béhaviorisme. Albert Bandura le béhaviorisme, y a incorporé l’idée que
partageait les principes de leur approche. les humains sont actifs lors du traitement
Mais il s’est éloigné de ses pairs en proposant de l’information et qu’ils pensent aux
que l’apprentissage recouvrait davantage que conséquences de leurs actes. Nos pensées
des associations de stimuli, des récompenses renforcent ou affaiblissent l’association
et des punitions. Selon lui, l’apprentissage entre ce qu’on observe et ce qu’on fait:
pouvait aussi passer par l’observation. Les c’est a médiation. Bandura a noté
enfants observent des modèles, des individus plusieurs mécanismes de médiation
adoptant un comportement social, et les qui déterminent si ce qu’on observe
reproduisent par imitation. Ils n’imitent pas changera ce qu’on fait: l’attention
n’importe quel comportement, cea dit. portée au comportement observé, a
L’imitation est sélective. capacité de l’imiter et l’envie de l’imiter.
Bandura a désigné un ensemble de Les applications des travaux de Bandera
facteurs rendant l’imitation plus certaine: sont nombreuses. Son expérience de a
percevoir le modèle comme simiaire à soi poupée Bobo (1961) nous a renseignés sur
(par exemple, son genre), le renforcement l’effet de a violence dans les médias sur les
simple (par exemple, des louanges) et ce qu’il enfants. Ses recherches ont aussi écairé les
appeait le renforcement par procuration rôles du genre, a survenue de changements
(quand le modèle reçoit une récompense sociaux et notre manière de donner du
pour son comportement). Les enfants sont sens à notre culture. La TAS a été critiquée,
aussi plus susceptibles d’imiter un modèle car elle ne tient pas compte du contrôle
s’ils le perçoivent comme doté de qualités cognitif exercé sur les comportements.
désirables. Bandura l’a donc révisée pour former une
La théorie de l’apprentissage social théorie cognitive sociale, en 1986, et lui
de Bandura (TAS), publiée en 1977, énonce intégrer l’idée d’agentivité.

58 APPRENTISSAGE
EXPÉRIENCE DE LA POUPÉE BOBO
DE BANDURA

La célèbre expérience de a poupée Bobo


consistait à pacer des enfants devant un
adulte interagissant avec une poupée.
Ils observaient ainsi une interaction douce ou
agressive, ou, dans le groupe de contrôle, pas
d’ interaction du tout. Puis les expérimentateurs
vérifiaient les interactions ultérieures entre
les enfants et a poupée. En moyenne, ceux
qui avaient observé une interaction agressive
étaient les plus nombreux à reproduire un
comportement agressif, ce qui démontrait
que les enfants peuvent apprendre des
comportements sociaux par l’observation.
Dix mie heres de
priqe eron-ees
de os n énie?
Pas tout à fait, mais presque. La règle
de Gadwell énonce que a clé de l’expertise
et de a performance, c’est le travail, une
pratique intense et méthodique pendant au
moins dix ans, ou dix mille heures.

La règle des dix mille heures de établir un chemin bien tracé entre deux
Malcolm Gadwell (2008) édicte que neurones demande du travail. Quand ils sont
le talent n’est jamais inné. Gadwell a activés en même temps, ils s’associent. Ainsi,
trouvé l’inspiration dans un article de Simon et une pratique répétée renforce leur connexion,
Chase de 1973 sur les joueurs d’échecs, dans et se souvenir des informations ou des gestes
lequel ils jugeaient que c’était grâce à leur répétés devient plus facile.
mémoire des schémas de partie constituée au Plus récemment, a règle de Gadwell a été
fil de leurs années de pratique que les joueurs sous le feu des critiques. Macnamara et Maitra
amélioraient leurs performances. Ils notaient (2019) ont renouvelé l’étude d’Ericsson sans
qu’aucun grand-maître n’avait dédié moins de parvenir à reproduire ses résultats. Les moins
dix ans au jeu avant de recevoir son titre. doués des violonistes avaient en effet moins
La règle des dix mille heures a été de dix mille heures de pratique, mais ce critère
formulée plus rigoureusement par Ericsson et ne fonctionnait plus pour séparer les bons et
ses collègues dans une étude de 1993 sur des les meilleurs (chaque groupe se situait autour
violonistes. Ils ont établi que les différences de 11000heures). Les heures de pratique
de compétence entre musiciens étaient n’expliquaient que le quart de l’écart de talent,
étroitement liées à leur niveau de pratique, contre 48% chez Ericsson.
et que les violonistes de premier pan avaient Gadwell a concédé que d’autres facteurs
intensément pratiqué leur instrument pendant agissaient (qualité de l’entraînement, privilège
dix ans au moins, 10000 heures en moyenne. de naissance, chance et talent inné). Des
Pourquoi? La réponse tient dans le études plus récentes montrent que les dix
mécanisme de consolidation de a mémoire. mille heures ne suffisent pas pour rejoindre
Le cerveau contient environ dix milliards l’élite. Néanmoins, il reste certain qu’une
de neurones. Comme dans une forêt dense, pratique assidue facilite l’accès aux sommets.

60 APPRENTISSAGE
RÈGLE DES 10 000 HEURES

L’ étude de Simon et Chase estimait que les


joueurs d’ échecs passaient entre 10000
et 50000heures à étudier pour devenir
grand-maître. Gadwell prend cet exemple
à dessein, car a règle des 10000heures ne
s’applique qu’à des activités demandeuses
sur le pan cognitif, telles que les échecs, qui
requièrent d’apprendre de longues listes
de possibilités afin d’atteindre a maîtrise,
à a différence des activités physiquement
demandeuses, comme a course.
Commen n enn
oi-i e monde?
Selon Piaget, les enfants voient le monde
sous le prisme de briques de connaissances qui
s’empilent pour construire une compréhension
globale de leur environnement. À mesure
qu’ils se développent, leurs briques évoluent.

Jean Piaget pensait que les bébés développant, nous construisons de nouveaux
naissent sans a conscience d’une schémas et nous améliorons les précédents.
réalité extérieure à eux-mêmes. En Piaget décrit deux façons d’organiser les
résumé, les enfants se comportent en nouvelles informations pour qu’elles intègrent
mini-scientifiques, ils font des expériences ou étendent un schéma existant. Ces
afin de construire leur représentation mécanismes aident les enfants à progresser
mentale du monde. Ces représentations dans les stades. Le premier mécanisme est
fournissent le cadre où se formera leur l’assimiation, dans lequel nous modelons
compréhension du monde qui les entoure. l’information pour qu’elle s’intègre à un
Selon a théorie du développement schéma, par exemple quand un enfant pointe
cognitif de Piaget, les enfants ne sont du doigt et crie «chien!» quand il voit un
pas moins intelligents que les adultes, loup à a télévision pour a première fois.
seulement, leur intelligence change en Le deuxième est l’accommodation,
grandissant. La théorie propose ainsi quatre quand de grands bonds de compréhension
étapes du développement cognitif: le stade sont effectués à l’occasion de a mise à jour
sensorimoteur (entre 0 et 2ans), le stade d’un schéma. Par exemple, quand le parent
préopérationnel (entre 2 et 7ans), le stade explique que l’animal est un loup, pas un
concret (de 7 à 11ans) et le stade formel (au- chien. L’enfant devient alors capable de
delà de 12ans) (page96). modifier les paramètres du schéma «chien»
Qu’est-ce qui change dans l ’intelligence en ajoutant que les chiens sont apprivoisés, et
d’un enfant qui se développe? La réponse à il ajoutera un schéma séparé pour les loups.
cette question est ce que Piaget appelle le Les stades de Piaget ont été critiqués dans
schéma. Ce sont des briques de construction leurs détails, mais ils ont néanmoins fourni un
cognitives : des unités de savoir qui ont trait cadre utile et influent pour a compréhension
à divers aspects du monde, tels que les du développement cognitif de l’enfant, dont de
objets, les actions ou les concepts. En nous nombreux travaux sont issus.

62 APPRENTISSAGE
ERREUR A-NON- B

A B A B

A B A B
Les expériences de Piaget ont démontré est caché enB. Malgré tout, le bébé le
un phénomène connu sous le nom d’erreur cherche encore enA. L’erreur illustre que
A-non-B, qui survient généralement au le schéma de permanence des objets est
stade sensorimoteur. Un enfant entre9 et alors incomplet: l’enfant n’a pas encore
12mois voit un objet, caché en A. L’enfant compris que les objets existent toujours
cherche enA. Après une répétition, l’objet même quand il ne les voit pas.
Qnd dire be be 
’pprenisse d’ne
noee ne?
Les partisans de l’hypothèse de a
période critique diront que c’est quelque
part entre 7 et 18ans. Quoi qu’il en soit, en
vieillissant, notre capacité d’apprentissage
d’une angue diminue significativement.

Que les très jeunes enfants puissent lequel nous sommes le plus sensibles aux
apprendre quelque chose d’aussi stimuli linguistiques. Au-delà, c’est peut-être
compliqué qu’une angue est trop tard pour parler comme un locuteur natif.
proprement incroyable. Pourtant, presque tous La porte du DAL pourrait être fermée.
les enfants de a Terre en apprennent une d’un De nombreuses hypothèses existent
genre ou d’un autre. Ils paraissent bien mieux pour dater a fin de cette période, a plupart
équipés que les adultes pour cette tâche, tournant autour du début de a puberté. Afin
comme s’ils étaient nés pour ça! de trancher, une étude de 2018 de Hartshorne,
Noam Chomsky estime que nous sommes, Tenenbaum et Pinker au MIT s’est servie de
en effet, nés pour apprendre une angue. En l’échantillon le plus grand jamais conçu pour
1969, il a formulé une théorie selon aquelle ce genre d’enquêtes: 670000 personnes. Ils
les humains sont munis d’un dispositif ont découvert que a période critique s’étend
d’acquisition du angage (DAL), composé de bien au-delà de ce que nous pensions, jusqu’à
principes linguistiques innés: un ensemble 17 ou 18ans. Toutefois, égaler le niveau d’un
de possibilités selon lesquelles une angue locuteur natif pourrait ne plus être possible à
peut fonctionner. Le DAL nous permet de moins d’avoir commencé avant 10ans.
comprendre a grammaire à partir de données Les adultes doivent-ils renoncer à
brutes tirées de l’environnement. Il nous suffit l’apprentissage des angues? Pas forcément.
d’entendre cette angue parlée autour de nous Les parler couramment sera peu probable,
pour l’apprendre parfaitement très tôt. mais les parler bien demeure possible. De plus,
Que se passe-t-il si nous ne sommes pas Singleton (1995) a noté que 5% des adultes
exposés à une angue avant un âge avancé, et bilingues le sont devenus à l’âge adulte.
que l’apport du DAL nous fait ainsi défaut? Cea nécessite de travailler dur, toutefois.
L’hypothèse de a période critique pose La facilité d’apprentissage dont bénéficient les
l’existence d’un intervalle de temps dans enfants s’enfuit avec le temps.

64 APPRENTISSAGE
ACQUISITION DU LANGAGE
CHEZ LE JEUNE ENFANT

90

l
r
r rr
l
r r l
l
l
l

80

Enfants américains
% CORRECT

70

60 Enfants japonais

50

6–8 mois 10–12 mois

ÂGE DES ENFANTS

Kuhl et son équipe ont démontré que font sans problème. Entre 6 et 8 mois, les
même a première année était critique pour enfants étaient tout aussi compétents les
former nos capacités d’apprentissage d’une uns que les autres pour distinguer les deux
angue. L’ étude comparait a capacité de sons. Mais, parvenus à 12 mois, les enfants
différencier le son «r» du son «l» chez américains y arrivent bien mieux que leurs
des enfants japonais et américains, car les camarades japonais. Ces données illustrent
adultes japonais ont du mal à différencier qu’ il existe une période critique pour a
les deux, tandis que les anglophones le discrimination de certains sons.
Pe-on méiorer
s mémoire?
En quelque sorte. Il n’y a pas de moyen
rapide pour mieux mémoriser l’information,
mais a psychologie nous indique comment
être bien plus efficace au prix d’un effort de
traitement mnémotechnique.

La capacité de notre esprit à stocker traitement profond, il falait déterminer si


de nouvelles informations est l’insertion du mot dans un espace aissé banc
illimitée. Comment se fait-il donc au sein d’une phrase donnée lui conférait le
que nous ayons tant de mal à nous rappeler sens adéquat. Plus tard, au cours d’un exercice
certaines choses ? Des études psychologiques de mémoire, ceux qui avaient effectué un
ont montré que nous pouvons améliorer a traitement superficiel se rappeaient 25% des
mémorisation en mettant plus d’intention mots, contre 75% pour les participants chargés
dans nos apprentissages. du traitement profond.
Pour comprendre comment améliorer En quoi consiste exactement ce
notre mémoire, il faut d’abord comprendre traitement profond? Il implique une
comment notre esprit traite l’information. répétition éaborée: une analyse plus aboutie
La théorie du traitement de l’information de l’information. Par exemple, lui associer
de George Miller découpe ce mécanisme en une image, l’incorporer dans une histoire, a
quatre étapes. L’apprenant entre en contact lier à d’autres faits ou a transformer en une
avec l’information, puis il encode, ou absorbe, anagramme. Tous ces outils sont des moyens
l’information s’il lui accorde de l’importance mnémotechniques: des opérations mentales
et de l’attention. Cette information est ensuite conçues pour aider a mémoire. Bower (1972)
stockée dans a mémoire, afin d’être rappelée a distribué à ses étudiants une liste de mots
plus tard si besoin. à mémoriser et découvert que ceux qui
Toutefois, tous les mécanismes ne sont pas employaient des moyens mnémotechniques
égaux. Craik et Lockhart (1972) différencient se rappeaient environ 44% de mots de plus
les traitements superficiels – l’encodage que les autres. Par ailleurs, l’entraînement
des propriétés physiques ou du son – et à ses moyens mnémotechniques compte:
les traitements profonds – l’encodage de a de nombreuses études ont conclu que les
signification. L’expérience de Craik et Tulving résultats s’amélioraient beaucoup quand on
(1975) illustre l’importance de ce traitement accordait un coup d’essai aux participants. Ils
profond en demandant aux participants pouvaient ainsi juger de a qualité des moyens
de réfléchir à des mots à différents niveaux mnémotechniques employés.
d’encodage. Pour un traitement superficiel, La prochaine fois que vous devez
on leur demandait si le mot était écrit mémoriser un long discours, rappelez-vous:
en majuscules ou en minuscules. Pour le c’est une question d’encodage.

66 APPRENTISSAGE
THÉORIE DU TRAITEMENT
DE L’INFORMATION

ÉTAPE 2
Encodage

ÉTAPE 3
Stockage

ÉTAPE 1
Observation

ÉTAPE 4
Rappel

La théorie du traitement de l’ information est stockée dans un lieu bien précis, par
de Miller s’ inspirait des ordinateurs. Pour le exemple votre dossier «Articles». Par a suite,
comprendre, imaginez que vous écrivez un quand il vous faudra le transmettre, vous
article sur votre ordinateur. Celui-ci constate accédez au fichier dans son dossier Articles,
les symboles que vous tapez. Il les encode où il était stocké, et vous l’envoyez. Comme
pour les afficher sur son écran. En cliquant les ordinateurs, nous traitons l’ information en
sur «Sauvegarder», l’ information encodée l’encodant, a sauvegardant et a rappeant.
CHAPITRE 4 PSYCHOLOGIE
BIOLOGIQUE

CERVEAU SENS

PERFORMANCE

HIPPOCAMPE

DROGUES

SYNESTHÉSIE
CHAPITRE 4

BIOLOGIQUE
SENTIMENTS PSYCHOLOGIE

PROPRIOCEPTION
INTRODUCTION

I
l est rare que des découvertes révolutionnent une
science, et pourtant a biopsychologie a connu plus
de révolutions que toutes les autres branches de a
psychologie. Ce chapitre en présente quelques-unes.
Les scientifiques nous ont révélé que nous disposons
de plus de 50 SENS. Un sens nous informe de a position
de nos membres dans l’espace, a proprioception, ce qui
nous permet d’amener une cuillère de gâteau jusqu’à
notre bouche les yeux fermés. L’équilibrioception tire parti
d’un liquide de notre oreille interne pour nous indiquer
où est le haut, et si nous ralentissons ou accélérons. Nous
nous reposons sur d’autres sens internes indispensables
pour vivre, par exemple pour savoir si nous avons faim ou
si notre vessie est pleine. Les sens interagissent, et cette
interaction poussée à son paroxysme est a SYNESTHÉSIE,
le méange permanent de certains sens.
Nos sens sont a clé de notre expérience du monde,
mais aussi de nos sentiments. Les émotions sont plus que
des sensations corporelles. Elles émergent de nos réponses
aux informations que nos sens nous transmettent, de nos
comportements, de nos pensées et de nos interprétations
du monde. Les psychologues savent que modifier un seul
de ces éléments modifiera nos sentiments, par exemple
quand notre niveau de stress augmente. La LOI DE YERKES
ET DODSON explique comment bien utiliser le stress. Elle
est liée au fait que nous sommes plus efficaces quand
a pression mentale ou physiologique augmente, jusqu’à

70 PSYCHOLOGIE BIOLOGIQUE
un certain point. Quand il y en a trop, le bien-être physique
et mental chute et les mauvaises habitudes s’installent.
C’est ainsi que nous finissons par nous en remettre au café.
La biopsychologie a écairé le fonctionnement de
médicaments tels que les antidépresseurs et de drogues
telles que a caféine ou a marijuana. Elles interagissent avec
des récepteurs et modifient les NEUROTRANSMETTEURS
du cerveau. Les antidépresseurs aident au recycage de
a sérotonine, l’un des 50 neurotransmetteurs découverts.
Quand nos cerveaux nous lâchent, par exemple
quand nous échouons à reconnaître un visage familier,
nous aimerions pouvoir blâmer l’abus d’une substance.
Cependant, ce n’est un problème que quand cea se répète
tout le temps, par exemple en cas de PROSOPAGNOSIE,
le trouble de a reconnaissance des visages qui survient en
raison de lésions au GYRUS FUSIFORME. En étudiant des
singes, les scientifiques ont localisé six zones cérébrales
responsables de a reconnaissance des visages. Certains
neurones détectent les distances entre les yeux, d’autres
répondent à a texture de a peau, etc.
L’une des découvertes les plus excitantes de a
biopsychologie est que nous pouvons renforcer a zone
de a mémoire dans notre cerveau, l’agrandir et améliorer
son fonctionnement. C’est ce qu’a découvert Maguire: les
chauffeurs de taxi londoniens musclent leur HIPPOCAMPE
en apprenant petit à petit le dédale des rues et des ruelles
de Londres et de ses environs.

PSYCHOLOGIE BIOLOGIQUE 71
PSYCHOLOGIE BIOLOGIQUE

AFFECT
Expérience d’une
sensation, d’une émotion ou SENS
d’une humeur. Contribue à une Nous avons plus de 50
variété de dispositions mentales, sens internes et externes,
pas seulement aux émotions, indispensables à notre
et peut influencer nos expérience du monde et
interprétations des stimuli. à nos sensations.

ESTIMATIONS RÉPONSE PHYSIOLOGIQUE


Nos interprétations, centrales Modifications corporelles qui
pour les émotions, peuvent font partie de notre expérience
être causales, quand nos émotionnelle et impliquent des
pensées en relation avec un mécanismes dans les nerfs et/ou
stimulus nous placent dans le système nerveux.
une disposition particulière.

NEURONES DU VISAGE
Dans le cerveau du macaque, six
RÉPONSES

COMBAT- FUITE régions différentes contiennent


Réponse instinctive aux des neurones qui s’activent durant
situations de stress: l’adrénaline la reconnaissance des visages, en
afflue, la respiration et le pouls fonction de différences de forme
s’accélèrent, préparant l’individu ou d’apparence (Tsao et Chang).
à combattre ou à fuir pour
retrouver la sécurité.

LOI DE YERKES ET DODSON


Nos performances augmentent
avec l’éveil physiologique ou
mental, mais quand l’excitation
est trop forte, elles déclinent.

EXCITATION
PROPRIOCEPTION
SYNESTHÉSIE
Mélange permanent des SYNAPSES
sens, la stimulation de Petits intervalles entre
l’un entraînant celle d’un les neurones ou entre un
ou plusieurs autres. neurone et un muscle ou
une glande, où les signaux
nerveux sont relayés, souvent
par des neurotransmetteurs.

HIPPOCAMPE
Structure dans le lobe
temporal du cerveau;
centrale pour la mémoire
GYRUS FUSIFORME NEUROTRANSMETTEURS et l’apprentissage, mais
Aire du cerveau à la base Messagers chimiques qui également impliquée dans
du crâne impliquée dans permettent aux neurones les émotions et les désirs.
la reconnaissance des de communiquer entre eux,
visages et la perception ou avec des glandes ou des
des couleurs. muscles, via une synapse; CELLULES DE LIEU
ils se lient à des récepteurs Cellules spécialisées
sur le neurone adjacent. de l’hippocampe qui
s’activent quand un animal
se trouve dans un certain
lieu (O’Keefe); découverte
PROSOPAGNOSIE central pour l’étude de la
Altération de la carte cognitive.
reconnaissance des
visages, issue de lésions
dans le gyrus fusiforme.

THÉORIE DES NEURONES


DE WALDEYER
Pose le système nerveux comme
étant constitué de cellules
cérébrales individualisées,
séparées par un vide.

SYSTÈMES NERVEUX
Le système sympathique
consiste en nerfs, glandes et
organes, il contrôle la réponse
combat-fuite; le système
parasympathique prépare le
corps au repos et à la digestion.
ons-nos bien
cinq sens?
Nous entendons, goûtons, sentons,
voyons et touchons, mais est-ce bien tout?
Les cinq sens sont un dogme bien enraciné.
Mais ce n’est pas si simple. Creusez un peu
plus et vous découvrirez que le nombre
exact est encore à déterminer.

Un sens est un système biologique Nous avons beaucoup de sens internes


qui nous aide à obtenir des également, qui détectent et réagissent aux
informations sur le monde et à changements dans nos corps. La faim et a soif
réagir à ses changements. On estime que sont les plus évidents, mais nous sentons aussi
l’idée des cinq sens remonte à Aristote, car a douleur et l’envie d’aller aux toilettes. Nous
son livre De l’ âme se divise en chapitres contrôlons et sentons inconsciemment bien
dédiés à a vision, l’ouïe, le toucher, l’odorat et d’autres signaux encore, tels que le battement
le goût. Nous savons aujourd’hui qu’elle est du cœur, a respiration et a pression artérielle.
trop simpliste. En tenant compte de tous, les scientifiques
Fermez les yeux et touchez votre nez avec estiment de nos jours qu’il pourrait y avoir plus
votre index. Cea n’est possible que grâce à de cinquante sens différents!
un autre sens corporel, a proprioception, qui De plus, les sens s’influencent les uns les
nous donne une conscience de a position autres. La couleur et a texture de a nourriture
de nos membres dans l’espace. Ce sont des affectent souvent son goût, par exemple. Chez
récepteurs spécialisés de nos muscles qui certains, les sens sont toujours méangés. Cea
indiquent au cerveau ce qu’il doit savoir pour s’appelle a synesthésie, qui peut prendre bien
ça. L’équilibrioception est un autre sens. des formes. Par exemple, certains goûtent les
Il s’appuie sur le liquide que contient l’oreille sons, d’autres entendent les couleurs. Environ
interne et nous aide à déterminer a gravité et 5% des gens ont une forme de synesthésie,
l’accélération, afin que nous sachions toujours mais c’est si personnel qu’on ne s’en aperçoit
où est le haut, et si nous accélérons ou pas toujours. Bien des synesthètes estiment
ralentissons. Ce sont deux exemples de sens simplement que tout le monde perçoit le
externes, en plus des cinq cassiques. monde à leur manière.

74 PSYCHOLOGIE BIOLOGIQUE
SYNESTHÉSIE
DU MIROIR

La synesthésie se produit quand un sens en


déclenche un autre. Il en existe de nombreux
types. Certaines personnes entendent les
mots comme des couleurs, ou a musique
comme des formes. L’une des variétés les
plus étranges est a synesthésie du miroir,
des gens qui sentent qu’on les touche quand
ils voient qu’on touche quelqu’un d’autre.
Quand ils voient qu’on tape sur l’ épaule
d’une autre personne, ils ont l’ impression
qu’on leur tape sur l’ épaule aussi.
os os sene bien?
La façon dont nous produisons puis
vivons un sentiment est liée à nos sensations
physiques, nos réactions à celles-ci, nos
comportements, nos pensées et a signification
que nous leur donnons. Modifiez l’un de ces
éléments et vous changerez de sentiment.

Nos changements internes et nos Nous comprenons les émotions comme


réponses physiologiques, respiration, un méange de stimuli, d’états corporels et
pulsation cardiaque, activité d’affects. Les affects sont notre expérience
neurochimique du système nerveux, sont des sensations, des émotions ou des humeurs.
partie intégrante de nos émotions. William Ils contribuent à une variété d’états mentaux,
James (1884) estimait que nous avons peur en pas seulement aux émotions. Les armes et
raison de ces réponses physiologiques; l’estomac noué sont-ils un signe de tristesse ou
d’autres pensent que les émotions de base de faim quand on découpe des oignons? Les
provoquent une réponse spécifique, qui varie affects influencent aussi nos interprétations
selon les individus et a culture. des stimuli. Nous percevons différemment
Nos évaluations ou nos interprétations une expression faciale selon nos humeurs
pèsent beaucoup sur nos émotions. Si nous ou nos biais d’interprétation. Si le biais
voyons un serpent et que nous interprétons d’interprétation est négatif, un geste ambigu
sa présence comme une menace, a réponse tel qu’un sourire pourra être vu comme une
physiologique de a peur s’ensuivra. Ici, moquerie plutôt qu’un signe amical.
l’évaluation du stimulus, du serpent, a été Ces théories nous écairent sur les
causale, les pensées qui lui étaient liées nous manières de mieux guider nos expériences
ont fait ressentir de a peur. Notre réponse émotionnelles en modifiant nos modes
physiologique et son évaluation peuvent de pensée, nos comportements ou nos
dépendre du contexte au sein duquel nous sensations. Quand nous tombons amoureux,
en faisons l’expérience. Par exemple, un cœur a sensation de panique et de nervosité est vue
qui bat vite et les mains moites peuvent être comme de l’excitation. Pourrions-nous voir a
évalués comme de l’anxiété avant un examen, panique comme de l’excitation dans d’autres
mais aussi de l’excitation dans un grand-huit. situations, et ainsi calmer nos nerfs?

76 PSYCHOLOGIE BIOLOGIQUE
BIAIS D’INTERPRÉTATION

Dans ce scénario, a personne qui tient le


serpent interprète a situation comme sans
risque et se sent donc à l’aise. Celle qui
a regarde avec ce serpent autour du cou
interprète a situation comme une menace
et en ressent de a peur.
Trions-nos miex
sos  pression?
Le stress peut donner les mains moites
et le souffle court. Personne n’aime se
sentir débordé au travail, mais les études
montrent qu’un peu de stress peut parfois
contribuer à améliorer les performances.

Que ce soit passer un examen ou qui a poussé les psychologues à formuler a


parler en public, les situations loi de Yerkes et Dodson. Elle indique que les
stressantes déclenchent parfois une performances s’améliorent avec l’excitation
série de signaux corporels gênants. Le niveau physiologique ou mentale, jusqu’à un certain
d’adrénaline monte, le pouls s’accélère, le point. Quand le niveau d’excitation est trop
souffle devient court. C’est a réponse élevé, les performances déclinent. Il est bon,
combat-fuite, qui prépare les individus à se par exemple, d’avoir un peu le trac avant de
défendre et combattre ou à fuir vers a prendre a parole en public, mais pas au point
sécurité, une stratégie de survie involontaire d’oublier son discours. Par ailleurs, le niveau
et ancienne contrôlée par un paquet de nerfs, d’excitation optimal dépend de a complexité
de gandes et d’organes nommé le système de a tâche et de sa familiarité. Il est très
nerveux sympathique. Le système nerveux possible de passer l’aspirateur dans le salon
parasympathique, lui, nous calme. Le souffle en écoutant une histoire d’horreur, mais on
et le pouls ralentissent. Il prépare le corps à ne voudrait pas qu’un chirurgien fasse pareil
se reposer et digérer. Ces deux systèmes lors d’une opération, par exemple.
antagonistes sont les pôles opposés du La loi a gardé son influence malgré de
spectre de l’excitation. Lequel vaut mieux? multiples critiques parues sur l’étude de
Des recherches ont montré que, en 1908. On l’a appliquée depuis à de nombreux
matière de performance, un peu de stress domaines de a performance humaine, en
serait bénéfique. Dans une étude de 1908, particulier au travail. Les critiques font ici
Robert Yerkes et John Dodson ont trouvé que remarquer qu’elle peut servir à légitimer
les souris apprenaient mieux sous l’effet d’un l’augmentation volontaire du stress, alors que
choc électrique «modéré» que sous celui des études récentes ont confirmé que cea
d’un choc «léger» ou «extrême». C’est ce détériorait a santé physique et mentale.

78 PSYCHOLOGIE BIOLOGIQUE
PERFORMANCE

Optimum

Vigilance
Anxiété

Sommeil Désorganisation

STRESS

STRESS ET PERFORMANCE
Selon a loi de Yerkes et Dodson, en quantité Il existe donc un niveau optimal de stress,
modérée le stress améliore a performance. situé quelque part entre a vigiance
Il est impossible d’ être performant quand on minimale et l’anxiété maximale, dans lequel
est si détendu qu’on s’endort, ou quand on notre attention est à son pic et notre jeu à
est si stressé que nos pensées se brouillent. son meilleur.
Commen es droes
on-ees pner?
Aucune drogue ne fait vraiment paner,
mais beaucoup altèrent notre fonctionnement.
Aiguisant a concentration ou déclenchant une
fringale, les drogues modifient l’équilibre
chimique délicat de notre cerveau.

À a fin du XIXe siècle, Santiago glutamate, augmentent l’activité neuronale.


Ramón y Cajal, neuroscientifique Ce sont des excitateurs. D’autres, comme
espagnol, a regardé dans un l’adénosine et a sérotonine, a ralentissent. Ce
microscope et dessiné ce qu’il voyait. Il a ainsi sont des inhibiteurs. Nous connaissons plus de
produit des centaines de belles images de 50 neurotransmetteurs différents.
cellules cérébrales. Ses expériences ont Les psychotropes fonctionnent en se liant
montré qu’il y avait des vides entre ces à ces récepteurs. Ils modifient l’équilibre
cellules, ce qui a poussé Heinrich Waldeyer à des neurotransmetteurs dans le cerveau. On
formuler sa théorie du neurone, qui édicte compte parmi eux des substances usuelles,
que le système nerveux est composé de comme a caféine, des médicaments prescrits,
cellules cérébrales séparées entre elles, tels que les antidépresseurs, ou des drogues à
vaable encore aujourd’hui. Le cerveau et a usage récréatif comme a marijuana.
moelle épinière sont en effet constitués de La caféine se lie aux récepteurs de
milliards de ces neurones, qui sont séparés les l’adénosine et bloque leur activité, ce
uns des autres par de petits intervalles qui fait chuter le taux d’adénosine. Or, a
appelés les synapses. hausse de ces taux rend a plupart des gens
Les neurones communiquent entre somnolents. La caféine s’y oppose, ce qui
eux et passent l’information dans tout donne cette sensation d’éveil, d’attention
le corps. L’information voyage dans les disponible. Elle cause aussi un rétrécissement
neurones sous forme électrique, mais, des vaisseaux sanguins, et provoque donc
quand elle atteint une synapse, elle doit parfois des migraines. Les antidépresseurs,
être convertie en signal chimique afin eux, se lient aux récepteurs de a sérotonine
de franchir l’espace entre deux cellules et augmentent pour un temps a disponibilité
et continuer son chemin. Ces molécules de celle-ci, tandis que a molécule active de
chimiques sont les neurotransmetteurs, qui a marijuana, le tétrahydrocannabinol (THC),
se lient à des protéines spécialisées sur le se lie aux récepteurs cannabinoïdes pour
neurone adjacent, les récepteurs. Certains produire un effet reaxant et augmenter les
neurotransmetteurs, comme l’adrénaline et le niveaux de dopamine.

80 PSYCHOLOGIE BIOLOGIQUE
PARCOURS DU
SIGNAL NERVEUX
Le signal nerveux se dépace le long des
neurones sous forme de signal électrique,
jusqu’ à ce qu’ il atteigne un vide appelé
synapse. Pour franchir ce vide et atteindre
le neurone suivant, il faut le convertir en un
signal chimique appelé neurotransmetteur.
Une fois de l’autre côté, il est reconverti en
signal électrique et poursuit son chemin.
Comme une voiture qui, parvenue au bout
de a route, doit prendre un ferry pour
traverser une rivière (qui représente ici a
synapse), et qui, une fois sur l’autre rive, peut
poursuivre son chemin.
Commen reconnîre
n ise?
Nous reconnaissons les gens surtout à
leur visage. Des cellules spécialisées du
cerveau réagissent à des critères faciaux
spécifiques, ce qui nous aide à reconnaître
un visage connu.

Pour sourire à son voisin et faire a Des études d’imagerie, où des patients
grimace à son chef, il est sains allongés dans un scanner regardent des
indispensable de reconnaître leur visages, ont depuis confirmé l’importance de
visage. C’est une compétence que nous cette région du cerveau, et des études sur les
acquérons presque tous assez tôt et que nous primates ont été encore plus loin en
conservons bien, malgré quelques gaffes repérant certaines des cellules individuelles
gênantes au passage, même dans le grand responsables. Le cerveau d’un macaque
âge. Cea nous aide à déterminer non adulte contient des milliards de neurones,
seulement l’identité des gens, mais aussi ce mais reconnaître les visages pourrait n’en
qu’ils ressentent et ce qu’ils pensent. impliquer que 200. Les travaux de Doris Tsao
Reconnaître les visages constitue ainsi une et Steven Le Chang ont montré que ces
part importante des interactions sociales. cellules sont localisées dans six régions
Les scientifiques ont beaucoup appris de différentes. Certaines réagissent aux
cette compétence en étudiant ceux qui différences spatiales, a distance entre les
l’avaient perdue, suite à des lésions cérébrales yeux ou a taille de a bouche par exemple,
ou à une maadie. Le célèbre neurologue d’autres aux différences de matière, par
Oliver Sacks a ainsi décrit «l’homme qui exemple le teint ou a texture de a peau.
prenait sa femme pour un chapeau». Un Le cerveau des macaques étant très
patient de Sacks souffrait d’agnosie visuelle, sembable à celui des humains, nous
un trouble rare qui le rendait incapable de pouvons extrapoler à partir de ces
reconnaître de visu des personnes ou des recherches jusqu’à nous: il se pourrait que
objets familiers. De même, les gens qui nos cerveaux traitent les visages comme
souffrent de prosopagnosie ne reconnaissent a somme subtile de plusieurs groupes
pas les visages, y compris le leur. Bien souvent, d’éléments plutôt que comme une seule
ce qui explique ces troubles est une lésion du entité. La prochaine fois que vous ignorez
gyrus fusiforme, région clé du cerveau située votre patron «sans faire exprès» en le
vers a base du crâne, apparemment formée croisant dans le couloir, vous pourrez blâmer
pour réagir aux visages. vos neurones!

82 PSYCHOLOGIE BIOLOGIQUE
MONSIEUR LÉGUME

Que voyez-vous? Pour a plupart, a réponse Ces troubles peuvent être héréditaires ou, plus
est évidente: un visage composé de légumes. rarement, être le résultat de lésions cérébrales
Mais pour les environ 2% d’entre nous qui ou d’une maadie. Les sujets atteints se
souffrent de troubles de a reconnaissance reposent alors sur des informations n’ayant
des visages, il n’y en a pas dans cette image; pas trait aux visages, par exemple a voix ou
ce sont les ingrédients d’un curry de légumes. les vêtements, pour reconnaître les gens.
Commen n xi se
soien des rccorcis?
Le temps et les efforts qu’un chauffeur
de taxi accorde à mémoriser le pan d’une
ville font grandir des régions de son cerveau,
avec des résultats spectacuaires sur son sens
de l’orientation.

Le compteur tourne et voilà que le Des études ont montré que, chez les
taxi prend à gauche sans prévenir. Il espèces qui cachent souvent leur nourriture,
vous assure que c’est plus court par l’hippocampe est en moyenne plus gros que
là, et avant même que vous ayez protesté, chez les espèces de même taille qui n’ont pas
vous voici à destination. Donnez-lui un besoin de se souvenir en détail d’informations
pourboire et dites merci au Knowledge! spatiales, ce qui a poussé Eleanor Maguire à
Le Knowledge est le célèbre test pour se demander s’il en alait de même pour les
les chauffeurs de taxi londoniens. Avant chauffeurs de taxi. Dans une étude de 2000,
d’être ancés dans les rues, ils doivent elle a comparé leurs cerveaux à ceux d’un
mémoriser les 25000 routes et les milliers groupe de contrôle à l’IRM et découvert qu’ils
de lieux connus. Ils passent de nombreux avaient en effet un plus gros hippocampe.
examens au fil de plusieurs années et, ceci Cea suggérait un lien entre taille de
fait, ils connaissent tous les raccourcis et les l’hippocampe et mémoire spatiale, mais cea
chemins de traverse sans jeter un œil à leur ne prouvait pas qu’apprendre le Knowledge
GPS. Comment font-ils? causait le changement. Peut-être qu’un gros
Les scientifiques soupçonnent ici hippocampe poussait leur propriétaire à
un rôle de l ’hippocampe, une structure devenir chauffeur de taxi...
cérébrale du lobe temporal. Il y a cinquante Maguire a scanné le cerveau de chauffeurs
ans, John O’Keefe a prouvé l’existence des débutants avant et après l’entraînement et
cellules de lieu: des cellules spécialisées comparé les scans à leurs scores lors du test.
dans l’hippocampe qui s’activent quand un Avant le Knowledge, a taille de l’hippocampe
animal arrive dans un endroit particulier. Sa ne varie pas; après, il grossit chez ceux qui
découverte a abouti sur l’idée des cartes l’ont obtenu. Apprendre le Knowledge fait
cognitives, les représentations mentales bien grossir le cerveau, et c’est comme ça que
que nous formons des lieux et des chemins. les taxis se souviennent des raccourcis.

84 PSYCHOLOGIE BIOLOGIQUE
LE KNOWLEDGE

Apprendre a carte des routes de Londres, emmagasinent eux aussi beaucoup


autrement dit passer le Knowledge, modifie d’ informations. Ici, le changement est une
le cerveau des chauffeurs de taxi. Une réponse à l’apprentissage de données de
structure clé de l’apprentissage et de a navigation, que le cerveau emploie pour
mémoire, l’hippocampe, grossit. Ce n’est construire une carte cognitive très détaillée
pas le cas pour les médecins, qui pourtant de son environnement proche.
HAPITRE 5 PSYCHOLOGIE DU
DÉVELOPPEMENT

IDENTITÉ

ÉGOCENTRICITÉ

STRESS
PERSONNEL

INTERACTION
SOCIALE

CONFUSION
CONFLIT
DES RÔLES
CHAPITRE 5

DÉELOPPEET
PSYCHOLOGIE DU
ATTACHEMENTS

DÉVELOPPEMENT
PSYCHOSOCIAL
INTRODUCTION

I
maginez devenir plus fin, plus grand, plus capable et
plus curieux en deux ans à peine. Ce n’est pas facile
d’imaginer un tel changement en si peu de temps.
Pourtant, vous avez déjà changé comme ça par le
passé, et même plusieurs fois. Nous ne nous développons
jamais plus rapidement que durant l’enfance. Ce chapitre
se consacre aux aspects fascinants de a croissance
et de l’apprentissage chez l’enfant, aux stades de
développement fondamentaux qu’il traverse sans même
s’en rendre compte.
Bowlby a noté que nos premières reations devaient
nous apporter plus que de a nourriture si nous voulions
survivre et nous épanouir. Le confort, a protection et a
sécurité sont des ingrédients clés pour créer des liens sains
et durables. MARY AINSWORTH a étudié comment les
enfants se lient à leur mère et pourquoi c’est important.
En observant des enfants réagir quand leur mère et un
étranger rejoignent ou quittent une pièce, elle a distingué
quatre styles d’attachement, du plus sûr au moins certain.
Ce dernier paraît lié à de futurs problèmes reationnels,
d’estime de soi et de santé mentale. Est-ce qu’il rend aussi
plus vulnérable au stress?
Avant de comprendre comment les gens surmontent
le stress, les psychologues ont dû trouver une manière de
le mesurer. À peu près en même temps que a théorie de
l’attachement se développait, Holmes et Rahe ont tenté de
mesurer dans le détail les effets des grands événements,
tels que les déménagements, deuils et divorces. Même

88 PSYCHOLOGIE DU DÉVELOPPEMENT
si on peut les relier à une dégradation de a santé plus
tard, Kanner a découvert que nos ENNUIS QUOTIDIENS
étaient bien plus problématiques qu’une période ponctuée
d’ÉVÉNEMENTS intenses. Depuis, les psychologues ont
découvert qu’un mode d’attachement insécure handicapait
beaucoup quand il s’agissait de soutenir un stress intense.
Les premières expériences qui nous pacent dans ce mode
peuvent rendre très inconfortables les stratégies ultérieures
de recherche d’aide, par exemple.
Quel que soit le lien qui se forme entre le parent
et l’enfant, celui-ci ne pourra voir le monde que par ses
propres yeux. Sur son chemin vers l ’âge adulte, il franchira
ce que Piaget a appelé les stades du développement
cognitif et se formera une THÉORIE DE L’ESPRIT. À
l’adolescence, il comprendra que les autres vivent, pensent
et ressentent différemment de lui, et pourra même inférer
leurs pensées et leurs sentiments.
La théorie d’Erikson du développement psychosocial
couvre toute l ’étendue de a vie. Bébés, nous apprenons
a confiance et a méfiance, sous le signe de l’espoir. Passé
65ans, nous nous concentrons sur l’acceptation, contre le
désespoir, sous le signe de a sagesse.
On peut trouver des significations à tous les gestes
d’un enfant, jusque dans ses jeux. Le jeu apprend aux
enfants à donner et recevoir, à communiquer. Dès 5ans, ils
sont capables de coopérer au cours de leur jeu, ce qui est
très important pour former leur capacité à colaborer et
essentiel au succès de notre espèce au sens arge.

PSYCHOLOGIE DU DÉVELOPPEMENT 89
PSYCHOLOGIE DU DÉVELOPPEMENT

COLLABORATION
Assigner et accepter les rôles
AUTISME distincts que chacun joue pour
Trouble du atteindre un objectif commun.
développement qui
dure toute la vie et affecte
la communication et les
interactions avec le monde;
JUDITH DUNN
Psychologue britannique (1939–)
un déficit de théorie de l’esprit
spécialisée dans la psychologie
est fréquent.
sociale du développement et la
compréhension sociale chez le
jeune enfant.

RELATIONS SOCIALES
THÉORIE DE L’ESPRIT
Liens formés avec des
Capacité de deviner les
personnes en dehors du foyer,
croyances, les émotions
au travers desquelles les enfants
et les pensées des autres,
apprennent à jouer de manière
employée dans toutes
«socialement acceptable».
les situations sociales.

FAUSSES CROYANCES
Opinions erronées issues de
raisonnements ou d’hypothèses
incorrects; aspect important de
la théorie de l’esprit.

ÉGOCENTRICITÉ
Percevoir le monde de son
TRACAS QUOTIDIENS seul point de vue et penser
Usure de tous les jours; l’échelle que les autres voient le
de Kanner (1981) se concentre sur monde comme nous.
117 de ces corvées pour évaluer
la probabilité d’apparition de
troubles liés à l’anxiété.

ÉCHELLE DE RÉAJUSTEMENT ÉVÉNEMENTS MAJEURS


SOCIAL (SRRS) Les expériences stressantes
Outil d’auto-évaluation de Holmes significatives de la vie. Holmes et Rahe
et Rahe (1967) pour la mesure du en incluent 43 dans leur questionnaire
stress, où les événements majeurs quantitatif du stress (1967) et leur
se comptent en «unités de assignent des valeurs relatives.
changement».

STRESS
DYNAMIQUE
THÉORIE DE
L’ATTACHEMENT
PERTURBATION LUDIQUE DE BOWLBY
Tester les limites des situations Liens développés avec ceux
sociales en externalisant un qui s’occupent de nous. Les liens
comportement. précoces nous apportent non
seulement la nourriture, mais
aussi le confort, la protection
et la sécurité.

REPRÉSENTATIONS MARY AINSWORTH


Modes de pensée qui nous aident à Psychologue canadienne
apprendre, depuis la représentation (1913–1999) qui a défini trois
énactive au début de la vie, puis la modes d’attachement chez
représentation imagée et enfin la les enfants entre 9 et 18 mois:
représentation symbolique (Bruner). sécure, insécure–évitant et
insécure–ambivalent.

STADES
VERTUS
Huit traits désirables qui peuvent
être acquis à chacun des huit
stades de la vie: espoir, volonté,
détermination, compétence,
fidélité, amour, sollicitude et
sagesse (Erikson).

CONFLITS PSYCHOSOCIAUX
Incompatibilité entre nos
besoins psychologiques et notre
environnement social. Leur
CONCEPT DE SOI résolution, ou leur non-résolution,
Sens de la direction que prend influence le développement de
notre vie, foi en nos propres notre personnalité (Erikson).
croyances et conscience de notre
rôle dans la société.
Es-ce qe e sress
coe ssi es mrins?
Un stress prolongé peut faire couler tout
le monde, marins inclus, mais il prend bien
des formes. Les psychologues ont conçu
plusieurs échelles pour le quantifier, lui et
les conséquences qu’il fait peser sur nos vies.

La vie est un parcours d’obstacles, suivantes. Plus le score et donc a gravité des
ponctué de défis et de surprises. événements vécus augmentent, plus sera
Certains changements, comme a fréquente a survenue de troubles mentaux.
naissance d’un bébé, ou un accident, sont Il semblerait bien que le stress puisse couler
source de stress, mais à quel point? Et quelles même les marins.
en sont les conséquences? L’American Institute of Stress considère
En 1967, les psychiatres Thomas Holmes qu’un score annuel au-dessus de 300
et Richard Rahe ont conçu un questionnaire donne 80% de chance de développer des
pour quantifier le stress. Il se concentre sur troubles liés au stress, tels qu’une anxiété
les grands événements, les expériences pathologique ou une dépression, dans les
susceptibles d’en produire le plus dans deux ans. Cependant, même si le stress
nos vies. Ils ont ainsi assigné des valeurs à chronique paraît bien contribuer à ces
43événements, selon leur impact potentiel. problèmes, les preuves manquent pour
La mort du conjoint est le plus élevé, avec un confirmer a pertinence du cassement.
indice de 100 «unités de bouleversement», D’une part, ces événements sont rares,
le divorce arrive en deuxième pace, avec73. si bien que se concentrer sur les tracas
Les événements moins dramatiques, comme quotidiens pourrait constituer une meilleure
changer d’école, partir en vacances ou recevoir mesure: angoisser sur son poids, perdre son
une amende de stationnement, ont des scores téléphone. En gardant cea en tête, A.D. Kanner
inférieurs à 20. Ils ont appelé cea l’Échelle de a conçu une autre échelle en 1981, qui identifie
réajustement social (ERS). 117 de ces tracas quotidiens et il a découvert
Trois ans plus tard, Rahe a testé son que a corréation entre ces tracas et les
questionnaire sur 2500 marins américains. Il a troubles liés au stress était supérieure à celle
découvert une corréation positive entre leurs constatée par l’ERS. De multiples sources de
scores et leur santé mentale dans les années stress mineur font aussi couler les marins...

92 PSYCHOLOGIE DU DÉVELOPPEMENT
ÉVÉNEMENTS MAJEURS
Perdre un être cher ou faire de a prison
est évidemment source de stress, mais des
événements supposés heureux le sont aussi.
Holmes et Rahe ont inclus des événements
aussi bien positifs que négatifs dans leur
Échelle de réajustement social. Une grossesse
ou des vacances, par exemple, sont des
événements à a fois positifs et stressants.
L’ échelle révèle l’effet cumuatif du stress,
donc le fait que de multiples événements
mineurs peuvent avoir autant d’ impact qu’un
événement majeur.

1. Mort du conjoint
2. Séjour en prison
3. Grossesse
4. Nouvelle école
5. Vacances

1  3  5

100 63 40 26 13
UNITÉS DE BOULEVERSEMENT
Doi-on êre sr
qnd on s’che?
Oui. Les attachements ne sont pas
toujours bons, et plus ils sont sûrs, mieux c’est.
Chez l’enfant, les attachements précoces avec
les adultes qui prennent soin de lui auront des
conséquences pour toute sa vie. S’ils sont
sécures, il gagnera plus tard en bien-être.

En termes psychologiques, un de l’attachement s’est retrouvée sous le feu


attachement ne se commande pas. des critiques pour son manque d’universalité
Selon a théorie de l’attachement, il culturelle. Keller (2018) a souligné que a
s’agit du lien émotionnel et physique qu’on théorie se fondait sur a structure de a
tisse avec ceux qui s’occupent de nous. Cea famille nucléaire occidentale de casse
se construit avec le temps et dépend de a moyenne typique, dans aquelle un enfant
nature et de a fréquence des soins qu’on crée un lien avec un adulte principal et où les
reçoit, ainsi que de leur style d’attachement. émotions sont reçues et encouragées d’une
La théorie de l’attachement a été certaine manière bien précise. Ce n’est pas
développée dans les années 1950 par John comme ça que le monde entier élève ses
Bowlby. Il a étudié a reation mère-enfant enfants. Certaines cultures s’appuient sur
chez les singes et montré l ’importance des stratégies parentales communautaires et
d’une reation précoce, non seulement pour répondent différemment aux émotions.
l’approvisionnement en nourriture, mais aussi Que signifient les styles d’attachement?
en confort, en protection et en éducation. Les études suggèrent que les styles
Mary Ainsworth a étendu les théories de développés dans l’enfance auront un impact
Bowlby dans les années 1970 et a proposé sur a vie de l’adulte. Les attachements
trois styles d’attachement chez l’enfant entre insécures ont été associés à une faible
9 et 18mois: sécure, insécure–évitant et estime de soi, des difficultés reationnelles
insécure–ambivalent. Pour déterminer le style et des risques accrus de troubles mentaux.
d’attachement des enfants, Ainsworth a analysé La théorie de l’attachement est ainsi
leur réaction quand leur mère ou un inconnu employée chez l’adulte pour comprendre son
entraient ou sortaient de a pièce. Un quatrième expérience de vie et informer le diagnostic et
style d’attachement a été ajouté plus tard pour y le traitement de troubles mentaux. Le style
pacer les enfants qui ne rentraient dans aucun d’attachement chez l’adulte est mesuré lors
des trois autres styles: désorganisé. d’un «entretien d’attachement chez l’adulte»
Même si elle a été très popuaire et très et cassé en détaché, autonome, préoccupé
consolidée par de multiples études, a théorie ou désorganisé.

94 PSYCHOLOGIE DU DÉVELOPPEMENT
STYLES D’ATTACHEMENT
Les quatre styles d’attachement chez les catégoriser. Il faut noter toutefois que
l’enfant tels qu’observés par a psychologue l’expérience était limitée à une organisation
Mary Ainsworth dans son expérience de familiale nucléaire de casse moyenne
a «situation étrange», qui lui a permis de cassique, typiquement occidentale.

SÉCURE INSÉCURE –AMBIVALENT


Angoissé au départ du parent. Très angoissé au départ du parent.
Évitant l’inconnu quand il est seul, mais amical en Évite et craint l’inconnu.
présence du parent. Cherche le parent, mais refuse le contact.
Positif lors du retour du parent. Difficile à rassurer.
Le parent peut rassurer l’enfant.

INSÉCURE –ÉVITANT DÉSORGANISÉ


Pas angoissé au départ du parent. Angoissé à l’approche du parent.
Pas angoissé en présence de l’inconnu, joue Confus ou désorienté.
normalement. Comportements contradictoires, par exemple crise de
Neutre lors du retour du parent. colère suivie de confusion.
Parent et inconnu peuvent rassurer l’enfant.
Qe e  ’eo?
Les jeunes enfants sont égocentriques et
ne voient le monde que de leur seul point de
vue. Mais ils se développent vite et passent
par quatre stades. À l’adolescence, ils auront
compris que les autres aussi ont des pensées
et des sentiments, différents des leurs.

En 1936, le psychologue suisse Piaget a fois (c’est a centration) et il est toujours très
a défini quatre stades fixes du absorbé par son propre monde visuel. Il peut
développement cognitif par se servir du angage dans ses jeux, mais pour
lesquels tout enfant passera entre 0 et 11ans. externaliser sa propre pensée plutôt que pour
En franchissant ces stades, les enfants communiquer avec les autres.
découvrent activement leur environnement, Le stade3, le stade opératoire concret
ils l’explorent et ainsi développent leurs (entre 7 et 11ans), marque le début de a
représentations mentales (voir les schémas, pensée logique et raisonnable, et de son
page62) du monde. Et selon Piaget, à mesure application aux objets physiques. L’enfant
qu’il les franchit, son égocentricité réduit. commence à comprendre que ses pensées et
Qu’est-ce que l’égocentricité? C’est a ses sentiments sont uniques et que les autres
tendance à percevoir le monde de notre seul ont les leurs propres, mais il n’est pas toujours
point de vue et de supposer que les autres capable de déterminer ou de s’intéresser à
voient le monde comme nous. Si les enfants l’expérience d’autrui.
sont égoïstes, c’est parce qu’ils sont encore en Le dernier stade, le stade formel (après
développement! 11ans), passe par l’application de a pensée
La première étape de a réduction de logique à des problèmes abstraits (c’est a
l’égocentricité est le stade sensorimoteur pensée abstraite). La capacité de penser par
(de 0 à 2ans). Un enfant relie ses sens à ses hypothèse aide l ’enfant à comprendre les
capacités motrices pour construire son points de vue différents du sien.
intelligence. Il commence par apprendre que Est-ce que nous nous arrêtons là dans
les objets existent même quand il n’est pas là notre développement? Oui, selon a théorie
(c’est a permanence des objets). de Piaget, mais d’autres pensent que nous
Puis vient le stade préopératoire (entre 2 continuons à développer nos schémas
et 7ans). L’enfant explore au travers de jeux mentaux et nos connaissances encore après,
symboliques ou de jeux de rôle. Il tend à se par nos interactions avec les autres dans
concentrer sur un seul aspect des situations à notre environnement social.

96 PSYCHOLOGIE DU DÉVELOPPEMENT
EXERCICE DES TROIS MONTAGNES

POINT

POINT

A
POINT

En se fondant sur a connaissance de


a scène complète, est-ce qu’un enfant
comprend ce que voit quelqu’un depuis le
point C ? Un enfant de 4 ans ne comprend
que ce qui est dans son champ de vision
(point A) ; à 7 ans sa perspective devient
plus objective (point B), mais ce n’est qu’ à
11 ans (le stade formel de Piaget) qu’ il pourra
correctement identifier un point de vue
différent du sien (point C). Comme si chaque
stade était une brique pour construire
un point de vue moins égocentrique.
Porqoi es bébés
imen-is es hoches?
Le développement cognitif est le passage
du concret à l’abstrait; des hochets aux jeux
d‘imagination. Nous passons d’une
connaissance fondée sur l’action à une autre,
fondée sur les symboles ou les images.

Jerome Bruner s’intéressait à a Entre un et six ans, nous nous mettons à


construction, a représentation et faire usage de représentations imagées – nous
l’organisation de a connaissance, et gérons les concepts via des images. Ce n’est
aux différents modes de pensée ou de pas toujours un stockage conscient, mais
représentation qui nous aide à développer il nous aide à digérer l’information, comme
notre apprentissage. le cas d’un schéma en camembert pour mieux
La représentation énactive est employée comprendre les fractions.
dans a première année de vie. C’est elle qui Le stockage fondé sur des actions fixées
nous indique pourquoi un bébé aime agiter ou des images (représentation énactive
un hochet. Notre pensée est alors fondée et imagée) est plutôt restrictif, car il est
sur l’action physique: nous apprenons en difficile d’employer ces connaissances dans
faisant plutôt qu’en raisonnant. Les bébés un contexte différent. Les représentations
se servent de leurs capacités motrices pour symboliques, où l’information est sous forme
agiter le hochet, tout en activant plusieurs de code ou de symbole comme dans le cas
sens, l’ouïe qui entend le bruit du hochet du angage, sont plus flexibles et faciles à
et a vue qui observe son mouvement. Non manipuler. Par exemple, nous apprenons qu’un
seulement c’est stimuant et amusant, mais en chien est une casse d’animal, ou comprenons
plus le hochet symbolise l’apprentissage et que + veut dire additionner. La pensée
a mémorisation de a connaissance. Le bébé symbolique se développe à partir de 7ans.
apprend que cette action donne ce résultat et Quelles conséquences pour l’éducation?
il développe une mémoire muscuaire en lien À a différence de Piaget (page 62), Bruner
avec cea, si bien que quand il reprendra le pensait qu’il était possible de tout apprendre
hochet en main, il saura quoi en faire. à tout âge, qu’il suffisait de structurer
Ce genre de représentation, fondée sur l’information selon le type de représentations
l’action, est aussi employée plus tard dans a disponibles, en commençant par des idées
vie quand, par exemple, on apprend à faire de simplifiées et en venant petit à petit à des
a bicyclette. idées plus complexes.

98 PSYCHOLOGIE DU DÉVELOPPEMENT
ÉCHAFAUDAGE
DE BRUNER
En 1983, Bruner et ses collègues ont employé
le terme «échafaudage» pour décrire
a manière dont parents et enseignants
créent un cadre (un échafaudage) pour
l’apprentissage des enfants. L’objectif est
de leur permettre de résoudre, à terme, les
problèmes par eux-mêmes. Comment? En
proposant une action, comme de secouer un
hochet, afin que le comportement soit imité,
en donnant des indices, ou en découpant
le problème en morceaux, comme un
échafaudage qui pourra s’agrandir.
L socibiié es-ee
n e d’enn?
Oui, répond Judith Dunn, spécialiste de
a psychologie du développement social.
Les recherches de Dunn explorent les façons
dont les enfants apprennent à interagir de
façon socialement appropriée.

Les enfants grandissent dans un sont plus disposés à partager leurs pensées et
environnement social complexe. leurs sentiments avec des amis, si bien qu’ils
Dans toute structure familiale, il y a en apprennent plus sur les autres.
des rôles, des règles que l’enfant doit découvrir Avec une meilleure communication vient
et apprendre, c’est ce qui les aide à se a colaboration facilitée. Assigner et accepter
développer socialement: le jeu social, a les rôles distincts de chacun dans le cadre
coopération et les taquineries. La qualité des d’une activité commune. Dunn a constaté que
interactions familiales et a capacité de l’enfant dès 4 ou 5ans, les enfants s’engagent dans
à discuter et réfléchir sur ses expériences des jeux coopératifs, comme de construire
forment leur compréhension des normes ensemble un bonhomme de neige. Ces
sociales et des émotions. Dunn a découvert compétences seront utiles quand il s’agira
que les enfants, dès 4ans, étaient capables de plus tard de colaborer à l’école ou dans
se rappeler leurs émotions et leurs pensées d’autres contextes sociaux, comme le sport.
lors de leur premier jour d’école, par exemple: Quand nous connaissons bien quelqu’un,
«J’étais triste, je vouais maman.» nous savons ce qui l’agace et ce qui lui paît,
Les expériences vécues hors de a nous pouvons ainsi le taquiner. Selon Dunn,
maison sont importantes. À 3 ou 4ans, les cea commence (sans surprise) entre frères
enfants gagnent en indépendance vis-à- et sœurs. La perturbation ludique, comme
vis des adultes. Ils ont plus d’opportunités prendre a Nintendo de sa sœur, sert à attirer
de sociabilisation avec d’autres enfants, au l’attention et à rigoler, mais aussi à tester les
travers desquelles ils apprennent à donner et limites de certaines situations sociales. Avec
recevoir afin de jouer de manière socialement l’aide des adultes (parents, enseignants), nous
acceptable. Il y a moins de dynamiques de découvrons ce qui est socialement acceptable,
pouvoir dans ces interactions, les enfants et ce qui ne l’est pas.

100 PSYCHOLOGIE DU DÉVELOPPEMENT


FAMILLES, FOYERS, Les premiers travaux de Dunn se fondaient
sur l’observation des reations entre frères

INTERACTIONS et sœurs dans le cadre familial. Cea incluait


d’ intéressantes observations de dynamiques
familiales à l’ heure des repas. Dès l’ âge
de 2ans, les enfants développent un sens
pratique très cair de a manière d’agacer ou
de consoler leur frère ou leur sœur. L’influence
de a famille et de l’environnement sur a
sociabilisation de l’enfant fait que, de ce point
de vue, les différences individuelles peuvent
être considérables.
Les enns isen-is
dns es pensées?
Les adultes peuvent tenter de deviner
ce que pensent les autres, et savoir qu’ils ne
penseront pas a même chose qu’eux.
Mais ce n’est pas de naissance! C’est une
capacité que nous développerons pour a
plupart autour de 5ans.

Une théorie de l’esprit est a supposeriez que votre ami est tout aussi
capacité de prêter des croyances, en forme que vous et, si ce n’était pas le
des émotions, des pensées à autrui. cas, vous ne comprendriez pas qu’il dise le
Pourquoi n’est-ce qu’une théorie? Nous ne contraire. C’est parfois ainsi que raisonnent
pouvons jamais être sûrs de ce que pensent les personnes autistes; le déficit de théorie
les autres ni de leurs raisons d’agir, mais nous de l’esprit est commun dans l’autisme,
pouvons nous en faire une idée, produire cea se remarque dès l’enfance, durant le
une théorie, de ce qui se passe dans leur développement.
tête. Sans théorie de l’esprit, nous serions Des recherches indiquent cinq étapes
réduits à supposer que tout le monde pense principales menant à a théorie de l’esprit
a même chose que nous. lors du développement, dont comprendre
Nous nous servons de cette théorie tous que les gens ont des désirs et des croyances
les jours, surtout dans les situations sociales. différents, qu’ils peuvent avoir des
Imaginez qu’il est vingt heures et que vous croyances fausses, qu’ils peuvent cacher
et un ami sortiez boire un verre; vous êtes leurs émotions. Ces domaines apparaissent
en pleine forme, mais lui bâille beaucoup autour de 3ans, mais leur ordre varie selon
et lutte pour garder les yeux ouverts. Vous les cultures. Chacun forme une partie
utilisez votre théorie de l’esprit pour deviner importante de a théorie de l ’esprit, et
ses sentiments et vous vous dites: «Il a l’air apparaît en général autour de 5ans.
fatigué, il veut peut-être rentrer.» Vous lui L’exercice le plus simple qu’emploient
demandez, il répond que tout va bien. Vous les psychologues pour étudier le
utilisez votre théorie de l’esprit une nouvelle développement de a théorie de l’esprit
fois pour vous demander si c’est bien ce chez un enfant est de déterminer s’ils
qu’il pense. Sans théorie de l’esprit, vous comprennent les croyances fausses.

102 PSYCHOLOGIE DU DÉVELOPPEMENT


EXERCICE DE CROYANCE FAUSSE
Ce scénario est décrit aux enfants pour voir chercher a balle là où elle l’a aissée. S’ ils
s’ ils comprennent les croyances fausses, un ne comprennent pas, ils vont supposer que
aspect important d’une théorie de l’esprit. Sally sait ce qu’ ils savent, et diront qu’elle ira
S’ ils comprennent, ils diront que Sally va chercher dans le pacard.

Sally met la balle dans


un coffre.

Sally s’en va.

Anne déplace la balle dans


le placard.

Où cherche Sally?
Qe es ore rôe dns
e hére de  ie?
La théorie des stades du développement
psychosocial d’Erikson considère a vie comme
un voyage de l’espoir vers a sagesse, en
passant par toutes les vertus intermédiaires,
au cours duquel se forme l’identité.

Erikson étudiait le développement l’entendons, mais nous avons encore besoin de


de l’identité, de a prime enfance à règles et de protection parentales.
l’âge adulte (il est l’inventeur de La période adolescente (12–18ans) est
l’expression «crise d’identité»). Dans les importante pour établir a conscience de soi.
années1950, il a proposé huit stades Adolescents, nous expérimentons beaucoup,
prédéterminés du développement et huit nous passons tous par de drôles de phases!
vertus de base qu’on pourra acquérir à chacun L’impulsion vient de nos pairs, nous voulons
des stades: espoir, volonté, détermination, être acceptés, mais nous sentons aussi une
compétence, fidélité, amour, sollicitude pression qui nous pousse à choisir un chemin.
et sagesse. Le succès à ce stade entraîne a fidélité envers
Erikson parait des conflits psychosociaux, nos propres principes, et a conscience de soi.
entre nos besoins psychologiques et notre Toutefois, certains demeurent peu sûrs d’eux-
environnement social, que nous traversons, et mêmes ou de leur pace dans a société. C’est
de leur impact sur le développement de notre a confusion des rôles.
personnalité. La résolution de ces conflits Si nous avons développé une conscience
peut mener au développement de certaines de soi forte, nous nous demanderons ce
vertus qui nous guide par a suite, tandis que que nous signifions pour les autres lors de
leur non-résolution entraîne une vulnérabilité. a prochaine étape (de 18 à 40ans). Pouvons-
Par exemple, ne pas développer a confiance nous tisser des reations intimes ou resterons-
et donc non plus l’espoir avant 2ans (en nous isolés? Plus tard (40–65ans), notre
raison de mauvais soins) risque d’entraîner rôle s’inverse, nous élevons des enfants qui
des problèmes amoureux autour de 25ans. continueront après nous. Le succès est alors
Certains conflits sont néanmoins nécessaires, de se sentir accompli, et de se mettre à aider
il s’agit de se forger un caractère! Lors du les autres. Enfin, après 65ans, nous cherchons
stade suivant, entre 2 et 4ans (développement à atteindre l’équilibre entre l’intégrité du soi
de a volonté), nous voulons de a liberté, par (acceptation) et le désespoir, ce qui, avec un
exemple celle de nous vêtir comme nous peu de chance, nous mènera à a sagesse.

104 PSYCHOLOGIE DU DÉVELOPPEMENT


VERTUS D’ERIKSON
Selon votre âge, quel rôle avez-vous? Un
enfant est en coulisses, il enfile son costume
et rentre dans son personnage (espoir,
volonté, détermination, entre 0 et 5ans). Un
adolescent est sur scène, il répète pour
construire sa confiance (compétence,
fidélité, entre 5 et 18ans). Un jeune est dans
les travées, il discute avec les autres (amour,
entre 18 et 40 ans). En vieillissant, nous
passons sur les côtés de a scène, à soutenir
les autres (sollicitude, entre 40 et 65ans),
puis nous nous asseyons pour réfléchir aux
rôles tenus auparavant (sagesse, après
65ans). C’est toutefois notre environnement
qui joue le plus grand rôle dans le
développement de ces vertus.

Dans les travées, Dans le public, réfléchissant


discutant avec aux rôles joués par le passé
les autres (amour, (sagesse, à 65ans et plus).
18–40ans).

Sur scène, en répétition


pour construire la
confiance (compétence,
fidélité, 5–18ans).
En coulisses pour
rentrer dans
le personnage
(espoir, volonté,
détermination, Caché sur le côté,
0–5ans). pour aider les autres
(sollicitude, 40–65ans).
CHAPITRE 6 DIFFÉRENCES INDIVIDUELLES

EFFET
QI FLYNN

PYRAMIDE
DES BESOINS
DE MASLOW
PERSONNALITÉ

INTELLIGENCE
ÉMOTIONNELLE

MOTIVATION
CHAPITRE 6

IDIIDUELLES
DIFFÉRECES
N –ACH

ANALYSE DES
FACTEURS
INTRODUCTION

C
ertains paraissent savoir tout faire. Quand nous
entendons parler de leurs exploits, nous nous
disons qu’ils doivent être plus volontaires ou plus
intelligents que nous, à moins qu’ils ne se soient
taillé une personnalité plus forte. Ce chapitre s’intéresse à
nos différences individuelles dans le cadre des activités qui font
notre succès ou notre bonheur.
Ce qui pousse les gens dans a vie est avant tout leurs
besoins élémentaires, remplir leur estomac, se protéger
du froid. Une fois ces besoins assurés, chacun ira chercher
ses motivations dans ce qui compte le plus pour elle ou
lui. Beaucoup s’attachent à des besoins pourvus d’une
signification profonde, tels que le lien avec autrui ; d’autres
courront après le succès ou l’expérience du pouvoir. Maslow
nous enseigne que a source de motivation a plus élevée est
a pulsion de centrage spirituel, un état d’esprit qui sépare les
bons des meilleurs et que bien peu atteignent.
Comparées au passé, les demandes de nos cerveaux
sont variées et complexes, ce qui explique en partie que
nous devenons de plus en plus intelligents. C’est l ’EFFET
FLYNN, qui désigne les gains d’intelligence qu’on observe
par comparaison entre deux périodes historiques. Ainsi,
en moyenne, les hommes du XIXe siècle lutteraient
pour obtenir les mêmes résultats que nous lors de tests
d’intelligence.
Cependant, a psychologie a renoncé à prendre les
performances sur un genre très spécifique de tests comme
étalon d’intelligence. Selon les psychologues, étant donné

108 DIFFÉRENCES INDIVIDUELLES


a multiplicité de talents constatés dans a popuation,
plusieurs types d’intelligence (jusqu’à huit) doivent être
pris en compte: linguistique, spatiale, et existentielle, entre
autres. Ces intelligences multiples sont le fruit de l’éducation
et de l’expérience, tandis que l’intelligence analytique que
mesurent les tests de QI est considérée comme innée.
Récemment, l’INTELLIGENCE ÉMOTIONNELLE (IE)
s’est imposée pour expliquer le succès de certaines
personnes. Quelqu’un pourvu d’une haute IE comprend ce
que ressentent les autres et adapte son comportement en
conséquence. Les leaders efficaces ont souvent une bonne
IE. Ils sont aussi dotés de fortes personnalités, une distinction
centrale entre les individus et pourtant très difficile à
mesurer. Les psychologues ont fini par réduire a mesure de
a personnalité en cinq traits centraux, les BIG FIVE. Chaque
trait est associé à un spectre: si par exemple vous obtenez
un score élevé sur le trait d’ouverture, cea signifie que vous
devez être curieux et inventif, des qualités disposées à une
extrémité du spectre, tandis qu’il y a peu de chances que vous
soyez régulier et prudent, celles associées à l’autre extrémité.
Motivation, intelligence et personnalité ne sont pas
distribuées en parts égales, mais elles forment notre image
du monde. La théorie a plus complète pour expliquer les
différences individuelles d’interprétation et de construction
de a réalité est due à Kelly, c’est a théorie des constructs
individuels. Elle établit que nous créons des CONSTRUCTS,
des briques de construction, qui influencent nos pensées,
nos comportements et jusqu’à nos sentiments.

DIFFÉRENCES INDIVIDUELLES 109


DIFFÉRENCES INDIVIDUELLES

PENSÉE ABSTRAITE
Raisonnement logique et
EFFET FLYNN
résolution de problèmes;
Gains constatés en intelligence
parfois nommée
(QI moyen) avec le temps,
«intelligence fluide».
d’environ 3 points de QI par
décennie dans les sociétés
PYRAMIDE
industrialisées (Flynn).
DE MASLOW
Structure de nos motivations:
de la plus basique, la survie, tout
en bas, à la plus élevée,
la transcendance, tout en haut,
en passant par l’amour
et l’appartenance.

THÉORIE
TRIARCHIQUE
DE L’INTELLIGENCE
DE STERNBERG
Plutôt que le QI, trois catégories INTELLIGENCES MULTIPLES
d’intelligence sont plus adaptées DE GARDNER (IM)
au monde moderne: pratique, Les humains sont dotés de
créative et analytique. différents niveaux de toutes
les IM, définies par un jeu de
critères stricts et divisées en
neuf types.
INTELLIGENCE

INTELLIGENCE
ÉMOTIONNELLE (IE)
Consiste en cinq compétences
pour maximiser ses
performances ou celles d’autrui:
recul, modération, motivation,
empathie, sociabilité (Goleman).

DANIEL GOLEMAN
Psychologue américain (1946–)
SYSTÈME LIMBIQUE
connu pour ses travaux sur
Partie du cerveau dans le
l’intelligence émotionnelle,
lobe temporal qui contrôle
surtout dans le domaine du
les pulsions, les sentiments
management et des affaires.
et les motivations. Goleman
conseille aux formations
professionnelles de se
concentrer sur sa stimulation.
BESOINS

INTERPRÉTATION
ALTERNATIVISME CONSTRUCTIF
Constructs alternatifs pour nous aider
en certaines situations; comprendre
les constructs d’autrui pour expliquer
leur comportement (Kelly).

ACCOMPLISSEMENT
Au sommet de la pyramide de CONSTRUCTS
Maslow: s’accomplir au mieux Schémas mentaux individuels
de ses capacités. Une personne qui nous aident à prévoir,
accomplie cherche toujours souvent fondés sur l’expérience
à s’épanouir encore. passée (Kelly). Ils influencent
aussi nos souvenirs.

LE ‘N - ACH’ DE MCCLELLAND
«Need for achievement» ou
«Besoin de succès»: trait présent
chez ceux pourvus du désir inné
d’accomplir de grandes choses. PÔLES ÉMERGENT/IMPLICITE
Les constructs naviguent entre
deux pôles: le pôle émergent,
d’où ils sont appliqués, et le
pôle implicite, qui n’est pas
activement appliqué.

BIG FIVE DE
MCCRAE ET COSTA
(OCEAN)
Cinq traits qui, en quantité
différente, peuvent décrire les
personnalités et caractériser
tous les adultes: Ouverte,
Consciencieuse, Extravertie, ANALYSE FACTORIELLE
Agréable, Névrotique. Technique d’analyse statistique qui
identifie les motifs cachés et réduit
de gros ensembles de données en un
petit nombre de facteurs sous-jacents.

PERSONNALITÉ
Pe-on ire de pin
e d’e rîche?
Physiologiquement, oui, mais nos
besoins émotionnels ne seront pas remplis.
Selon Maslow, nous sommes tous poussés
par un besoin d’épanouissement et de
découverte de soi. Une fois les besoins
biologiques assurés, nous passons à
d’autres, plus ambitieux.

Qu’est-ce qui nous pousse? Maslow personnalité. David McCleland a ainsi inventé
a proposé une organisation le N-Ach («Need for ACHievement», besoin
pyramidale de nos besoins, avec les de succès) pour décrire une personnalité
besoins basiques de a survie, eau, nourriture, abritant le désir inné d’accomplir de
chaleur, tout en bas. Une fois ceux-ci assurés, grandes choses. Les personnes N-Ach sont
ce qui nous pousse devient plus compliqué, incapables de vivre de pain et d’eau fraîche,
par exemple le besoin d’intégrer un groupe il leur faut des défis et de l’indépendance,
(amour et appartenance), jusqu’à des besoins et a récompense a plus précieuse à leurs
transcendants, au-delà de soi, comme ceux yeux: de a reconnaissance. Il faut qu’on
de a foi religieuse (transcendance). reconnaisse leurs succès. De l’autre côté de a
Ces besoins sont décrits selon une barrière, les personnes dépourvues de N-Ach
hiérarchie où les besoins de base doivent être ne cherchent pas le succès et pourront
remplis avant de passer à a suite, mais on choisir de s’investir dans des tâches faciles
estime que certains «sautent» les niveaux en afin de réduire le risque d’échec, ou au
fonction de leurs différences individuelles ou contraire si difficiles que l’échec n’y est
de leur environnement. Atteindre le sommet plus très embarrassant. McCleland
ne signifie pas que plus rien ne nous pousse. montrait à ces sujets d’expérience des
Une personne accomplie cherche toujours à images de situations sociales ambiguës
s’épanouir encore, à faire tout ce qu’elle peut et leur demandait de décrire le
pour cea. Toutefois, Maslow estime que seuls scénario constaté, afin de déterminer
2% des gens atteignent ce stade, l’un d’eux quel rôle jouait le N-Ach chez chacun.
ayant été Albert Einstein! Il existe aussi une cassification en
D’autres psychologues pensent que les fonction du besoin d’affiliation (N-Aff)
motivations de chacun sont fonction de sa et du besoin de pouvoir (N-Pow).

112 DIFFÉRENCES INDIVIDUELLES


N -ACH

Schultheiss et ses collègues ont découvert


que des niveaux élevés de N-Ach (besoin de
succès) étaient des prédicteurs de réponse
faible au cortisol, l’ hormone relâchée en cas
de stress, par exemple lors de compétitions
ou de tâches ardues, comme gravir une
montagne. Ils ont émis l’ hypothèse que les
personnes au N-Ach élevé avaient appris à
associer les tâches difficiles au paisir de les
surmonter, ce qui impliquerait une réponse
au stress plus faible que chez ceux qui n’ont
pas appris une telle association.
Sommes-nos ps
ineiens q’n?
Selon les tests de QI, oui. Mais ces
gains ne sont pas réalisés grâce à
l’accroissement des connaissances; ils sont
dus à l’amélioration de a pensée abstraite.

James Flynn a été le premier à moyen en 1910 vaudrait à cette personne


étudier a hausse du QI dans les d’être cassée aujourd’hui dans les derniers
années 1980, si bien que échelons.
l’accroissement de l’intelligence humaine Selon Robert Sternberg, le QI n’a qu’une
avec le temps est aujourd’hui appelée l’effet importance très reative dans le monde
Flynn. Au niveau de a popuation entière, nous d’aujourd’hui. (Ne vous en faites donc pas si
avons progressé dans tous les domaines, mais vous n’êtes pas cassés parmi les surdoués.)
les plus gros gains ont été effectués dans a Dans a théorie de Sternberg, a théorie
pensée abstraite, qui désigne les capacités de triarchique de l’intelligence, celle-ci se
compréhension logiques et de résolution des divise en trois catégories. La première est
problèmes. On l’appelle aussi parfois l’intelligence pratique, qui se préoccupe du
«l’intelligence fluide». contexte; c’est elle qui nous permet de nous
Il se pourrait que l’amélioration soit due adapter aux nouveaux environnements et de
à a généralisation de l’accès à l’éducation, à les modeler à notre convenance. Vient ensuite
l’amélioration de a santé et de a nutrition, ou l’intelligence créative, qui donne souplesse
à une exposition argement supérieure à ce et créativité, surtout utile pour appréhender
genre de tests. Par exemple, a popuarité des les tâches nouvelles. La dernière, selon a
jeux vidéo favorise les tâches de raisonnement théorie, est a seule qui soit mesurable lors
spatial. Manipuler et dépacer des objets des tests de QI traditionnels: l’intelligence
dans un monde virtuel est un excellent analytique, celle qui nous permet d’acquérir
entraînement aux tests de QI, où certaines des informations nouvelles, de comprendre et
questions demandent de réaliser des rotations résoudre les problèmes.
mentales de formes déterminées. De ce fait, Ainsi, afin de savoir si les humains gagnent
une personne aujourd’hui dans a moyenne de vraiment en intelligence ou s’ils s’améliorent
ces tests obtiendrait des scores dans les 2% simplement aux tests, il nous faudrait peut-
les plus élevés en 1910, tandis qu’un résultat être changer nos techniques de mesure.

114 DIFFÉRENCES INDIVIDUELLES


ÉCHELLE DE L’INTELLIGENCE CRÉATIVE

L’ intelligence créative se divise elle-même en répéter une tâche jusqu’ à l’automatiser,


plusieurs catégories, selon Sternberg. Certains l’accomplir sans plus y penser, au besoin en
sont doués pour a nouveauté, pour trouver de même temps que d’autres. Être doué pour une
nouvelles façons de résoudre les problèmes, tâche précise ne signifie pas que vous le serez
de nouvelles voies. D’autres parviennent à pour une autre, toutefois!
Combien  --i
d’ineiences?
La définition traditionnelle de
l’intelligence aisse entendre qu’il s’agirait
d’une capacité d’analyse uniforme.
Les psychologues, dont Howard Gardner,
proposent de leur côté de distinguer huit
types d’intelligence.

En 1983, Gardner a avancé que les a synchronisation et à a capacité d’améliorer


humains ne naissaient pas avec une ses réflexes, très utile si vous voulez devenir
seule quantité d’intelligence définie un athlète. Si vous parvenez toujours à savoir
qu’ils conserveraient toute leur vie. Selon lui, comment les autres se sentent, ou à savoir
il existe des intelligences multiples (IM), que quoi dire pour les calmer ou les réjouir, vous
chacun possède à des degrés divers. Ces avez sûrement beaucoup d’intelligence
types d’intelligence sont définis par un jeu de interpersonnelle: a capacité d’identifier et de
critères fixes. Elles étaient six au début, mais répondre aux humeurs. Dans a même veine,
on en compte aujourd’hui neuf. votre intelligence intrapersonnelle vous permet
La première des IM est linguistique: a d’identifier vos propres sentiments et de vous en
capacité d’apprendre et employer le angage, servir pour guider votre comportement.
celle des gens de plume, des journalistes Savoir reconnaître et cassifier de
et des juristes. La deuxième est logico- nombreuses espèces dans un environnement
mathématique: a sensibilité aux motifs donné et mettre cette capacité à profit est
numériques et logiques qui donne accès aux l’intelligence naturaliste, comme chez Charles
raisonnements. La troisième est musicale: si Darwin ou Jane Goodall. Enfin, l’intelligence
vous avez a capacité de produire et d’évaluer existentielle est a capacité de penser et de
un rythme, une bonne oreille, voire l’oreille méditer sur les questions les plus profondes,
absolue, vous en êtes richement doté. La telles que le sens de a vie.
capacité de visualiser les choses par a Même si Gardner estimait que notre
pensée, de jauger des distances et manipuler méange unique d’intelligences était influencé
mentalement les objets est l’intelligence par a génétique, les IM doivent être cultivées
spatiale, celle des pilotes et des chirurgiens. et renforcées par l’expérience vécue et
L’intelligence kinesthésique s’intéresse au l’éducation. Les ignorer ou ne pas s’en servir
contrôle du corps et à a motricité, ainsi qu’à régulièrement les affaiblirait.

116 DIFFÉRENCES INDIVIDUELLES


INTELLIGENCES MULTIPLES

Interpersonnelle

Kinesthésique Linguistique

Logique

Musicale

Naturaliste
Intrapersonnelle

Spatiale Existentielle

Gardner pensait chacun pourvu de tous les d’ intelligence leurs élèves privilégient.
types d’intelligence à différents niveaux. Il Adapter son enseignement selon cette
estimait qu’elle se localisait en différentes information facilitera leur apprentissage,
régions cérébrales, mais cea n’a jamais le système éducatif traditionnel ayant
été prouvé. Il peut être toutefois utile tendance à ne se concentrer que sur les
aux professeurs de reconnaître quel type intelligences linguistique et logique.
Un bon che pe-i
monrer ses émoions?
On ne s’y attendait pas, et pourtant,
en1998, les recherches de Daniel Goleman ont
montré que les meilleurs leaders étaient ceux
dotés d’une grande intelligence émotionnelle,
autrement dit capables de comprendre leurs
émotions et celles des autres.

Une entreprise qui avait mis en pace l’empathie, facile à détecter chez les chefs
les découvertes de Goleman en ne capables. Encadrer une équipe revient
sélectionnant que des cadres dotés souvent à méanger un chaudron bouillonnant
d’une haute intelligence émotionnelle a vu son d’émotions. Il faut comprendre les émotions
chiffre d’affaires annuel augmenter de 20%. de tout le monde et les traiter en fonction.
Qu’est-ce que l’intelligence émotionnelle (IE)? Enfin, être capable d’entretenir des reations
L’IE est composée de cinq capacités qui et de construire un réseau, autrement dit a
permettent de maximiser ses performances et sociabilité, est une composante clé. Goleman
celles des autres. a décrit comme «l’amabilité orientée». C’est
D’abord, le recul sur soi, qui permet de le point culminant des autres capacités.
comprendre pleinement ses propres émotions La bonne nouvelle, c’est que, même
et pulsions, et de déterminer leurs effets si les recherches pointent vers une forte
sur autrui. Cea peut se traduire par une soif composante génétique à l’IE, elle s’apprendrait
de critiques constructives ou un goût pour aussi. Goleman conseille aux entreprises de se
l’auto-dérision. Ensuite, a modération, qui concentrer sur des formations qui stimulent
désigne le contrôle et a canalisation des le système limbique, a partie du cerveau
pulsions, ainsi que a suspension du jugement. qui contrôle les pulsions et les émotions.
Goleman a découvert que cea produisait Actuellement, a plupart des formations de
une atmosphère de confiance et d’équité. Un cadres parlent plutôt au néocortex, impliqué
chef au tempérament colérique ne fait pas un dans les capacités analytiques et techniques.
bon leader. Vient ensuite a motivation: un Même sans entraînement, l’intelligence
bon chef doit avoir a passion de son emploi, émotionnelle s’améliore avec l’âge: plus nous
au-delà du saaire ou du prestige qu’il lui vieillissons, plus nous gagnons en maturité
procure. Il tiendra souvent un tableau de ses émotionnelle. Bien sûr, certains chefs auront
succès, une démarche contagieuse qui pourra toujours besoin d’un long entraînement, quel
se propager dans le bureau. Quatrièmement, que soit leur âge.

118 DIFFÉRENCES INDIVIDUELLES


QI CONTRE INTELLIGENCE
ÉMOTIONNELLE EN MANAGEMENT

QI et capacités techniques

Intelligence
émotionnelle (deux
fois plus importante)

Quand Goleman et ses collègues ont voir les choses en grand. Mais l’intelligence
analysé les données de grandes entreprises, émotionnelle jouait un rôle deux fois plus
ils ont découvert que l’ intelligence des important que les capacités liées au QI ou
cadres était un bon prédicteur du succès. aux aspects pratiques. Et plus le saarié en
C’ était tout particulièrement vrai de talents question était haut pacé dans l’entreprise,
spécifiques, comme d’avoir une vision à plus l’intelligence émotionnelle jouait un
long terme du marché ou l’habitude de rôle important dans son succès.
Commen mesrer
 personnié?
Les réponses des psychologues varient.
Freud pensait que le secret de a personnalité
résidait dans l’inconscient, formé avec les
expériences de l’enfance. D’autres ont
employé des méthodes statistiques.

Afin de mesurer a personnalité, des amicales et compatissantes, les autres sont


psychologues ont effectué une plutôt critiques et rationnelles.
analyse factorielle sur toutes les N comme Névrotique: parfois nommé
données récoltées par de multiples instabilité émotionnelle, le névrotisme
questionnaires. Cette analyse consiste à désigne des personnalités sensibles et
chercher des réguarités cachées et ainsi nerveuses, opposées aux personnalités
réduire d’énormes ensembles de données en assurées et tenaces.
un petit nombre de facteurs sous-jacents. McCrae et Costa ont posé l’hypothèse que
Quand McCrae et Costa ont employé cette tous les adultes pouvaient se caractériser à
technique sur les données de personnalité, en l’aide de ces cinq traits. Chaque personnalité
1996, ils ont mis en évidence cinq de ces serait un méange différent de ces «Big Five»
facteurs émergents, capables de a décrire. et de leur manière d’influencer les modes
Chacun des traits est mesuré sur sa propre de pensée, les sentiments et les actions.
échelle et ils forment à eux tous l’acronyme Selon cette théorie, les traits de personnalité
OCEAN, ou CANOE: seraient donc des tendances fondamentales,
O comme Ouvert à l’expérience: les innées, qui ont leur origine en nous. Ils se
scores élevés dénotent une personnalité développent ensuite durant l’enfance et
inventive et ouverte, les scores faibles des deviennent stables et matures à l’âge adulte.
personnalités stables et prudentes. Mesurer a personnalité de cette manière
C comme Consciencieuse: les richement est utile pour prévenir certains ennuis de
dotés sont efficaces et organisés, les autres santé physique ou mentale. L’avantage de
plutôt extravagants et nonchaants. cette théorie est qu’elle est bien plus facile
E comme Extravertie: un score élevé à mesurer que celle de l’inconscient de
pour les gens faciles à repérer, énergiques et Freud. Par ailleurs, elle tient bien compte
spontanés, un score faible pour les personnalités de nos sentiments, à a différence des
solitaires et réservées. théories qui tiennent a personnalité pour
A comme Agréable: comme on s’y attend, une pure interaction entre un individu et son
les scores élevés dénotent les personnalités environnement particulier.

120 DIFFÉRENCES INDIVIDUELLES


MESURE DE PERSONNALITÉ
DANS LA STRUCTURE CÉRÉBRALE

NEUROTICISME

OUVERTURE

Épaisseur corticale Plis corticaux

Nous pourrions mesurer les traits de le cortex (a couche externe du cerveau)
personnalité en observant nos cerveaux. et à moins de plis, tandis que l’ouverture
Des chercheurs menés par Richelli se sont montrait l’opposé: un cortex plus fin et plus
servis de scanners pour découvrir que de replis. Ces différences dans a structure
de hauts niveaux de névrotisme étaient cérébrale indiqueraient que les traits de
associés à des parties plus épaisses dans personnalité ont une part génétique.
Consrisons-nos os
nore propre monde?
Oui, en un sens. Dans les années1950, le
psychologue George Kelly a avancé que
chacun de nous développe ses propres
constructions qui nous servent à interpréter
le monde. Tels des scientifiques, nous testons
en permanence nos propres hypothèses.

En toute situation, des «constructs» ambigus, aux multiples interprétations


sous-jacents, des schémas mentaux possibles, peut-être pour nous pacer sous le
en quelque sorte, nous aident à meilleur jour possible.
prévoir un résultat. Ils se fondent en général Ces constructs peuvent être vus comme
sur nos expériences passées. Par exemple, binaires, ou sur un spectre entre deux pôles.
vous avez peut-être un construct lié aux Le plus proche est alors le pôle «émergent»
salutations. Vous tendez a main et prédisez et l’autre le pôle «implicite». Le continuum
qu’on vous a saisira pour a serrer, ce qui se aventure-sécurité est par exemple un de ces
fonde sur ce que vous avez vu et vécu par le spectres. Si vous étiez pourvu d’un construct
passé. Si l’interaction se passe comme prévu, ayant l ’aventure comme pôle émergent, vous
c’est que le construct est utile et vous le a verriez comme désirable et excitante et
réutiliserez. Sinon, si par exemple a personne seriez bien plus enclins à participer à des
vous fait a bise, il vous faudra le modifier activités à haut risque que quelqu’un qui
ou l’abandonner. pacerait a sécurité au pinacle et aurait donc
Comme nous avons tous des constructs l’aventure en pôle implicite.
différents issus de nos expériences vécues, Les constructs ne sont pas figés. Kelly a
nous voyons tous le monde à notre façon, précisé qu’il y a avait toujours des constructs
grâce à ces prévisions. Cea pèse aussi sur alternatifs qui peuvent nous aider à tout
nos souvenirs, ce qui explique comment moment. Il appeait ça l’alternativisme
deux personnes peuvent se souvenir très constructif. Nous voyons tout le monde par
différemment de a même scène. Selon Kelly, notre lorgnette, mais il est important de
nous choisissons les constructs à appliquer comprendre les constructs des autres, cea
quand nous réfléchissons à des événements nous aide à expliquer leurs comportements.

122 DIFFÉRENCES INDIVIDUELLES


CYCLE DE L’EXPÉRIENCE

2
Événement

3
Réflexion

4
1
Planification
Intégration

5
Anticipation

Selon Kelly, les principaux moteurs de l’esprit


humain sont l’anticipation et a prévision. À
chaque fois que nous visons un événement,
nous anticipons le construct qui s’appliquera
et nous évaluons sa pertinence. Après coup,
quand nous réfléchirons au résultat, nous
pourrons apporter des modifications au
construct et les intégrer à notre anticipation de
a prochaine occurrence de l’ événement. Cette
récurrence est fondamentale, car les constructs
naissent de a répétition.
CHAPITRE 7 THÉRAPIE

STYLE
ATTRIBUTIONNEL

RAISONNEMENT
ÉMOTIONNEL

COMPORTEMENTS
DE SÉCURITÉ

TRE
TCC
CHAPITRE 7
TRAUMA

ÉCOUTE
THÉRPIE
RÉFLECTIVE

STRATÉGIE
D’ADAPTATION
INTRODUCTION

N
ous savons tous que cea fait du bien de parler
de nos problèmes. C’est ainsi qu’amis, coiffeurs
et chauffeurs de taxi sont transformés en
thérapeutes, parce qu’on se sent bien après
leur avoir parlé. Mais une thérapie véritable nécessite un
peu plus que d’écouter et de poser quelques questions.
La thérapie sert à traiter des troubles tels que l’anxiété
ou a dépression, les traumas et autres problèmes plus
graves. Les plus anciennes thérapies connues remontent
à plus de 3500ans: des textes grecs et égyptiens parlent
de «soigner avec des mots». Le philosophe grec Épictète
a remarqué que quand les temps étaient durs, ce qui
comptait avant tout était le sens qu’on donnait aux
problèmes rencontrés. Cette idée que a pensée oriente les
sentiments est a pierre anguaire de a thérapie considérée
aujourd’hui comme a plus efficace: a THÉRAPIE COGNITIVE
COMPORTEMENTALE (TCC). Il fallut néanmoins plus de mille
ans pour que des études confirment le rôle que jouent nos
pensées dans les mécanismes d’anxiété et de dépression.
Avant cea, une autre forme de thérapie, a psychanalyse,
était popuaire. Ce traitement aidait les patients à écairer
des aspects de leur personnalité et même de leurs rêves.
Cependant, aussi fascinantes ces découvertes soient-
elles, elles entraînent rarement un changement. Cette
thérapie n’a jamais été appliquée avec succès pour traiter
des troubles mentaux graves. Le domaine a ainsi stagné
jusqu’au milieu du XXe siècle et à l’apparition soudaine de
nouveaux développements et de a THÉAPIE HUMANISTE
de Carl Rogers, une approche chaleureuse fondée sur le
pouvoir guérisseur de a gentillesse.

126 THÉRAPIE
À peu près au même moment, le béhaviorisme
a gagné en influence. Il s’agissait là d’examiner les
comportements animaux pour mieux comprendre
comment nous apprenons et désapprenons a peur. Wolpe
a ainsi développé a désensibilisation systématique, une
forme de thérapie qui aidait les patients à confronter,
plutôt qu’éviter, leurs sources d’angoisse. Cependant,
cette approche ne modifiait pas toujours les modes de
pensée. On pouvait ainsi continuer de juger les araignées
dangereuses tout en apprenant à vivre en leur présence.
La problématique de gestion du stress a pris de
l’ampleur dans les années1960 avec Lazarus, qui jugeait
que le stress était le mariage entre les pensées reatives
à une situation difficile et a capacité de s’y adapter. Les
thérapies ont donc évolué pour renforcer l’adaptation,
grâce à des outils comme a reaxation. Mais que peut a
reaxation contre a peur ou une tristesse persistante? Ellis
a ainsi créé a THÉRAPIE RATIONNELLE ÉMOTIVE (TRE), qui
se concentre sur le lien entre pensées et émotions.
Beck a renforcé cette idée en abordant a question
de a santé mentale dans une démarche scientifique.
Il a développé des modèles psychologiques solides et
rigoureusement évalués pour expliquer ce qui cause et
maintient a détresse. Il a ensuite ciblé les pensées et les
comportements problématiques de façon scientifique et
logique, proposant aux patients de tester leurs pensées
au cours d’expériences et de les modifier grâce aux
résultats de celles-ci. Aujourd’hui, a thérapie cognitive
comportementale est a plus étudiée à travers le monde,
celle dont les preuves d’efficacité sont le plus solides.

THÉRAPIE 127
THÉRAPIE

CONGRUENCE THÉRAPIE HUMANISTE


Atteindre l’idéal de soi en Approche thérapeutique de
développant son estime de soi Rogers fondée sur la gentillesse
au travers de l’amour et de la et la foi en la part de bien
tolérance (Rogers). de chacun, et en sa volonté
d’atteindre son plein potentiel.

APPROCHE CENTRÉE SUR


LA PERSONNE (ACP)
Approche thérapeutique de
Rogers fondée sur des principes
humanistes et la fourniture
d’un espace non directif, sans THÉRAPIE
jugement, dans lequel le patient COGNITIVE
trouve le pouvoir d’aboutir à COMPORTEMENTALE
ses propres remèdes. (TCC)
Petits changements dans les
pensées et les comportements
quotidiens grâce auxquels obtenir
des améliorations significatives
et durables de nos
émotions (Beck).
TRAITEMENT

THÉRAPIE RATIONNELLE
ÉMOTIVE (TRE)
Développée par Albert Ellis, approche
dans laquelle patient et thérapeute
identifient les croyances irrationnelles
et les pensées nuisibles afin de les
remplacer par des stratégies utiles.

DISTORTION COGNITIVE
Schéma de pensée irrationnel
ou exagéré, par exemple tout-
ou-rien ou trop pessimiste.
On peut y remédier par
la thérapie, surtout la TCP.

RAISONNEMENT
ÉMOTIONNEL
Fonder nos conclusions sur ce
qu’elles nous font ressentir –
schéma de pensée nuisible, car ne
reflétant pas toujours la réalité.
COMPORTEMENTS
RICHARD LAZARUS ET
EXPÉRIENCE SUSAN FOLKMAN
Psychologues américains
COMPORTEMENTALE
(1922–2002 & 1938–) ayant
Mise en place de changements
mis en avant la notion
de comportement modestes
du stress comme un
et pragmatiques, au cours
déséquilibre entre la
d’une TCC, pour surmonter
demande et les ressources.
les difficultés, par exemple en
diminuant l’évitement.
STRATÉGIES
D’ADAPTATION
Mécanismes qu’un sujet met
en place pour gérer son stress. SEAU DU STRESS
Les stratégies fondées sur le Analogie qui consiste à voir
problème cherchent à supprimer les expériences stressantes
celui-ci, celles fondées sur les comme de l’eau remplissant
émotions à orienter celles un seau (notre capacité
que le stress génère. de stress). Les stratégies
d’adaptation ouvrent un
MOTIVATION MAGIQUE robinet pour vider le seau
DE KEARNS ET GARDINER (Brabban et Turkington).
Espoir que la motivation naîtra
comme par magie pour nous
pousser à l’action, alors que PENSÉES NÉGATIVES
c’est se lancer dans une tâche INTRUSIVES
qui génère l’envie. Pensées nuisibles intempestives
qui viennent à l’esprit. Ceux qui
s’en trouvent gênés ou préoccupés
peuvent développer des troubles
de l‘anxiété.

CONSCIENCE
DES MÉCANISMES
DE PENSÉE
Prendre conscience de
ses mécanismes de pensée,
par exemple grâce à la thérapie,
permet d’adopter une vision
d’ensemble et de se montrer
plus juste envers STYLE ATTRIBUTIONNEL
soi-même. Approche que nous prenons
d’ordinaire pour assigner une cause
à un malheur (Seligman). Ceux
prompts à s’attribuer le blâme sont
davantage sujets à la dépression.

PENSÉES
Commen renconrer
son moi idé?
Peut-être est-ce déjà fait! Ou vous
vous sentez en accord avec votre moi idéal
a plupart du temps. Mais pour d’autres, le
moi idéal semble très loin et l’atteindre
exige de travailler sur ses reations et ses
sentiments pour construire l’estime de soi.

Le psychologue Carl Rogers parait chaleureux et compréhensif, il accepte le


d’incongruence pour décrire un état patient tel qu’il est. Il l’aide à comprendre ses
de dissociation franche entre le soi pensées et ses émotions du mieux possible,
et le soi idéal. Il pensait que nous avons tous afin de lui donner l’opportunité de parler
besoin d’être estimés, respectés et aimés, de ses problèmes et d’aboutir à ses propres
d’être toujours considérés de façon positive. conclusions sur les changements à adopter.
C’est ce qui nous permet de développer L’idée d’un soi idéal parfaitement
l’estime de soi et de nous amener à a fonctionnel n’est peut-être que ça, une
congruence, de nous rapprocher du soi idéal. idée plutôt qu’un but réel. Il a aussi été
L’approche de Rogers se fondait sur les dit que ce concept n’était peut-être pas
idées humanistes: il y a du bon en chacun aussi pertinent dans d’autres cultures,
de nous, nous cherchons tous à atteindre aux valeurs très différentes. Toutefois,
notre plein potentiel afin de nous épanouir. a plupart des thérapies pratiquées de
Ce fut a fondation pour un nouveau type nos jours en psychologie tirent encore
de thérapies désignées comme «centrées parti des travaux de Rogers. Toutes ont
sur a personne» (TCP). Un élément clé de intégré l’idée d’un thérapeute portant un
ces thérapies est d’être non directives: regard inconditionnellement positif sur le
le thérapeute fournit un espace ouvert et patient et travailant à établir une reation
bienveilant, mais n’oriente pas a discussion. thérapeutique forte. Cette reation, parfois
Chacun étant le meilleur expert sur lui- appelée «l’alliance thérapeutique», est l ’une
même, le patient est le mieux pacé pour des composantes aujourd’hui jugées comme
trouver les solutions qui lui permettront essentielles pour une thérapie aboutissant
d’atteindre ses objectifs. Le thérapeute reste à des résultats positifs.

130 THÉRAPIE
ÉCOUTE RÉFLECTIVE

Vous a-t-on déjà vraiment écouté? Et vous,


avez-vous déjà vraiment écouté quelqu’un?
Écouter, ce n’est pas attendre son tour
pour parler. L’ écoute réflective est une
stratégie fondée sur les travaux de Rogers,
qui se déroule en deux étapes: 1) essayer de
véritablement comprendre le point de vue
de l’autre et 2) le lui renvoyer afin de vérifier
que vous l’avez correctement compris. On
l’utilise très souvent en thérapie pour assurer
a compréhension mutuelle et une reation
thérapeutique forte.
Commen on s’dpe?
Tout dépend du seau! Quand le «seau
de stress» déborde, cea signifie que les
demandes, ce qu’on exige de nous, dépassent
nos ressources, nos capacités d’adaptation.
Il est alors temps de demander de l’aide…

Une vie entièrement dépourvue de structurée, une optimisation du temps ou


stress est malheureusement une aide extérieure. Les stratégies centrées
impossible. Le stress remplit une sur les émotions cherchent au contraire
fonction: en quantité modérée, il peut nous à gérer les émotions que le stress induit.
pousser à aller plus loin et améliorer nos Il s’agit par exemple de stratégies de
performances. Mais quand il y en a trop, il distraction, de méditation, de prière, ou
peut devenir nocif et provoquer toute une d’expression des émotions ressenties. Cea
gamme d’émotions négatives. inclut aussi des stratégies visant à nous
L’idée du stress comme d’un déséquilibre aider à court terme, mais susceptibles de
entre les demandes et les ressources a provoquer des effets graves à long terme, par
été avancée par Richard Lazarus et Susan exemple faire usage d’alcool ou de drogues
Folkman. Ils ont souligné que ces demandes, ou éviter complètement le problème.
ce qu’on exige de nous, pouvaient être L’adaptation centrée sur le problème est
externes, mais aussi seulement perçues souvent efficace parce qu’elle s’attaque aux
comme telles, voire complètement internes. causes du stress. Mais tous les problèmes ne
Et si nous sentons que nos ressources s’y prêtent pas et un méange des deux types
personnelles et sociales sont insuffisantes de stratégie est souvent le plus efficace, du
pour remplir ces demandes, le stress naît. moment qu’on évite les plus négatives.
Lazarus et Folkman estimaient que a nature Il est raisonnable de dire que, a plupart
de a situation et le caractère de a personne du temps, nous parvenons à nous adapter
impliquée affectaient son interprétation et au stress que nous accumulons. Mais il est
donc a réaction d’adaptation. tout aussi normal ou fréquent de traverser
L’adaptation est un mécanisme de des périodes ou ne parvenons plus à nous
gestion du stress. Les stratégies d’adaptation adapter. Quand cea se produit, le mieux à
se divisent en deux types. Les stratégies faire est de parler à quelqu’un, un ami, un
centrées sur le problème cherchent à membre de a famille, un professionnel de
supprimer le problème, par exemple, santé. Il n’est jamais obligatoire de traverser
à mettre en application une résolution seul ces mauvais moments.

132 THÉRAPIE
SEAU DE STRESS Le «seau de stress» de Brabban et
Turkington est un moyen simple de visualiser
notre capacité de gestion du stress afin
d’ éviter un «débordement» de sentiments
nuisibles à notre bien-être. Les expériences
stressantes sont l’eau qui remplit le seau.
L’adaptation et les stratégies de délestage
du stress ouvrent le robinet qui libère l’eau.
En surveilant le niveau d’eau et en gardant
le robinet ouvert, nous pouvons maintenir
notre stress sous contrôle.

Ennu
is de

eil
Prob

sant

somm

s
el
lèm

nn
tio
ue de
es fi

l
ai
la

av
re
nan

tr
So
Manq

es

au
uc

m
cier

ss
i

re
ob

St
s

Pr

n Réso
io lutio
xat n de
la s pr
Re Div
oblè
mes
ert
iss Soin
em s
en
t
s
he
ps

c
ro
m

sp
te

e
du

d
ien
n

t
tio

u
So
s
Ge
D’o iennen es
pensées néies?
Certaines nous viennent parce nous
sommes stressés, tristes ou fatigués. Pour
d’autres, ce n’est que notre cerveau qui prend
un raccourci en s’appuyant sur nos
expériences passées. Certaines sont
aléatoires. Aucune n’est nécessairement vraie
pour autant.

Il n’y a probablement pas de cause appelle le style attributionnel, une approche


unique de nos pensées négatives. que nous adoptons souvent lorsque nous
Certaines sont intempestives, attribuons une cause à un malheur. Certains
aléatoires. Des études ont montré que ce type ont un style qui consiste à supposer, face à
de pensées négatives intrusives est très un événement stressant ou négatif, qu’ils ont
commun. Elles concernent souvent des sujets commis une erreur, qu’il y a peu de chance
que nous tenons pour gênants, basphématoires que cea change et que cea risque d’affecter
ou blessants. Beaucoup parviennent à les de nombreux domaines de leur vie. Ce style
ignorer, leur dénier du sens, mais pour d’autres, est soupçonné d’augmenter les risques de
ces pensées sont préoccupantes, ce qui a été dépression, car il nous pousse à nous sentir
associé à des troubles anxieux tels que les démunis, comme si nous n’avions aucun
troubles obsessionnels compulsifs. contrôle sur a situation.
Se sentir triste ou anxieux est suffisant Ce qu’il est important de savoir,
pour générer davantage de ces pensées c’est qu’une pensée négative n’est pas
négatives. Nous ruminons des erreurs nécessairement vraie. Les pensées ne sont
passées, nous nous jugeons durement ou pas des faits. Par ailleurs, nos styles de pensée
nous nous inquiétons de l’avenir. Martin ne sont pas gravés dans le marbre. Devenir
Seligman a avancé l’idée selon aquelle nos conscient de nos mécanismes de pensée
esprits sont formés d’une certaine manière permet de savoir quand nous manquons
par nos expériences précoces. Ceux qui de recul ou quand nous nous jugeons
ont connu une enfance difficile s’attendent trop durement, et ainsi d’adopter un angle
souvent à ce que les choses tournent mal, différent. C’est une idée centrale dans de
et que ce sera de leur faute. C’est ce qu’on nombreuses psychothérapies.

134 THÉRAPIE
RÉPRESSION DE PENSÉES
Jetez un œil à ce gros apin rouge et duveteux.
Maintenant, fermez les yeux 30secondes et
ne pensez surtout pas à ce apin. Essayez du
mieux que vous pouvez de ne pas y penser. Les
études montrent que plus on essaye d’ éviter
de penser à quelque chose, plus cette chose
sera susceptible de nous venir à l’esprit. Tenter
de réprimer les pensées négatives pourrait
augmenter leur fréquence, en réalité. Bien des
patients en psychothérapie font bon usage de
cette découverte.
Porqoi sommes-nos
irrionnes?
Sûrement parce que nous ne sommes
pas des robots. Nous n’agissons pas purement
rationnellement parce que nos émotions
guident nos décisions et
nos comportements.

Il est souvent utile de se aisser irrationnelles et autres schémas de


guider par nos émotions, de «suivre pensée néfastes, afin que ces croyances
notre instinct» ou «d’écouter notre soient mises en question et rempacées
cœur». L’influence des émotions nous aide à par des stratégies plus utiles. Dans l’exemple
nous concentrer sur ce qui compte le plus, ci-dessus, un lieu fréquenté a entraîné un
tout en évitant les situations dangereuses ou sentiment de danger, mais il n’y avait aucun
dépaisantes. Mais, parfois, n’écouter que nos signe de danger en réalité. Cette conclusion
émotions peut nous entraîner dans des erronée n’était fondée que sur a seule anxiété
schémas de pensée ou de comportements ressentie. C’est un exemple de pensée néfaste:
néfastes, qui renforcent les sentiments le raisonnement émotionnel.
d’anxiété et de dépression. Par exemple, Parmi ces schémas de pensée
quelqu’un d’anxieux dans un lieu très appelés aussi des distorsions cognitives,
fréquenté pourra penser: «comme je suis on trouve le raisonnement tout-ou-rien,
nerveux, c’est que a situation est quand nous voyons tout en banc ou
dangereuse», et décider ainsi de rentrer chez noir. Par exemple, «Si ce que je fais n’est
lui et d’éviter les foules à l’avenir, ce qui pas parfait, c’est un échec total». Un autre
renforcera son sentiment de solitude ou de se consiste à écarter systématiquement les
sentir étouffé par l’angoisse. aspects positifs, trouver toujours des raisons
Cette tendance à nous retrouver coincés de les minimiser et tendre au contraire à
dans des schémas de pensée néfastes est ruminer et magnifier les aspects négatifs. En
au cœur de nombreuses thérapies, dont a évoquant ces schémas en thérapie, le patient
thérapie reationnelle émotive (TRE) conçue par se trouvera mieux à même de les identifier
Albert Ellis. Dans cette thérapie, le patient et le quand il y succombera et pourra ainsi choisir
thérapeute œuvrent à identifier les croyances de répondre différemment à ces situations.

136 THÉRAPIE
AFFRONTER LA PEUR
Les phobies spécifiques, celles des araignées, à a fois. Elles sont par ailleurs renforcées par
du sang ou de l’avion, sont des troubles nos modes de pensée et nos comportements.
fréquents. Les gens atteints savent le plus Heureusement, les psychothérapies
souvent que ce sont des peurs irrationnelles, fonctionnent plutôt bien contre les phobies
mais cea ne les empêche pas de ressentir de en général, au point qu’une seule session
a peur dans ces situations. Les phobies ont peut parfois suffire au patient pour faire des
des causes génétiques et environnementales progrès très significatifs.
Le comporemen
modiie--i es
pensées?
Nos pensées, nos émotions et nos
comportements sont tous liés. Ainsi, nous
pouvons influencer les unes en modifiant
les autres, un pouvoir qui bénéficie à tous.

Nombre d’entre nous ont déjà remis réguliers dans nos façons de pensée et nos
quelque chose à plus tard, par exemple comportements, nous pouvons obtenir des
se mettre au sport ou débuter un progrès significatifs et durables. Le patient et
nouveau projet. Nous avons manqué de le thérapeute y œuvrent ensemble à trouver
motivation et, comme cea s’est reproduit jour des modifications de comportement qui
après jour, nous avons fini par nous sentir battront en brèche les croyances négatives
coupables, le moral en berne, mécontents de responsables de l ’anxiété ou du mauvais
nous-mêmes. Hugh Kearns et Maria Gardiner (2011) moral du patient. Si, par exemple, celui-ci
décrivent cette attitude par l’expression est sujet à des troubles du stress post-
«attendre qu’apparaisse a fée Motivation» – traumatique après un accident de train, il se
espérer qu’un jour a motivation nous viendra pourrait qu’il ait peur de reprendre ce moyen
comme par magie pour nous pousser à l’action. de transport, craignant un nouvel accident,
Mais a fée n’existe pas et les études montrent que et même qu’il évite tout transport public,
c’est l’inverse qui se produit, en réalité. Débuter ce qui lui interdit de voir certains amis. Une
une tâche génère a motivation de continuer et expérience comportementale, dans ce cas-là,
d’en faire plus. Modifier notre comportement et pourrait consister en quelques changements
nous ancer même quand nous n’en avons pas courageux pour réduire cet évitement, par
envie pourra avoir un effet très positif sur notre exemple prendre le métro en présence de
moral et nos pensées. son thérapeute pour une ou deux stations.
Ce lien entre pensées, émotions En modifiant ainsi son comportement, il
et comportements est au centre de a constate que ses prédictions catastrophistes
thérapie cognitive comportementale (TCC), n’ont pas tant de chance de se réaliser. Il
développée initialement par Aaron Beck. En fait ainsi des progrès significatifs en vue de
effectuant des changements modestes, mais surmonter ses difficultés.

138 THÉRAPIE
COMPORTEMENTS DE SÉCURITÉ

Les patients atteints de phobie sociale ont se sentir en sécurité dans les situations
souvent l’ impression qu’on les dévisage. Afin angoissantes. Cependant, ils empirent
d’ éviter cette sensation, ils gardent souvent a souvent les choses. En se ançant dans une
tête basse et évitent de croiser les regards. Ces TCC, on découvre une vérité puissante: les
stratégies sont appelées des comportements autres ne les dévisagent pas, cette sensation
de sécurité, car on les met en œuvre pour n’est qu’une illusion que l’anxiété provoque.
CHAPITRE 8 PSYCHOLOGIE POSITIVE

ESTIME
DE SOI

PIC DE
DIVERSITÉ PERFORMANCE
DES
ÉMOTIONS

FLOW PSYCHOLOGIQUE

MENTALITÉ
DE
CROISSANCE
CHAPITRE 8

POSITIE
PSYCHOLOGIE
TRAITS DE
PERSONNALITÉ

BONHEUR

RÉCOMPENSE
INTRODUCTION

B
eaucoup d’entre nous estiment que gagner au
loto leur apporterait le bonheur instantané. Mais
en réalité le bonheur n’a pas grand-chose à voir
avec le contenu d’un compte en banque. Ce
chapitre s’intéresse aux ingrédients clés de a vie heureuse
que a psychologie a identifiés au fil des années, aux
raisons de leur importance et aux façons de les rassembler.
D’abord, il nous faut une robuste ESTIME DE SOI.
Autrement dit, il nous faut nous forger une opinion positive
de nous-mêmes. Comme les opinions sont des pensées,
et non des faits, il est possible d’améliorer notre estime de
soi en modifiant nos modes de pensée. Le renforcement
d’attitudes saines peut nous y aider énormément, par
exemple trouver paisir aux nouveaux défis, croire en nos
chances de succès et nous juger dignes de mérite.
Les gens dotés d’une bonne estime de soi tendent à
avoir plus de succès dans les domaines qu’ils explorent,
ils sont même en meilleure santé, tandis que ceux qui ont
du mal à se voir de façon positive, surtout les adolescents,
ont plus de chance de développer des troubles. Le BIAIS
D’AUTO-COMPLAISANCE peut nous aider à protéger et
établir notre estime de soi. Il s’agit de s’attribuer les mérites
et de renvoyer les fautes sur les autres. Les psychologues
estiment qu’il faut du temps pour être bien dans sa peau,
que l’estime de soi se stabilise autour de a soixantaine, une
fois acquises a sagesse et des habitudes de pensée saines.

142 PSYCHOLOGIE POSITIVE


Le deuxième ingrédient clé est d’assurer en nous une
DIVERSITÉ DES ÉMOTIONS, aller chercher des opportunités
de ressentir des émotions heureuses. Une MENTALITÉ DE
CROISSANCE est importante aussi. Il s’agit de croire en
l’importance de l ’effort et de a pratique, davantage qu’au
talent inné, et de saisir plutôt qu’éviter les opportunités
de défis nouveaux. Les parents contribuent à a mentalité
de croissance de leurs enfants en récompensant leurs
efforts plutôt qu’en louant leur talent. Les avantages de
cette mentalité sont qu’elle nous protège dans les mauvais
moments et qu’elle se développe avec a pratique. Des
études portant sur des familles modestes ont découvert
que les enfants qui en étaient pourvus obtenaient de
meilleurs résultats que les autres de même condition, mais
de dispositions différentes.
On est plus heureux quand on est absorbé dans une
tâche, quelle qu’elle soit, répondre à notre courrier ou faire a
vaisselle: c’est a découverte de MIHALY CSIKSZENTMIHALYI.
On l’est même encore plus quand a tâche est assez complexe
pour être engageante, mais pas au point de s’en trouver
dépassé. S’y absorber empêche l’esprit de vagabonder et fait
affluer les sentiments de récompense.
Tous les ingrédients qui aissent en pace les émotions
heureuses sont également importants pour ce que les
psychologues appellent l’épanouissement. S’entraîner à
être heureux augmente les chances de s’épanouir.

PSYCHOLOGIE POSITIVE 143


PSYCHOLOGIE
Glossary POSITIVE

INVENTAIRE D’ESTIME DE SOI MODÈLE PERMA™ DE SELIGMAN


DE COOPERSMITH Ensemble de cinq briques prouvées
Questionnaire développé pour pour promouvoir l’épanouissement:
mesurer l’estime de soi des enfants émotions positives, engagement,
dans le contexte parental, scolaire, liens, sens et accomplissement.
amical et personnel en cherchant
les signes primaires, cognitifs ou
comportementaux, de l’estime de
soi.

PSYCHOLOGIE
POSITIVE
Se concentrer sur les
expériences et les concepts
ESTIME DE SOI positifs et enrichissants tels
Opinion que nous tenons de qu’une mentalité de croissance,
nous-mêmes, peut s’améliorer en la formation d’un esprit heureux,
changeant de mode de pensée. l’épanouissement, l’estime
Protégée par le biais d’auto- de soi et la diversité
complaisance. des émotions.

BIAIS D’AUTO-
COMPLAISANCE
DURÉE DE VIE EN BONNE SANTÉ S’attribuer les succès et rejeter
Durée de vie sans maladie invalidante. sur autrui les responsabilités
Concept issu de l‘allongement de des échecs, protégeant ainsi
la durée de vie et de l’augmentation l’estime de soi.
de la qualité de vie.

MENTALITÉ FIGÉE
Mode de pensée inflexible;
rester dans les domaines
maîtrisés et éviter les
nouveaux défis.

DIVERSITÉ DES ÉMOTIONS


Vivre une abondance d’émotions positives,
clés du bien-être, d’un niveau de stress
réduit et d’une meilleure santé.
ÉPANOUISSEMENT
MARQUEURS
PHYSIOLOGIQUES
Changements mesurables
tels que la pression sanguine,
l’amplitude du souffle, la
pulsation cardiaque, l’activation
de certains groupes
de muscles. PIC DE PERFORMANCE
Quand des comportements
optimaux se combinent pour
FLOW PSYCHOLOGIQUE produire une performance
Capacité universelle des d’exception.
humains qui exige de la
positivité et une attention
prolongée, sans effort apparent,
clé d’une vie accomplie
(Csikszentmihalyi).

COMPOSANTES DU MIHALY CSIKSZENTMIHALYI


BONHEUR D’ARGYLE Psychologue (1934–2021) qui a
Trois facteurs clés du bonheur, aux voyagé partout dans le monde
aspects cognitifs et émotionnels: pour étudier le bonheur et la
la satisfaction générale, créativité et a inventé le terme
la présence d’affects positifs «flow psychologique».
et l’absence d’affects négatifs.

TRAITS DE PERSONNALITÉ
Attributs qui distinguent
un individu, souvent positifs
ou négatifs et stables dans
le temps, donc difficiles à
MENTALITÉ DE CROISSANCE
FLOW

modifier.
Être ouvert au changement et
aux opportunités; accueillir
les nouveaux défis; apprendre
de ses échecs; croire aux vertus
de l’effort et de la pratique.
Qe e-os?
Vous pourriez répondre en fonction de
l’estime en aquelle vous vous tenez. Si vous
pensez valoir autant que les autres, être
«bien pourvu» en qualités et assez heureux
pour sortir de votre zone de confort, alors
vous avez une bonne estime de soi.

Qui vous vient à l’esprit quand vous Plus récemment, des études à grande échelle
pensez à quelqu’un pourvu d’une à partir de a naissance et jusqu’à l’âge adulte
bonne estime de soi? Est-ce une ont montré que les chutes dans l’estime de
personne surhumaine ou simplement soi à l’adolescence pouvaient entraîner des
quelqu’un capable de se voir positivement? risques pour a santé physique et mentale, des
L’estime de soi est l’opinion que nous avons de comportements à risque et de moins bonnes
nous-mêmes. Et comme les opinions sont des chances de réussite sociale. Une mauvaise
pensées et non des faits, cette estime peut estime de soi enclenche un cercle vicieux
changer en modifiant nos pensées. Parmi les d’expériences malheureuses, de mauvais
indicateurs d’une bonne estime de soi, on résultats qui se dégradent de plus en plus et
trouve a confiance d’exprimer ses opinions, de détresse générale.
a volonté de tenter de nouvelles expériences, Heureusement, certains de nos biais
a foi en nos succès futurs et le sentiment de cognitifs nous aident à conserver une saine
valoir au moins autant que les autres. estime de soi. Ainsi, grâce au biais d’auto-
La recherche en ce domaine a débuté compaisance, nous avons tendance à
à a fin des années 1960 dans les écoles nous attribuer les succès et à assigner a
américaines où enseignants et chercheurs responsabilité des échecs à autrui. Quand
ont réalisé les bénéfices à tirer d’une école quelque chose tourne bien, nous nous
qui améliorerait l’estime de soi des élèves racontons que nous y sommes pour quelque
plutôt que de se focaliser sur leurs résultats. chose, quand cea tourne mal, nous cherchons
L’inventaire de Coopersmith a été développé d’autres explications. La bonne nouvelle,
pour mesurer l’estime de soi des enfants dans c’est que l’estime de soi, une fois passée
le contexte amical, parental, scoaire et intime, l’adolescence, peut s’améliorer en continu
à l’aide de questionnaires visant à déceler des au fil des bonnes expériences jusqu’à a
signes cognitifs ou comportementaux très soixantaine, âge auquel elle se stabilise pour
simples d’estime de soi. ne retomber ensuite qu’autour de 90ans.

146 PSYCHOLOGIE POSITIVE


MESURE DE L’ESTIME DE SOI

?
?
?

Nous mesurons l’estime de soi en posant des


questions. Nous demandons par exemple à
un enfant s’ il pense bien réussir à l’ école, s’ il
estime se faire facilement des amis ou s’ il
pense que ses parents le chérissent. À l’ âge
adulte, les questions portent sur a fierté qu’on
tire de son travail et autres succès, ou sur les
amis et a famille.
Qe es e rccorci
ers e bonher?
À court terme, essayez de vous mettre
un crayon en travers des dents, mais à long
terme, d’autres stratégies fonctionnent mieux.
La science psychologique a beaucoup à dire
sur a manière de déclencher et préserver
des émotions heureuses.

Si vous cherchez un raccourci vers le Nous savons qu’au-delà d’un certain seuil
bonheur ou, au moins, un point de de sécurité, l’argent ne fait plus le bonheur.
départ, essayez déjà ça. Tout ce qu’il Nous savons aussi que nous sommes de plus
vous faudra, c’est un crayon ou un stylo. Des en plus heureux avec l’âge. Que pouvons-
études ont montré que mettre un crayon entre nous changer pour trouver un raccourci vers
ses dents active les mêmes muscles qu’un le bonheur? Des travaux en neuroscience
sourire, ce qui rend plus heureux et plus allègre. nous ont fourni des indices et suggèrent que
Cet effet se dissipe toutefois dès qu’on l’enlève, a formule d’une vie heureuse contient une
et ne marcherait bien que pour les personnes part d’attentes remplies ou dépassées. Il ne
déjà heureuses à a base. Pour une solution plus s’agit pas de savoir si cea se passe bien pour
pérenne, mieux vaudra donc vous concentrer sur nous, mais si cea se passe mieux que prévu.
les ingrédients clés du bonheur. Des recherches plus récentes indiquent
Michael Argyle a reconnu l’importance que a clé du bien-être consiste à se
du bonheur, ou du «bien-être subjectif» (à concentrer sur a variété des émotions
partir de recherches dans les années1980), ressenties au quotidien plutôt que sur le
alors qu’il existait dix-sept fois plus d’études bonheur en lui-même. Vivre une abondance
sur a dépression que sur le bonheur. Les d’émotions positives, telles que l’intérêt, le
composantes du bonheur, selon Argyle, calme, a fierté, a gratitude ou l’inspiration
contiennent des aspects cognitifs et réduit le stress et les infammations, ce qui
émotionnels distincts. Les trois facteurs clés contribue à une meilleure santé et donc à
sont a satisfaction générale (vie sociale, vie davantage de raisons de sourire. Il n’y a peut-
professionnelle et loisirs), a présence d’affects être pas de raccourci, mais de nombreux
positifs et l’ absence d’affects négatifs. chemins mènent au bonheur!

148 PSYCHOLOGIE POSITIVE


DIVERSITÉ DES ÉMOTIONS POSITIVES

Se mettre un crayon entre les dents


déclenchera un sourire, ce qui ouvre un
raccourci vers le bonheur. Cependant,
pour un résultat à long terme, cherchez
des activités déclenchant une variété de
sensations paisantes, comme apprendre une
nouvelle recette ou résoudre un problème
avec l’aide de collègues. Nous sommes plus
heureux quand nous sommes créatifs et que
nous tissons des liens.
Chner de menié
chne--i ’espri?
Oui! Passer d’une mentalité figée à
une mentalité de croissance transformera
non seulement votre mode de pensées, cea
rendra plus savoureux vos succès,
améliorera vos performances et votre
capacité de rebond après les échecs.

Passer d’une mentalité figée à une étude portant sur un grand nombre de familles
mentalité de croissance est du Chili a découvert que les enfants issus de
excellent sur le pan professionnel, familles modestes avaient moins de chance
familial ou scoaire. Carol Dweck estime que d’exprimer une mentalité de croissance.
changer sa mentalité revient à remettre en Pourtant, chez ceux qui le faisaient, elle agissait
question ses croyances sous-jacentes sur comme une protection contre l’impact négatif
l’apprentissage et l’intelligence. des faibles revenus sur le succès. La mentalité
Que diriez-vous si on vous proposait de croissance peut ainsi être un vrai facteur
d’apprendre une discipline entièrement protecteur dans des circonstances difficiles.
nouvelle, loin de votre zone de confort? Si Surtout, elle s’apprend. Des études
vous répondez «je ne sais pas faire ça, je me montrent qu’enseigner à des enfants qu’ils
contente de ce que je connais», sachez que vont s’améliorer, que leur cerveau va se
ceux qui répondent «je ne sais pas encore développer, tout en les incitant à sortir de leur
faire ça, mais je vais essayer» montrent en zone de confort, sont a clé d’une croissance
général plus de résilience après un échec. harmonieuse. Comparés à des enfants
Ils apprennent de leurs erreurs plutôt que persuadés que leurs capacités sont gravées
d’abandonner et connaissent ainsi plus de dans le marbre, ceux élevés dans a mentalité
succès. Dans le cadre d’une mentalité de de croissance obtiennent de meilleures notes
croissance, l’effort et a pratique valent au et sont plus motivés. Récompenser l ’effort
moins autant que le talent inné. plutôt que les capacités crée de a confiance
L’idée de a mentalité de croissance est et de a persistance. Adopter cette mentalité
importante pour a psychologie positive parce dans un environnement difficile, face à un
qu’elle remet en cause l ’idée que l’intelligence, défi ou à un échec, c’est se concentrer sur
l’éducation et le revenu sont des éléments l’après, aller de l ’avant, savoir affronter de
indispensables au succès. Par exemple, une nouveaux défis et apprendre sans cesse.

150 PSYCHOLOGIE POSITIVE


MENTALITÉ DE CROISSANCE
Changer de mentalité influence vos croyances, vos efforts,
votre approche des défis et votre réaction face à l’ échec. Une
mentalité de croissance permet de se concentrer sur l’avenir,
affronter de nouveaux défis et atteindre leur plein potentiel.
Pe-on s’épnoir
rce   pschooie?
Pour vivre une vie heureuse, survivre ne
suffit pas: il faut prospérer. La psychologie
nous montre comment atteindre notre plein
potentiel et transformer une vie ordinaire en
un parcours extraordinaire.

Nous savons tous que le stress, les l’épanouissement. La théorie PERMA™


troubles mentaux, les inégalités s’appuie sur a hiérarchie des besoins de
sociales et les traumas nuisent au Maslow. Ce n’est qu’une fois posées les
bien-être. Mais saviez-vous que nous en briques élémentaires de a vie (sécurité, abri,
payons aussi le prix sur le pan de a longévité, nourriture, eau) que les briques de niveau
de l’espérance de vie? Les avancées de a plus élevé (estime de soi, besoins cognitifs
médecine l ’ont tant allongée que le concept et esthétiques, auto-réalisation) viennent
s’est aujourd’hui éargi pour inclure au centre de l’attention d’un individu ou
l’espérance de vie en bonne santé. Comment d’une communauté. Les cinq facteurs
garantir a qualité de cette quantité d’années qui contribuent à l’épanouissement sont
supplémentaires obtenues? Comment vivre les émotions positives, l’engagement, les
une bonne vie? reations, le sens et l’accomplissement.
La recherche en psychologie nous en Nous savons que ce que nous mesurons
apprend beaucoup sur a recette d’une vie influence nos actions. Si nous nous
extraordinaire. Plutôt que de se concentrer concentrons uniquement sur les indicateurs
sur l’élimination ou le dépassement de a cassiques du succès, tels que l’argent, a
souffrance et de a détresse, elle s’intéresse propriété individuelle, les notes obtenues
à présent surtout à a manière de s’épanouir. à l’école ou a productivité politique ou
L’épanouissement consiste à atteindre économique, a théorie PERMA™ estime que
son plein potentiel, à transformer une vie nous ratons entièrement des aspects critiques
ordinaire en un parcours extraordinaire. de l’épanouissement. Tout comme
Il signifie vivre nos vies avec détermination. a recherche médicale ne se contente pas de
Les développements de Martin Seligman viser à l’allongement de a durée de vie,
dans le domaine de a psychologie positive les chercheurs en psychologie doivent à leur
incluent un ensemble de clés, fondées tour intégrer le nombre d’années passées
sur des études solides, pour atteindre à s’épanouir.

152 PSYCHOLOGIE POSITIVE


CLÉS DE L’ÉPANOUISSEMENT

Accomplissements

Sens

Relations

Engagement

Pour nous épanouir, il nous faut d’abord


remplir nos besoins de sécurité, de logement
et d’alimentation. Nous pouvons ensuite tenter
d’aligner notre vie sur nos valeurs, le sens que
nous lui donnons. S’engager dans des reations
profondes, se créer des occasions de se sentir
bien et trouver du sens à nos actions sont
parmi les clés de l’ épanouissement. Émotions positives
Doi-on se isser
porer pr e o?
Si vous voulez atteindre votre pic de
bien-être et de performance, a réponse est
oui! Le psychologue Mihaly Csikszentmihalyi
a noté que le flow exigeait de a positivité
et une attention soutenue, et qu’il pourrait
être a clé d’une vie qui mérite d’être vécue.

Quand avez-vous été totalement Le pianiste de talent se sentira alerte, mais


absorbé dans une activité pour a calme, confiant, mais stimulé, et totalement
dernière fois? Un moment où vous absorbé dans le moment présent. Il produira
avez repoussé vos limites, atteint votre pic et où une performance musicale exceptionnelle.
vous vous êtes senti mieux que jamais? Mihaly Nous pourrions être tentés de chercher
Csikszentmihalyi a voyagé partout dans le à atteindre le flow pour augmenter notre
monde pour poser cette question. En analysant productivité, mais les études montrent que
cette expérience chez des musiciens, des l’expérience est bénéfique en elle-même.
chirurgiens, des danseurs, des agriculteurs, des L’effort soutenu pour atteindre le flow mène
bergers et des ouvriers, il a conclu que le à a satisfaction et au bonheur.
«flow» psychologique était une capacité Le flow et le pic de performance ne
universelle des humains et qu’elle était a clé sont pas seulement associés à a musique.
d’une vie vécue à plein. L’état de flow, appelé Certains l’atteignent dans le sport, au
parfois a «zone», est un méange d’attention travail, dans les loisirs créatifs, les jeux, a
totale et sans effort et d’émotions positives. religion ou a spiritualité, l’apprentissage. Les
Que se passe-t-il dans le corps et dans individus consciencieux tendent à l’atteindre
l’esprit d’un pianiste qui livre sa meilleure plus souvent, tandis que d’autres traits de
performance? Des études ont mis en évidence personnalité ont tendance à le bloquer.
des variations mesurables de certains La mauvaise humeur, par exemple, l’empêche.
marqueurs physiologiques durant le flow. Nous avons plus de chance de l’atteindre
Parmi eux, on trouve a pression sanguine, le quand nous sommes absorbés dans une
volume du souffle, a pulsation cardiaque et tâche assez exigeante pour être stimuante,
l’activation des groupes de muscles du sourire. mais pas au point de devenir insurmontable.
Si le flow désigne un état mental requis, le pic Même s’il existe de nombreuses barrières qui
de performance décrit lui les comportements nous empêchent d’accéder à a zone, le jeu en
optimaux à atteindre. vaut a chandelle.

154 PSYCHOLOGIE POSITIVE


PIC DE PERFORMANCE ET FLOW
Le flow mental est une immersion dans de sa vie sera dans le flow au pic de celle-ci.
l’activité en cours. C’est un méange Être dans le flow, ou dans a «zone», c’est
d’attention renforcée, d’ émotions agréables comme se aisser emporter par le courant,
et d’une absence complète d’embarras. Un se détendre et profiter du voyage plutôt que
pianiste en train de livrer a performance d’angoisser sur sa destination.
POUR EN SAVOIR PLUS

LIVRES JOURNAUX
Beck, A.T., et al. Cognitive Therapy of Borke, H. (1975). Piaget’s mountains revisited.
Depression. New York, Guilford Press, 1979 Developmental Psychology, 11(2), 240–243

Bond, M. Wayfinding: The Art and Science Bruner, J. (1981). The social context of anguage
of How We Find and Lose Our Way. Londres, acquisition. Language & Communication,
Picador, 2020 1(2), 155–178

Corsini, R. & Wedding, D. Current Dunn, J. (1994). Understanding others and the
Psychotherapies. Totnes, Brooks/Cole social world: Current issues in developmental
Publishing Company, 2018 research and their reation to preschool
experiences and practice.Journal of Applied
Deutsch Lezak, M., et al. Neuropsychological
Developmental Psychology, 15(4), 571–583
Assessment. Oxford, Oxford University
Press, 2012 Favell, J.H. (1996). Piaget’s legacy.
Psychological Science, 7(4), 200–203
Dunn, J. The Beginnings of Social
Understanding. Cambridge (Massachusetts), Kearns, H. & Gardiner, M. (2011). Waiting for
Harvard University Press, 1988 the motivation fairy. Nature, 472, 127

Erikson, E. Identity, Youth, and Crisis. Nader, K., Schafe, G.E. & LeDoux, J.E. (2000a).
New York, W. W. Norton, 1968 The abile nature of consolidation theory.
Nature Reviews Neuroscience. 1, 216–219
Erikson, E.H. Childhood and Society.
New York, W. W. Norton & Company, 1993 Wegner, D.M., et al. (1987). Paradoxical effects
of thought suppression.Journal of Personality
Morrison, A.P. (ed.). A Casebook of Cognitive
and Social Psychology, 53(1), 5–13
Therapy for Psychosis. New York, Brunner-
Routledge, 2002
RESSOURCES EN LIGNE
Seth, A. Being You. Londres, Faber & www.simplypsychology.org
Faber, 2021
www.nationalelfservice.net
Sater, A. & Bremner, J. An introduction to
Developmental Psychology. Londres, BPS Every Mind Matters
Backwell. 3eédition, 2017 www.nhs.uk/every-mind-matters

Mind www.mind.org.uk
Wild, J. Be Extraordinary: 7 Key Skills to
Transform Your Life From Ordinary to
Extraordinary. Boston, Little Brown Book
Group, 2020

156
NOTES SUR LES CONTRIBUTEURS

DIRECTRICE DE PUBLICATION Aimee McKinnon


Aimee McKinnon est psychologue clinicienne
Dr Jennifer Wild
et chercheuse. Dans ce cadre, elle s’est tournée
Jennifer Wild est professeure à l’Université
vers a prévention et les interventions précoces
de Melbourne et professeure affiliée de
contre les troubles du stress post-traumatique.
l’Université d’Oxford, où elle développe à
partir d’études scientifiques des interventions
Helen Pilcher
visant à prévenir les troubles du stress post-
Helen Pilcher est docteure en biologie
traumatique dans les services de secours,
celluaire du London’s Institute of Psychiatry,
utilisées partout dans le monde. Elle a écrit
diplômée de psychologie et de neurosciences.
pour Nature News, a BBC, le Times , le Sunday
Elle a été journaliste pour Nature, admise au
Times et le Psychologist. Elle applique
programme Royal Society’s Science in Society.
les sciences à a compréhension et à a
Aujourd’hui, elle écrit et parle de sciences. Elle
prévention des troubles mentaux.
a écrit de nombreux livres et articles pour le
Guardian, New Scientist et Science Focus.
ILLUSTRATEUR
Robert Brandt Dr Graham Thew
Depuis le Royaume-Uni, Robert Brandt Le Dr Graham Thew est psychologue clinicien
utilise le graphisme pour a communication et chercheur de l’Université d’Oxford et de
depuis plus de vingt ans. Il s’est spécialisé l’Oxford Health NHS Foundation Trust. Il assure
dans l’illustration technique et scientifique, des psychothérapies scientifiques pour les
pour des sujets alant de l’astrophysique à a adultes présentant des troubles de l’anxiété
biochimie. Il travaille avec des experts pour ou dépressifs, et mène des recherches sur
rendre les sujets complexes accessibles au l’application des technologies numériques aux
grand public dans l’édition, l’industrie et thérapies pour aider les patients à bénéficier le
l’éducation. plus possible du traitement.

CONTRIBUTEURS Gabriela Tyson


Gabriela Tyson suit une thèse à l ’Université
Michael Bond
d’Oxford sur a prévention des troubles
Michael Bond est un auteur spécialisé dans
mentaux dans les popuations à risque.
a psychologie et le comportement. Il a été
rédacteur en chef du New Scientist .
Trinity de Simone
Trinity de Simone est assistante-chercheuse
Sophie Grant
diplômée à l’OxCADAT. Son équipe est
Sophie Grant est assistante de recherche
spécialisée dans a recherche et le traitement
au Oxford Centre for Anxiety Disorders
des troubles du stress et de l’anxiété.
and Trauma (OxCADAT). Elle travaille sur
l’application de thérapies cognitives en ligne,
Abbie Wilkins
pour soigner l’anxiété sociale et les troubles
Abbie Wilkins est maître de conférences
du stress post-traumatique, au sein du NHS.
en psychothérapies à l’Université d’Exeter,
spécialisée dans les troubles du stress post-
traumatique complexes à l’OxCADAT.

157
INDEX

A Csikszentmihalyi, Mihaly 143, 154


cube de Necker 15
F
accommodation (apprentissage) 62 familles 94, 100, 101
cycle de l’expérience 123
adaptation 150 faux souvenirs 11, 18, 20
cycle perpétuel (de a perception) 18
affect 76 flow 145, 154–155
affordance 16 Flynn, James 106, 114
Ainsworth, Mary 88, 90, 94, 95 D Folkman, Susan 132
antidépresseurs 80 défis 143, 150 fratrie 101
anxiété 92, 134, 136, 139 dépression 92

anxiété sociale 139 désensibilisation systématique 127


développement cognitif 51, 52–53,
G
aphantasie 82, 83 Gardner, Howard 116, 117
apprentissage 48–67, 78, 84–85 62–63, 96–99
gestion du stress 127, 132–133
Argyle, Michael 148 développement psychosocial
Gibson, James J. 16, 17
Asch, Solomon 30, 38, 39 100–105
Gadwell, Malcolm 51, 60
assimiation 62 différences individuelles 106–123
Goleman, Daniel 118, 119
Atkinson, Richard 22 discrimination 42–43
Gregory, Richard 14
dispositif d’acquisition du angage

B (DAL) 51, 64–65


dissonance cognitive 13, 24 H
Baddeley, Aan 11, 22 Helmholtz, Hermann von 14
distorsion cognitive 136
Bandura, Albert 58 heuristique 27
diversité émotionnelle 143, 148
Beck, Aaron 127, 128, 138 hiérarchie des besoins de Maslow
drogues 71, 80
béhaviorisme 50, 54–7, 127 112, 152
Dunn, Judith 90, 100, 101
besoins 109 hippocampe 71, 84
Dweck, Carol 150
biais d’auto-compaisance 142, 146 Holmes, Thomas 92, 93
biais d’interprétation 77
biais de l’endogroupe et de E hypothèses 10, 14–15

échelle de réajustement social (ERS)


l’exogroupe 42, 43
biais inconscients 26 92, 93 I
écoute réflective 131 idées scientifiques 44–5
bien-être 148
effet de désinformation 21 identité 31, 32, 42–43
big five 107, 109
effet du témoin 30, 36–37 identité sociale 31, 32, 42–43
boîte de Skinner 57
effet Flynn 106, 114 illusion 12–13, 14, 15
bonheur 143, 145, 148–149
égocentricité 96 imitation 58
Bowlby, John 90, 94
Ekman, Paul 30, 32, 34 incongruence 130
Bruner, Jerome 98, 99
Ellis, Albert 136 inférence inconsciente 14
influence du groupe 39, 42–43
C émotions 34, 70, 76, 136
épanouissement 152–153 influences culturelles 33, 34
carte cognitive 84 influences sociales 33
équilibrioception 74
cellules cérébrales 80 intelligence 106–7, 110, 114–17
erreur fondamentale d’attribution
cellules de lieu 84 intelligence créative 114, 115
46–47
Chomsky, Noam 51, 53, 64 intelligence émotionnelle (IE) 107,
estime de soi 142, 146–7
comportements 129 118–119
événements majeurs 89, 92, 93
comportements de sécurité 139 interactions sociales 100
expérience de a poupée Bobo 58, 59
conditionnement cassique 50, 52, interprétation 109
expérience de pression sociale 39
54–55 intuition 26
expérience du choc électrique de
conditionnement opérant 50, 56–57 inventaire d’estime de soi 146
Milgram 31, 40–41
conformisme 38
expérience du groupe minimal 43
conformisme de groupe 30
constructs 109, 122
expressions faciales 30, 34–35 J
James, William 76
crise d’identité 104
jeu 89, 100
croyances fausses 103
Jones, Edward 46, 47

158
K perception visuelle 16
personnalité 106–107, 111, 120–121
stades de a vie 91
stades du développement 104–105
Kahneman, Daniel 26
peur 137 Sternberg, Robert 114, 115
Kanner, A.D. 92
phobies 137 stratégies d’adaptation 132
Kelly, George 122, 123
Piaget, Jean 52, 62, 63, 96 stress 78–79, 90, 92, 113
pic de performance 154, 155 structure cérébrale 121
L préjugés sociaux 42, 43 style attributionnel 134
Lazarus, Richard 127, 129, 132 prise de décision 11, 26 synapses 80, 81
leadership 118–119 proprioception 73, 74 synesthésie 70, 74, 75
LeDoux, Joseph 10, 12, 22 prosopagnosie 71 système nerveux 78
Loftus, Elizabeth 11, 20, 21 psychanalyse 126
loi de Yerkes et Dodson 70, 78, 79 psychobiologie 68–85 T
psychologie cognitive 8–28 Tajfel, Henri 31, 32, 42, 43
M psychologie de l’enfant 86–105 taxi 84, 85
médiation 58 psychologie du développement témoignages ocuaires 20
mémoire 10, 18–23, 60, 66, 84–85, 98 86–105 test du chauffeur de taxi 84, 85
mémoire à court terme (MCT) 22, 23 psychologie positive 140–55 tests de QI 114
mémoire à long terme (MLT) 22, 23 psychologie sociale 30–47 théorie de l’apprentissage social
mémoire de travail 11, 22, 23 psychotropes 80 (TAS) 51, 53, 58
mémoire épisodique 22 théorie de l’attachement 88, 91,
mémoire sémantique 22 R 94–95
mémoire sensorielle 22 Rahe, Richard 92, 93 théorie de l’esprit 89, 102
mentalité 143, 150–151 raisonnement émotionnel 136 théorie de a consolidation 10, 13, 22
mentalité de croissance 143, 150–151 récepteurs 80 théorie des perspectives 26
Miller, George 66, 67 reconnaissance des visages 71, 82–83 théorie des stades d’Erikson 104–105
mnémotechnique 51 règle des 10000heures de Gadwell théorie du développement cognitif
modèle PERMA™ 152 51, 60–61 de Piaget 51, 62–63
modèles mentaux 62 reation thérapeutique 130 théorie du traitement de
Moscovici, Serge 32, 44, 45 renforcement 56–58 l’information 66–67
motivation 106, 112, 138, 150 réponse combat/fuite 78 théorie écologique de Gibson 10,
réponses 72 16–17
N réponses physiologiques 76 thérapie 124–139
N-Ach 112, 113 représentation de a connaissance thérapie centrée sur a personne 130
Nader, Karim 10, 12, 22 98 thérapie cognitive comportementale
Neisser, Ulric 18 représentation symbolique 98 (TCC) 126, 127, 138
neurones 80 représentations sociales 44 thérapie humaniste 126, 130, 131
neurotransmetteurs 71, 80, 81 répression des pensées 135 thérapie rationnelle émotive (TRE)
niveau de performance 60–61, 78, Rogers, Carl 130 127, 136
79, 154, 155 rôles genrés 58 tracas quotidiens 89, 92
traitement ascendant 10, 16–17

O S traitement descendant 10, 14–15

obéir aux ordres 40–41 Sacks, Oliver 82 transmission des idées 44

O’Keefe, John 84 schéma 18, 62 trouble du stress post-traumatique

orientation 84–85 seau du stress 133 138

Seligman, Martin 134, 152 troubles du stress 92

P sens 70, 74 (voir aussi perception) Tversky, Amos 26

parentalité 94–95 sens inconscients 74

Pavlov, Ivan 50, 54–55 sentiments voir émotions W


pensée de groupe 11, 14, 24–25 Shiffrin, Richard 13, 22 Waldeyer, Heinrich 80

pensées 129, 134–135 signal nerveux 81 Watson, John B. 54

pensées négatives 134–136 Skinner, B.F. 50, 52, 56

perception 8–28 (voir aussi sens) spectre d’excitation 72, 78

159
REMERCIEMENTS
Je souhaiterais exprimer mon immense
gratitude envers notre éditrice, Kate Duffy,
qui a appliqué exactement le bon méange
de pugnacité, de conseils et de patience pour
mener ce projet à bien, dans les temps. Je
remercie aussi Katie Crous pour sa relecture
éditoriale et l’attention qu’elle a portée aux
détails. Un grand merci à tous les coauteurs
pour avoir bien voulu partager leurs vivifiants
écairages psychologiques et avoir ainsi rendu
ce livre possible. Je remercie ma sœur, Allison
Wild, pour son expertise et sa capacité à
considérer et discuter les idées. Merci à Mark
Todd et Amy Sedgwick pour les discussions
passionnantes, et pour leur gentillesse. Enfin,
grand merci à mes compagnons d’écriture,
Sami, Zak et Pip.

Jennifer Wild

UniPress Books remercie Robert Brandt pour


ces illustrations lumineuses et Luke Herriott
pour sa maquette si élégante.

Vous aimerez peut-être aussi